Vous êtes sur la page 1sur 64

17.

Vector Calculus with Applications


17.1 INTRODUCTION

In vector calculus, we deal with two types of functions: Scalar Functions (or Scalar Field) and
Vector Functions (or Vector Field).

Scalar Point Function

A scalar function 𝐹(𝑥, 𝑦, 𝑧)defined over some region R of space is a function which associates, to
each point 𝑃(𝑥, 𝑦, 𝑧) in R, a scalar value 𝐹 𝑃 = 𝐹(𝑥, 𝑦, 𝑧) . And the set of all scalars 𝐹(𝑃) for all
values of P in R is called the scalar field over R.

Precisely, we can say that scalar function defines a scalar field in a region or on a space or a curve.
Examples are the temperature field in a body, pressure field in the air in earth’s atmosphere.

Moreover, if the position vector of the point P is 𝑟 , then we may also write the scalar field as
𝐹 𝑃 = 𝐹(𝑟). This notation emphasizes the fact that the scalar value 𝐹(𝑟) is associated with the
position vector 𝑟 in the region R.

E.g. 1) The distance 𝐹 𝑃 of any point 𝑃(𝑥, 𝑦, 𝑧) from a fixed point 𝑃′(𝑥 ′ , 𝑦′, 𝑧′) in the space is a
scalar function whose domain of definition is the whole space and is given by
𝐹 𝑃 = (𝑥 − 𝑥 ′ )2 + (𝑦 − 𝑦 ′ )2 + (𝑧 − 𝑧 ′ )2 . Also 𝐹 𝑃 defines a scalar field in space.

E.g. 2) The function 𝐹 𝑥, 𝑦, 𝑧 = 𝑥𝑦 2 + 𝑦𝑧 + 𝑥 2 for the point (𝑥, 𝑦, 𝑧) inside the unit sphere
𝑥 2 + 𝑦 2 + 𝑧 2 = 1 is a scalar function and also is defines a scalar field throughout the sphere.

Note: In the physical problems, the scalar function F depends on time variable t in addition to the
point P and then we write it as 𝐹 𝑃, 𝑡 = 𝐹 𝑟, 𝑡 = 𝐹(𝑥, 𝑦, 𝑧, 𝑡) . The example of such a time
dependent scalar function is the temperature distribution throughout a block of metal heated in such
a way that its temperature varies with time.

Vector Point Function

A vector functions 𝐹 𝑥, 𝑦, 𝑧 defined over some region R of space is a function which associates, to
each point 𝑃(𝑥, 𝑦, 𝑧) in R, a vector value 𝐹 𝑃 = 𝐹 (𝑥, 𝑦, 𝑧) and the set of all vectors 𝐹 𝑃 for all
points P in R is called the vector field over R

Moreover, if the position vector of the point P is 𝑟 , then we may write the vector field as
𝐹 𝑃 = 𝐹 (𝑟). This notation emphasizes the fact that the vector value 𝐹 (𝑟) is associated with the
position vector 𝑟 in the region R. Also the general form (component form) of the vector function is
𝐹 𝑟 = 𝐹1 𝑟 𝑖 + 𝐹2 𝑟 𝑗 + 𝐹3 𝑟 𝑘 , where the components 𝐹1 𝑟 , 𝐹2 𝑟 and 𝐹3 𝑟 are the scalar
functions.

E.g. 1) The function 𝐹 𝑥, 𝑦, 𝑧 = 2𝑥𝑦𝑖 + sin 𝑥 𝑗 + 3𝑧 2 𝑘 for point P( x, y, z ) inside an ellipsoid


𝑥2 𝑦2 𝑧2
+ + = 1 is a vector function and defines a vector field throughout the ellipsoid.
9 16 4

E.g. 2) The force field given by 𝐹 𝑥, 𝑦, 𝑧 = 𝑥𝑖 + 2𝑦𝑗 + 𝑧 2 𝑘 is a vector field.

1
Note: Like the time dependent scalar field, time dependent vector field also exists. Such a field depends on
time variable t in addition to the point in the region R and may be expressed as
𝐹 𝑟, 𝑡 = 𝐹1 𝑟, 𝑡 𝑖 + 𝐹2 𝑟, 𝑡 𝑗 + 𝐹3 𝑟, 𝑡 𝑘 , where 𝐹1 , 𝐹2 and 𝐹3 are scalar functions. An example of time
dependent vector field is the fluid velocity vector in the unsteady flow of water around a bridge support
column, because this velocity depends on the position vector 𝑟 in the water and the time variable t and is
given as 𝑉 (𝑟, 𝑡).

Vector Function of Single Variable

A vector function 𝐹 of single variable t is a function which assigns a vector value 𝐹 (𝑡) to each
scalar value t in interval 𝑎 ≤ 𝑡 ≤ 𝑏 . In the component form, it may be written as
𝐹 𝑡 = 𝐹1 𝑡 𝑖 + 𝐹2 𝑡 𝑗 + 𝐹3 𝑡 𝑘 where 𝐹1 , 𝐹2 and 𝐹3 are called components and are scalar functions
of the same single variable t.

For example, the functions given by𝐹 𝑡 = 𝑡 𝑖 + sin(𝑡 − 2) 𝑗 + cos 3𝑡 𝑘 and 𝐺 𝑡 = 𝑡 2 𝑖 + 𝑒 𝑡 𝑗 + log 𝑡 𝑘
are vector functions of a single variable t.

Limit of a Vector Function of Single Variable

A vector function 𝐹 𝑡 = 𝐹1 𝑡 𝑖 + 𝐹2 𝑡 𝑗 + 𝐹3 (𝑡)𝑘 of single variable t is said to have a limit


𝐿 = 𝐿1 𝑖+𝐿2 𝑗+𝐿3 𝑘 as 𝑡 → 𝑡0 , if 𝐹 𝑡 is defined in the neighborhood of t0 and Lt 𝑡→𝑡 0 𝐹 𝑡 − 𝐿 = 0
or Lt 𝐹1 𝑡 − 𝐿1 = Lt 𝑡→𝑡 0 𝐹2 𝑡 − 𝐿2 = Lt 𝑡→𝑡 0 𝐹3 𝑡 − 𝐿3 = 0, then we write it as Lt 𝑡→𝑡 0 𝐹 (𝑡) = 𝐿.
𝑡→𝑡 0

Continuity of a Vector Function of Single Variable

A vector function 𝐹 𝑡 = 𝐹1 𝑡 𝑖 + 𝐹2 𝑡 𝑗 + 𝐹3 (𝑡)𝑘 of a single variable t is said to be continuous at


t = t0, if it is defined in some neighborhood of t0 and Lt𝑡→𝑡 0 𝐹 𝑡 = 𝐹 𝑡0 .

Moreover, 𝐹 𝑡 is said to be continuous at 𝑡 = 𝑡0 if and only if its three components F1, F2 and F3
are continuous as 𝑡 = 𝑡0 .

DIFFERENTIAL VECTOR CALCULUS

17.2 DIFFERENTIATION OF VECTORES

Differentiability of a Vector Function of Single Variable

A vector function 𝐹 𝑡 = 𝐹1 𝑡 𝑖 + 𝐹2 𝑡 𝑗 + 𝐹3 (𝑡)𝑘 of a single variable t defined over the interval


𝑎 ≤ 𝑡 ≤ 𝑏 is said to be differentiable at t = t0 if the following limit exists.
𝐹 𝑡 −𝐹 (𝑡 0 )
Lt 𝑡→𝑡 0 = 𝐹 ′ (𝑡0 )
𝑡−𝑡 0

And 𝐹 ′ (𝑡0 ) is called the derivative of 𝐹 𝑡 at t = t0.


Also 𝐹 𝑡 is said to be differentiable over the interval 𝑎 ≤ 𝑡 ≤ 𝑏, if it is differentiable at each of the
points of the interval. In component form, 𝐹 𝑡 is said to be differentiable at t = t0 if and only if its
three components are differentiable at t = t0. In general, the derivative of 𝐹 (𝑡) is given by

2
𝐹 𝑡+∆𝑡 −𝐹 (𝑡)
𝐹′ 𝑡 = Lt 𝑡→𝑡 0 , provided the limit exists and in terms of components
∆𝑡
𝑑𝐹 𝑑𝐹1 𝑑𝐹2 𝑑𝐹3
𝐹 ′ 𝑡 = 𝐹1′ 𝑡 𝑖 + 𝐹2′ 𝑡 𝑗 + 𝐹3′ (𝑡)𝑘 or = 𝑖+ 𝑗+ 𝑘.
𝑑𝑡 𝑑𝑡 𝑑𝑡 𝑑𝑡
𝑑 2𝐹 𝑑 𝑑𝐹 𝑑 3𝐹 𝑑 𝑑 2𝐹 𝑑2 𝑑𝐹
In the similar manner, = , = = .
𝑑𝑡 2 𝑑𝑡 𝑑𝑡 𝑑𝑡 3 𝑑𝑡 𝑑𝑡 2 𝑑𝑡 2 𝑑𝑡

Rules for Differentiation of Vector Functions

If 𝐹 𝑡 , 𝐺 𝑡 & 𝐻 𝑡 are the vector functions and 𝑓(𝑡) is a scalar function of single variable 𝑡 defined
over the interval 𝑎 ≤ 𝑡 ≤ 𝑏, then

𝑑𝐶
1. = 0, where 𝑐 is a constant vector.
𝑑𝑡
𝑑 𝐶𝐹 𝑡 𝑑𝐹
2. =𝐶 , where C is a constant.
𝑑𝑡 𝑑𝑡
𝑑 𝐹 𝑡 ±𝐺 (𝑡) 𝑑𝐹 𝑑𝐺
3. = ±
𝑑𝑡 𝑑𝑡 𝑑𝑡
𝑑 𝑓 𝑡 𝐹 𝑡 𝑑𝐹 𝑑𝑓
4. =𝑓 𝑡 + 𝐹 (𝑡)
𝑑𝑡 𝑑𝑡 𝑑𝑡
𝑑 𝐹 𝑡 ∙ 𝐺 (𝑡) 𝑑𝐹 𝑑𝐺
5. = ∙ 𝐺 (𝑡) + 𝐹 (𝑡) ∙
𝑑𝑡 𝑑𝑡 𝑑𝑡
𝑑 𝐹 𝑡 ×𝐺 (𝑡) 𝑑𝐹 𝑑𝐺
6. = × 𝐺 (𝑡) + 𝐹 (𝑡) ×
𝑑𝑡 𝑑𝑡 𝑑𝑡
𝑑 𝑑𝐹 𝑑𝐺 𝑑𝐻
7. 𝐹 𝑡 , 𝐺 𝑡 , 𝐻 (𝑡) = 𝑑𝑡
, 𝐺 𝑡 , 𝐻 (𝑡) + 𝐹 𝑡 , 𝑑𝑡
, 𝐻 (𝑡) + 𝐹 𝑡 , 𝐺 𝑡 , 𝑑𝑡
𝑑𝑡
𝑑 𝑑𝐹 𝑑𝐺 𝑑𝐻
8. 𝐹 𝑡 × 𝐺 𝑡 ×𝐻 𝑡 = × 𝐺 𝑡 × 𝐻 (𝑡) + 𝐹 𝑡 × × 𝐻 (𝑡) + 𝐹 𝑡 × 𝐺 𝑡 ×
𝑑𝑡 𝑑𝑡 𝑑𝑡 𝑑𝑡

9. If 𝐹 𝑡 is differentiable function of 𝑡 and 𝑡 = 𝑡(𝑠) is differentiable function then


𝑑𝐹 𝑑𝐹 𝑑𝑡
= .
𝑑𝑠 𝑑𝑡 𝑑𝑠

Observations:
𝑑𝐹 2
(i) If 𝐹 (𝑡) has a constant magnitude, then 𝐹 ∙ 𝑑𝑡
= 0. For 𝐹 𝑡 ∙ 𝐹 𝑡 = 𝐹 (𝑡) = 𝑐𝑜𝑛𝑠𝑡𝑎𝑛𝑡,
𝑑𝐹 𝑑𝐹
implying 𝐹 ∙ = 0 or 𝐹 ⊥ .
𝑑𝑡 𝑑𝑡
𝑑𝐹
(ii) If 𝐹 (𝑡) has a constant (fixed) direction, then 𝐹 × 𝑑𝑡 = 0.
Let 𝐹 𝑡 = 𝑓(𝑡)𝐺 (𝑡), where 𝐺 (𝑡) is a unit vector in the direction of 𝐹 (𝑡).
dF 𝑑 𝑓 𝑡 𝐺 𝑡 𝑑𝐺 𝑑𝑓 𝑑𝑓 𝑑𝐺
∴ dt
= 𝑑𝑡
=𝑓 𝑡
𝑑𝑡
+ 𝑑𝑡 𝐺 𝑡 = 𝑑𝑡 𝐺 𝑡 𝑠𝑖𝑛𝑐𝑒, 𝐺 𝑖𝑠 𝑎 𝑐𝑜𝑛𝑠𝑡𝑎𝑛𝑡, 𝑠𝑜
𝑑𝑡
=0
𝑑𝐹 𝑑𝑓 𝑑𝑓
and 𝐹 × 𝑑𝑡
= 𝑓(𝑡)𝐺 (𝑡) ×
𝑑𝑡
𝐺 𝑡 =𝑓 𝑡
𝑑𝑡
𝐺 𝑡 ×𝐺 𝑡 =0 𝑠𝑖𝑛𝑐𝑒, 𝐺 × 𝐺 = 0

Theorem 1: Derivative of a constant vector is a zero vector. A vector is said to be constant if


both its magnitude and direction are constant (fixed).
Proof: Let 𝑟 = 𝑐 be a constant vector, then 𝑟 + 𝛿𝑟 = 𝑐.
𝛿𝑟
On subtraction, 𝛿𝑟 = 0.Which further implies that = 0.
𝛿𝑡

3
𝐿𝑡 𝛿 𝑟 𝑑𝑟
Implying, = 0 i.e. = 0.
𝛿𝑡 → 0 𝛿𝑡 𝑑𝑡

Theorem 2: The necessary and sufficient condition for the vector function 𝑭 of a single
𝒅𝑭
variable t to have constant magnitude is 𝑭 ∙ = 𝟎.
𝒅𝒕
Proof:
2
Necessary condition: Suppose 𝐹 has constant magnitude, so 𝐹 𝑡 ∙ 𝐹 𝑡 = 𝐹 (𝑡) = 𝑐𝑜𝑛𝑠𝑡𝑎𝑛𝑡.
𝑑 𝑑𝐹 𝑑𝐹
=> 𝐹∙ 𝐹 =0 i.e. 𝐹∙ + ∙𝐹=0
𝑑𝑡 𝑑𝑡 𝑑𝑡
𝑑𝐹 𝑑𝐹
=> 2𝐹 ∙ =0 i.e. 𝐹∙ = 0.
𝑑𝑡 𝑑𝑡
𝑑𝐹 𝑑𝐹
Sufficient condition: Suppose 𝐹 ∙ =0 => 2𝐹 ∙ =0
𝑑𝑡 𝑑𝑡
𝑑𝐹 𝑑𝐹 𝑑
=> 𝐹∙
𝑑𝑡
+ ∙𝐹=0 => 𝐹∙ 𝐹 =0
𝑑𝑡 𝑑𝑡
2
=> 𝐹 ∙ 𝐹 = 𝑐𝑜𝑛𝑠𝑡𝑎𝑛𝑡 => 𝐹 = 𝑐𝑜𝑛𝑠𝑡𝑎𝑛𝑡
Therefore 𝐹 has a constant magnitude.

Theorem 3: The necessary and sufficient condition for the vector function 𝑭 of a single
𝒅𝑭
variable t to have a constant direction is 𝑭 × = 𝟎.
𝒅𝒕
𝐹
Proof: Suppose that 𝑓 is a unit vector in the direction of 𝐹 and 𝐹 = 𝐹 , then 𝑓= i.e.
𝐹
𝐹 = 𝐹𝑓 … (1)
𝑑𝐹 𝑑𝑓 𝑑𝐹
And =𝐹 + 𝑓 … (2)
𝑑𝑡 𝑑𝑡 𝑑𝑡
𝑑𝐹 𝑑𝑓 𝑑𝐹
Thus 𝐹 × = 𝐹𝑓 × (𝐹 + 𝑓) (using (1) and (2))
𝑑𝑡 𝑑𝑡 𝑑𝑡
𝑑𝑓 𝑑𝐹
= 𝐹2 𝑓 × +𝐹 (𝑓 × 𝑓 )
𝑑𝑡 𝑑𝑡
𝑑𝑓
= 𝐹2 𝑓 × (since 𝑓 × 𝑓 = 0 ) … (3)
𝑑𝑡
Necessary condition: Suppose 𝐹 has a constant direction, then 𝑓 has a constant direction and
𝑑𝑓 𝑑𝐹
constant magnitude. So = 0. Thus from (3), 𝐹 × = 0.
𝑑𝑡 𝑑𝑡
𝑑𝐹
Sufficient condition : Suppose that 𝐹 × = 0.
𝑑𝑡
𝑑𝑓 𝑑𝑓
Then by (3), 𝐹 2 𝑓 × = 0 i.e. 𝑓 × =0 … (4)
𝑑𝑡 𝑑𝑡
𝑑𝑓
Since 𝑓 has a constant magnitude, so, by theorem 2, 𝑓 ∙ =0 … (5)
𝑑𝑡
𝑑𝑓
Form (4) and (5), = 0.
𝑑𝑡
Which implies 𝑓 is a constant vector i.e. 𝑓 has a constant direction. Hence 𝐹 has a constant
direction.

Example 1: Show that if 𝒓 = 𝒂 𝐬𝐢𝐧 𝝎𝒕 + 𝒃 𝐜𝐨𝐬 𝝎𝒕 where 𝒂, 𝒃 and 𝝎 are constants, then

4
𝒅𝟐 𝒓 𝒅𝒓
= −𝝎𝟐 𝒓 and 𝒓 × = −𝝎(𝒂 × 𝒃).
𝒅𝒕𝟐 𝒅𝒕

Solution: Given 𝑟 = 𝑎 sin 𝜔𝑡 + 𝑏 cos 𝜔𝑡


𝑑𝑟
Differentiating w. r. to t, = 𝑎𝜔 cos 𝜔𝑡 − 𝑏𝜔 sin 𝜔𝑡
𝑑𝑡
𝑑 2𝑟
Again differentiating w. r. to t, = −𝑎 𝜔2 sin 𝜔𝑡 − 𝑏 𝜔2 cos 𝜔𝑡
𝑑𝑡 2
= − 𝜔2 𝑎 sin 𝜔𝑡 + 𝑏 cos 𝜔𝑡 = −𝜔2 𝑟
𝑑𝑟
Also 𝑟 × = 𝑎 sin 𝜔𝑡 + 𝑏 cos 𝜔𝑡 × (𝑎𝜔 cos 𝜔𝑡 − 𝑏𝜔 sin 𝜔𝑡)
𝑑𝑡
= 𝑎 × 𝑎 𝜔 sin 𝜔𝑡 cos 𝜔𝑡 + 𝑏 × 𝑎 𝜔 cos 2 𝜔𝑡 − 𝑎 × 𝑏 𝜔 sin2 𝜔𝑡 − 𝑏 × 𝑏 𝜔 sin 𝜔𝑡 cos 𝜔𝑡
= − 𝑎 × 𝑏 𝜔(cos 2 𝜔𝑡 + sin2 𝜔𝑡) (since 𝑎 × 𝑎 = 𝑏 × 𝑏 = 0)
= − 𝑎 × 𝑏 𝜔 = −𝜔(𝑎 × 𝑏).
𝝏𝟐
Example 2: If 𝒂 = 𝒙𝟐 𝒚𝒛 𝒊 − 𝟐𝒙𝒛𝟑 𝒋 + 𝒙𝒛𝟐 𝒌 and 𝒃 = 𝟐𝒛 𝒊 + 𝒚 𝒋 − 𝒙𝟐 𝒌 , find 𝒂 × 𝒃 at
𝝏𝒙𝝏𝒚
(1, 0, -2).
Solution: Here 𝑎 × 𝑏 = 𝑥 2 𝑦𝑧 𝑖 − 2𝑥𝑧 3 𝑗 + 𝑥𝑧 2 𝑘 × 2𝑧 𝑖 + 𝑦 𝑗 − 𝑥 2 𝑘
= 𝑥 2 𝑦 2 𝑧 𝑘 + 𝑥 4 𝑦𝑧 𝑗 + 4𝑥𝑧 4 𝑘 + 2𝑥 3 𝑧 3 𝑖 + 2𝑥𝑧 3 𝑗 − 𝑥𝑦𝑧 2 𝑖
(∵ 𝑖 × 𝑖 = 𝑗 × 𝑗 = 𝑘 × 𝑘 = 0 𝑎𝑛𝑑 𝑖 × 𝑗 = 𝑘 , 𝑗 × 𝑖 = −𝑘 𝑒𝑡𝑐.)
= 2𝑥 3 𝑧 3 − 𝑥𝑦𝑧 2 𝑖 + 𝑥 4 𝑦𝑧 + 2𝑥𝑧 3 𝑗 + 𝑥 2 𝑦 2 𝑧 + 4𝑥𝑧 4 𝑘
𝜕2 𝜕2
Now 𝑎×𝑏 = 2𝑥 3 𝑧 3 − 𝑥𝑦𝑧 2 𝑖 + 𝑥 4 𝑦𝑧 + 2𝑥𝑧 3 𝑗 + 𝑥 2 𝑦 2 𝑧 + 4𝑥𝑧 4 𝑘
𝜕𝑥𝜕𝑦 𝜕𝑥𝜕𝑦
𝜕 𝜕
= 2𝑥 3 𝑧 3 − 𝑥𝑦𝑧 2 𝑖 + 𝑥 4 𝑦𝑧 + 2𝑥𝑧 3 𝑗 + 𝑥 2 𝑦 2 𝑧 + 4𝑥𝑧 4 𝑘
𝜕𝑥 𝜕𝑦
𝜕
= −𝑥𝑧 2 𝑖 + 𝑥 4 𝑧 𝑗 + 2𝑥 2 𝑦𝑧 𝑘 = −𝑧 2 𝑖 + 4𝑥 3 𝑧 𝑗 + 4𝑥𝑦𝑧 𝑘
𝜕𝑥
𝜕2
At the point (1, 0, -2) 𝑎 × 𝑏 = −4𝑖 − 8𝑗
𝜕𝑥𝜕𝑦

𝒅
Example 3: If 𝑷 = 𝟓𝒕𝟐 𝒊 + 𝒕𝟑 𝒋 − 𝒕𝒌 and 𝑸 = 𝟐 𝐬𝐢𝐧 𝒕 𝒊 − 𝐜𝐨𝐬 𝒕 𝒋 + 𝟓𝒕𝒌 , then find (a) 𝑷∙𝑸
𝒅𝒕
𝒅
(b) 𝑷×𝑸 .
𝒅𝒕

Solution: Consider 𝑃 = 5𝑡 2 𝑖 + 𝑡 3 𝑗 − 𝑡𝑘 and 𝑄 = 2 sin 𝑡 𝑖 − cos 𝑡 𝑗 + 5𝑡𝑘


𝒅𝑃 𝒅𝑄
So = 10𝑡 𝑖 + 3𝑡 2 𝑗 − 𝑘 and = 2 cos 𝑡 𝑖 + sin 𝑡 𝑗 + 5𝑘
𝒅𝒕 𝒅𝒕
𝑑 𝒅𝑃 𝒅𝑄
a) 𝑃∙𝑄 = ∙𝑄+𝑃∙
𝑑𝑡 𝒅𝒕 𝒅𝒕
= 10𝑡 𝑖 + 3𝑡 2 𝑗 − 𝑘 ∙ 2 sin 𝑡 𝑖 − cos 𝑡 𝑗 + 5𝑡𝑘
+ 5𝑡 2 𝑖 + 𝑡 3 𝑗 − 𝑡𝑘 ∙ 2 cos 𝑡 𝑖 + sin 𝑡 𝑗 + 5𝑘
= 20 𝑡 sin 𝑡 − 3𝑡 2 cos 𝑡 − 5𝑡 + 10 𝑡 2 cos 𝑡 + 𝑡 3 sin 𝑡 − 5𝑡
= 𝑡 3 sin 𝑡 + 7𝑡 2 cos 𝑡 + 20 𝑡 sin 𝑡 − 10 𝑡

5
𝑑 𝒅𝑃 𝒅𝑄
b) 𝑃×𝑄 = ×𝑄+𝑃×
𝑑𝑡 𝒅𝒕 𝒅𝒕
= 10𝑡 𝑖 + 3𝑡 2 𝑗 − 𝑘 × 2 sin 𝑡 𝑖 − cos 𝑡 𝑗 + 5𝑡𝑘
+ 5𝑡 2 𝑖 + 𝑡 3 𝑗 − 𝑡𝑘 × 2 cos 𝑡 𝑖 + sin 𝑡 𝑗 + 5𝑘

𝑖 𝑗 𝑘 𝑖 𝑗 𝑘
= 10 𝑡 3𝑡 2 −1 + 5𝑡 2 𝑡3 −𝑡
2 sin 𝑡 − cos 𝑡 5𝑡 2 cos 𝑡 sin 𝑡 5
= 𝑖 15𝑡 3 − cos 𝑡 + 𝑗 −2 sin 𝑡 − 50 𝑡 2 + 𝑘 −10𝑡 cos 𝑡 − 6𝑡 2 sin 𝑡
+ 𝑖 5𝑡 3 + 𝑡 sin 𝑡 + 𝑗 −2𝑡 cos 𝑡 − 25 𝑡 2 + 𝑘 5𝑡 2 sin 𝑡 − 2𝑡 3 cos 𝑡
= 𝑖 20𝑡 3 + 𝑡 sin 𝑡 − cos 𝑡 − 𝑗 2𝑡 cos 𝑡 + 2 sin 𝑡 + 75 𝑡 2
−𝑘 2𝑡 3 cos 𝑡 + 10𝑡 cos 𝑡 + 𝑡 2 sin 𝑡

𝒅𝑼 𝒅𝑽 𝒅
Example 4: If = 𝑾 × 𝑼 and = 𝑾 × 𝑽 , then prove that 𝑼 × 𝑽 = 𝑾 × (𝑼 × 𝑽).
𝒅𝒕 𝒅𝒕 𝒅𝒕
𝑑𝑈 𝑑𝑉
Solution: Given = 𝑊 × 𝑈 and =𝑊×𝑉 … (1)
𝑑𝑡 𝑑𝑡
𝑑 𝑑𝑈 𝑑𝑉
Consider 𝑈×𝑉 = ×𝑉+𝑈× = 𝑊×𝑈 ×𝑉+𝑈× 𝑊×𝑉
𝑑𝑡 𝑑𝑡 𝑑𝑡
= 𝑊∙𝑉 𝑈− 𝑈∙𝑉 𝑊+ 𝑈∙𝑉 𝑊− 𝑈∙𝑊 𝑉 = 𝑊∙𝑉 𝑈− 𝑈∙𝑊 𝑉
= 𝑊 × (𝑈 × 𝑉 )
𝑢𝑠𝑖𝑛𝑔 𝑎 × 𝑏 × 𝑐 = 𝑎 ∙ 𝑐 𝑏 − 𝑏 ∙ 𝑐 𝑎 𝑎𝑛𝑑 𝑎 × 𝑏 × 𝑐 = 𝑎 ∙ 𝑐 𝑏 − 𝑎 ∙ 𝑏 𝑐

7.3 CURVES IN SPACE

1. Tangent Vector:
Let 𝑟 𝑡 = 𝑥 𝑡 𝑖 + 𝑦 𝑡 𝑗 + 𝑧(𝑡)𝑘 be the position vector
of a point P. Then for different values of the scalar
parameter t, point P traces the curve in space (Fig.
17.1). For neighboring point Q with position vector
𝛿𝑟
𝑟(𝑡 + 𝛿𝑡) , 𝛿𝑟 = 𝑟 𝑡 + 𝛿𝑡 − 𝑟 𝑡 implying =
𝛿𝑡
𝑟 𝑡+𝛿𝑡 −𝑟 (𝑡)
is directed along the chord PQ.
𝛿𝑡
𝛿𝑟
As 𝛿𝑡 → 0, becomes the tangent to the space curve
𝛿𝑡
at P provided there exists a non zero limit.
𝑑𝑟
Thus a vector = 𝑟 ′ is a tangent to the space curve 𝑟 = 𝐹 (𝑡).
𝑑𝑡
Let P0 be a fixed point on the space curve corresponding to t=t0, and the arc length 𝑃0 𝑃 = 𝑠 , then

𝛿𝑠 𝛿𝑠 𝛿𝑟 𝑎𝑟𝑐 𝑃𝑄 𝛿𝑟
= = . … (1)
𝛿𝑡 𝛿𝑟 𝛿𝑡 𝑐𝑕𝑜𝑟𝑑 𝑃𝑄 𝛿𝑡

𝑎𝑟𝑐 𝑃𝑄 𝑑𝑠 𝑑𝑟
As 𝑄 → 𝑃 along the curve QP, i.e. 𝛿𝑡 → 0, then the → 1 and = = 𝑟 ′ (𝑡) … (2)
𝑐𝑕𝑜𝑟𝑑 𝑃𝑄 𝑑𝑡 𝑑𝑡

6
𝑑𝑟 𝑡 𝑑𝑟 𝑡
If is continuous, then 𝑠 = 𝑡 0 𝑑𝑡
𝑑𝑡 = 𝑡0
𝑥 ′ 2 + 𝑦 ′ 2 + 𝑧 ′ 2 𝑑𝑡 … (3)
𝑑𝑡

Further, if we take s as the parameter in place of t, then the magnitude of the tangent vector i.e.
𝑑𝑟 𝑑𝑟
= 1. Thus denoting the unit tangent vector by 𝑇, we have 𝑇 = …(4)
𝑑𝑠 𝑑𝑠

Example 5: Find the unit tangent vector at any point on the curve 𝒙 = 𝒕𝟐 + 𝟐, 𝒚 = 𝟒𝒕 − 𝟓,
𝒛 = 𝟐𝒕𝟐 − 𝟔𝒕 where t is variable. Also determine the unit tangent vector at t = 2.

Solution: Let 𝑟 be the position vector of any point 𝑥, 𝑦, 𝑧 on the given curve,
then 𝑟 = 𝑥𝑖 + 𝑦𝑗 + 𝑧𝑘
∴ 𝑟 = 𝑡 2 + 2 𝑖 + 4𝑡 − 5 𝑗 + 2𝑡 2 − 6𝑡 𝑘
𝑑𝑟
The vector tangent to the curve at any point 𝑥, 𝑦, 𝑧 is = 2𝑡 𝑖 + 4 𝑗 + 4𝑡 − 6 𝑘
𝑑𝑡
𝑑𝑟 2 2 2
Now = 2𝑡 + 4 + 4𝑡 − 6 = 2 5𝑡 2 − 12𝑡 + 13
𝑑𝑡
𝑑𝑟 𝑑𝑟 2𝑡 𝑖 + 4 𝑗 + 4𝑡−6 𝑘
Therefore unit tangent vector at 𝑥, 𝑦, 𝑧 = =
𝑑𝑡 𝑑𝑡 2 5𝑡 2 −12𝑡+13

𝑑𝑟 𝑑𝑟 2𝑡 𝑖 + 4 𝑗 + 4𝑡−6 𝑘 2𝑖 +2𝑗 +𝑘
And the unit tangent vector at t = 2 is 𝑑𝑡 𝑑𝑡 = = .
𝑡=2 2 5𝑡 2 −12𝑡+13 𝑡 =2 3

𝑑𝑇 𝑑𝑇
2. Principal Normal: Since 𝑇 is a unit vector, so ∙ 𝑇 = 0 i.e. either is perpendicular to 𝑇 or
𝑑𝑠 𝑑𝑠
𝑑𝑇
= 0, in which case 𝑇 is a constant vector w.r.t. the arc length s and so has a fixed direction i.e.
𝑑𝑠
𝑑𝑇
the curve is a straight line. Now, if we denote the unit normal vector to the curve at P by 𝑁, then 𝑑𝑠
is in the direction of 𝑁 which is known as the principal normal to the curve at P. The plane of 𝑇
and 𝑁 is called osculating plane of the curve at P.

3. Binormal: A unit vector 𝐵 = 𝑇 × 𝑁 is called the binormal at P. As 𝑇 and 𝑁 both are unit vectors,
so 𝐵 is also a unit vector normal to both 𝑇 and 𝑁
i.e. to the osculating plane of 𝑇 and 𝑁.
Thus at each point P on the curve C, there are
three mutually perpendicular unit vectors 𝑇 , 𝑁
and 𝐵, which form a moving trihedral such that
𝐵 = 𝑇 × 𝑁, 𝑁 = 𝐵 × 𝑇 , 𝑇 = 𝑁 × 𝐵 . … (1)
This moving trihedral determines three
fundamental planes at each point of the curve C.

(i) The osculating plane of 𝑇 and 𝑁.


(ii) The normal plane of 𝑁 and 𝐵.
(iii) The rectifying plane of 𝐵 and 𝑇.

𝑑𝑇
4. Curvture: The arc rate of turning of the tangent viz. is called the curvature of the curve
𝑑𝑠
𝑑𝑇 𝑑𝑇
and is denoted by . As is in the direction of the principal normal 𝑁, therefore = 𝑁 … (2)
𝑑𝑠 𝑑𝑠

7
𝑑𝐵 𝑑𝐵
5. Torsion: As the binormal 𝐵 is a unit vector, so ∙ 𝐵 = 0. Also 𝐵 ∙ 𝑇 = 0, therefore ∙𝑇+𝐵∙
𝑑𝑠 𝑑𝑠
𝑑𝑇 𝑑𝐵 𝑑𝐵 𝑑𝐵
=0 or ∙ 𝑇 + 𝐵 ∙ 𝑁 = 0 or ∙ 𝑇 = 0. Hence, is perpendicular to both 𝐵 and 𝑇 and
𝑑𝑠 𝑑𝑠 𝑑𝑠 𝑑𝑠
𝑑𝐵
is, therefore, parallel to 𝑁. The arc rate of turning of the binormal viz. is called torsion of the
𝑑𝑠
curve and is denoted by 𝜏. So, we can write it as
𝑑𝐵 𝑑𝐵
= −𝜏𝑁 (the negative sign indicates that for 𝜏 > 0, has a direction of −𝑁) …(3)
𝑑𝑠 𝑑𝑠
Further, we know that 𝑁 = 𝐵 × 𝑇, which on differentiation gives ,

𝑑𝑁 𝑑𝐵 𝑑𝑇
= ×𝑇+𝐵× = −𝜏𝑁 × 𝑇 + 𝐵 × 𝑁
𝑑𝑠 𝑑𝑠 𝑑𝑠

𝑑𝑁
= 𝜏𝐵 − 𝑇 (using (1)) … (4)
𝑑𝑠

The relations in (1), (2) and (3) constitutes the well known Frenet Formulas for the curve C.

Observations:
1
(i) 𝜌 = is called the radius of curvature.

1
(ii) 𝜍 = 𝜏 is called the radius of torsion.
(iii) 𝜏 = 0 for a plane curve.

Example 6: Find 𝑵(𝒕) and 𝑵(𝟏) for the curve represented by 𝒓 𝒕 = 𝟑𝒕 𝒊 + 𝟐𝒕𝟐 𝒋.

Solution: For given 𝑟 𝑡 , we have 𝑟 ′ 𝑡 = 3𝑖 + 4𝑡𝑗 and 𝑟 ′ 𝑡 = 9 + 16𝑡 2


𝑟′ 𝑡 3𝑖 +4𝑡𝑗
Which implies that the unit tangent vector 𝑇 𝑡 = = … (1)
𝑟′ 𝑡 9+16𝑡 2
1 16𝑡 12(−4𝑡𝑖 +3𝑗 )
Differentiating 𝑇 𝑡 w. r. to t, 𝑇 ′ 𝑡 = 4𝑗 − 3 (3𝑖 + 4𝑡𝑗) = …(2)
9+16𝑡 2 9+16𝑡 2 3/2
9+16𝑡 2 2
9+16𝑡 2 12
And 𝑇′ 𝑡 = 12 = … (3)
9+16𝑡 2 3 9+16𝑡 2
𝑇 ′ (𝑡) −4𝑡𝑖 +3𝑗
Therefore, the principal unit normal vector is 𝑁 𝑡 = = … (4)
𝑇 ′ (𝑡) 9+16𝑡 2
1
But at 𝑡 = 1, the principal unit normal vector is 𝑁 1 = (−4𝑖 + 3𝑗).
5

Example 7: Find the angle between the tangents to the curve 𝒓 = 𝒕𝟐 𝒊 + 𝟐𝒕 𝒋 − 𝒕𝟑 𝒌 at the point
t = ± 1.

Solution: Differentiating the given curve w. r. to t, we get


𝑑𝑟
= 2𝑡 𝑖 + 2 𝑗 − 3𝑡 2 𝑘 which is the tangent vector to the curve at any point t.
𝑑𝑡

Let 𝑇1 & 𝑇2 are the tangent vectors to the curve at t = 1 and t = -1 respectively, then
𝑇1 = 2 𝑖 + 2 𝑗 − 3 𝑘 and 𝑇2 = −2 𝑖 + 2 𝑗 − 3 𝑘
Let 𝜃 be the angle between the tangents 𝑇1 & 𝑇2 , then
𝑇1 ∙ 𝑇2 2 𝑖 +2 𝑗 −3 𝑘 ∙ −2 𝑖 +2 𝑗 −3 𝑘 −4 + 4 + 9 9
𝐶𝑜𝑠 𝜃 = = = =
𝑇1 𝑇2 2 𝑖 +2 𝑗 −3 𝑘 −2 𝑖 +2 𝑗 −3 𝑘 17 17 17
8
9
∴ 𝜃 = 𝑐𝑜𝑠 −1 17

Example 8: Find the curvature and torsion of the curve 𝒙 = 𝒂 𝒄𝒐𝒔 𝒕, 𝒚 = 𝒂 𝒔𝒊𝒏 𝒕, 𝒛 = 𝒃𝒕.
(This curve is drawn on a circular cylinder cutting its generators at a constant angle and is known as a circular helix)

Solution: Equation of the given curve in vector form is


𝑟 = 𝑎 cos 𝑡 𝑖 + 𝑎 sin 𝑡 𝑗 + 𝑏𝑡 𝑘
Differentiating w. r. to t,
𝑑𝑟
= −𝑎 sin 𝑡 𝑖 + 𝑎 cos 𝑡 𝑗 + 𝑏 𝑘
𝑑𝑡
Now, the arc length of the curve from P0 (t = 0) to any
point P (t) is given by
𝑡 𝑑𝑟
𝑠= 0 𝑑𝑡
𝑑𝑡 = 𝑎2 + 𝑏 2 𝑡
𝑑𝑠
∴ = 𝑎2 + 𝑏 2
𝑑𝑡
Now, the unit tangent vector,
𝑑𝑟 𝑑𝑟 𝑑𝑡 −𝑎 sin 𝑡 𝑖 + 𝑎 cos 𝑡 𝑗 + 𝑏 𝑘
𝑇= = =
𝑑𝑠 𝑑𝑠 𝑑𝑡 𝑎 2 +𝑏 2
𝑑𝑇 𝑑𝑇 𝑑𝑡 −𝑎 cos 𝑡 𝑖 − 𝑎 sin 𝑡 𝑗
So = =
𝑑𝑠 𝑑𝑠 𝑑𝑡 𝑎 2 +𝑏 2
𝑑𝑇 𝑎
∴ = = is the curvature of the given curve.
𝑑𝑠 𝑎 2 +𝑏 2
(b sin 𝑡 𝑖 −𝑏 cos 𝑡𝑗 +𝑎𝑘 )
Also, the unit normal vector is 𝑁 = −(cos 𝑡 𝑖 + sin 𝑡 𝑗) and 𝐵 = 𝑇 × 𝑁 =
𝑎 2 +𝑏 2
𝑑𝐵 𝑑𝐵 𝑑𝑡 𝑏(cos 𝑡 𝑖 +sin 𝑡 𝑗 )
So = = = −𝜏𝑁 = 𝜏(cos 𝑡 𝑖 + sin 𝑡 𝑗)
𝑑𝑠 𝑑𝑠 𝑑𝑡 𝑎 2 +𝑏 2
𝑏
Hence 𝜏 = .
𝑎 2 +𝑏 2

Example 9: A circular helix is given by the equation 𝒓 = 𝟐 𝐜𝐨𝐬 𝒕 𝒊 + 𝟐 𝐬𝐢𝐧 𝒕 𝒋 + 𝒌. Find the
curvature and torsion of the curve at any point and show that they are constant.
Solution: Equation of the given curve in vector form is
𝑟 = 2 cos 𝑡 𝑖 + 2 sin 𝑡 𝑗 + 𝑘
𝑑𝑟
Differentiating w. r. to t, = −2 sin 𝑡 𝑖 + 2 cos 𝑡 𝑗 + 0 𝑘
𝑑𝑡
Now, the arc length of the curve from P0 (t = 0) to any point P (t) is given by
𝑡 𝑑𝑟 𝑑𝑠
𝑠= 0 𝑑𝑡
𝑑𝑡 = 2 𝑡 implying =2
𝑑𝑡
𝑑𝑟 𝑑𝑟 𝑑𝑡 −2 sin 𝑡 𝑖 + 2 cos 𝑡 𝑗 + 0 𝑘
Now, the unit tangent vector, 𝑇 = = =
𝑑𝑠 𝑑𝑠 𝑑𝑡 2
𝑑𝑇 𝑑𝑇 𝑑𝑡 − cos 𝑡 𝑖 − sin 𝑡 𝑗
So = =
𝑑𝑠 𝑑𝑠 𝑑𝑡 2
𝑑𝑇 1
∴ = = is the curvature of the given curve and is a constant.
𝑑𝑠 2
Also, the unit normal vector is 𝑁 = −(cos 𝑡 𝑖 + sin 𝑡 𝑗) and
𝐵 = 𝑇 × 𝑁 = − sin 𝑡 𝑖 + cos 𝑡 𝑗 × − cos 𝑡 𝑖 − sin 𝑡 𝑗 = 𝑘
𝑑𝐵 𝑑𝐵 𝑑𝑡
So = = 0 = −𝜏𝑁 = 𝜏(cos 𝑡 𝑖 + sin 𝑡 𝑗)
𝑑𝑠 𝑑𝑠 𝑑𝑡

9
Hence 𝜏 = 0 is the torsion of the given curve and is constant.

Example 10: Show that for the curve 𝒓 = 𝒂 𝟑𝒕 − 𝒕𝟑 𝒊 + 𝟑𝒂𝒕𝟐 𝒋 + 𝒂(𝟑𝒕 + 𝒕𝟑 )𝒌, the curvature
equals torsion.
Solution: Given curve is 𝑟 = 𝑎 3𝑡 − 𝑡 3 𝑖 + 3𝑎𝑡 2 𝑗 + 𝑎(3𝑡 + 𝑡 3 )𝑘
𝑑𝑟
Differentiating w. r. to t, 𝑑𝑡
= 𝑎 3 − 3𝑡 2 𝑖 + 6𝑎𝑡 𝑗 + 𝑎 3 + 3𝑡 2 𝑘

Now, the arc length of the curve P0 (t = 0) to any point P (t) is given by
𝑡 𝑑𝑟 𝑡 2 2
𝑠= 0 𝑑𝑡
𝑑𝑡 = 0
𝑎 3 − 3𝑡2 + 6𝑎𝑡 2
+ 𝑎 3 + 3𝑡2 𝑑𝑡

𝑡 𝑡3
= 3𝑎 2 0
𝑡 2 + 1 𝑑𝑡 = 3𝑎 2 +𝑡
3

𝑑𝑠
∴ = 3𝑎 2 𝑡 2 + 1
𝑑𝑡
Now, the unit tangent vector,
𝑑𝑟 𝑑𝑟 𝑑𝑡 𝑎 3−3𝑡2 𝑖+6𝑎𝑡 𝑗+𝑎 3+3𝑡2 𝑘 1−𝑡2 𝑖+2𝑡 𝑗+ 1+𝑡2 𝑘
𝑇= = = =
𝑑𝑠 𝑑𝑠 𝑑𝑡 3𝑎 2 𝑡2 +1 2 𝑡2 +1

𝑑𝑇 𝑑𝑇 𝑑𝑡 −2𝑡 𝑖 + 1−𝑡 2 𝑗
So = =
𝑑𝑠 𝑑𝑠 𝑑𝑡 3𝑎 1+𝑡 2 3

𝑑𝑇 1
∴ = = is the curvature of the given curve.
𝑑𝑠 3𝑎 1+𝑡 2 2

−2𝑡 𝑖+ 1−𝑡2 𝑗
Also, the unit normal vector is 𝑁 = and
1+𝑡2
1−𝑡 2 𝑖 +2𝑡 𝑗 + 1+𝑡 2 𝑘
𝐵 =𝑇×𝑁 =
2 1+𝑡 2

𝑑𝐵 𝑑𝐵 𝑑𝑡 −2𝑡 𝑖+ 1−𝑡2 𝑗 1 −2𝑡 𝑖+ 1−𝑡2 𝑗


So = =− 3 =− 2. = −𝜏𝑁
𝑑𝑠 𝑑𝑠 𝑑𝑡 3𝑎 1+𝑡2 3𝑎 1+𝑡2 1+𝑡2

1
∴ 𝜏= 2 is the torsion of the given curve.
3𝑎 1+𝑡2

Hence curvature equals torsion for the given curve.

ASSIGNMENT 1
𝜕2
1. If 𝑎 = 𝑥 2 𝑦𝑧 𝑖 − 2𝑥𝑧 3 𝑗 + 𝑥𝑧 2 𝑘 and 𝑏 = 2𝑧 𝑖 + 𝑦 𝑗 − 𝑥 2 𝑘, find 𝑎 × 𝑏 at (1, 0, -2).
𝜕𝑥𝜕𝑦

𝑑 2𝑟
2. Given 𝑟 = 𝑡 𝑚 𝐴 + 𝑡 𝑛 𝐵, where 𝐴 and 𝐵 are constant vectors, show that, if 𝑟 and are parallel
𝑑𝑡 2
vectors, then 𝑚 + 𝑛 = 1, unless 𝑚 = 𝑛.
𝜋
3. Find the equation of tangent line to the curve 𝑥 = 𝑎 cos 𝜃 , 𝑦 = 𝑎 sin 𝜃 , 𝑧 = 𝑎𝜃 tan 𝛼 at = .
4

10
4. Find the unit tangent vector at any point on the curve 𝑥 = 𝑡 2 + 2, 𝑦 = 4𝑡 − 5, 𝑧 = 2𝑡 2 − 6𝑡, where t
is any variable. Also determine the unit tangent vector at the point 𝑡 = 2.
𝑑𝑟 𝑑 2𝑟 𝑑𝑟 𝑑 2 𝑟 𝑑 3 𝑟
5. If 𝑟 = 𝑎 cos 𝑡 𝑖 + 𝑎 sin 𝑡 𝑗 + 𝑎𝑡 tan 𝛼 𝑘 , find the value of (a) × (b) .
𝑑𝑡 𝑑𝑡 2 𝑑𝑡 𝑑𝑡 2 𝑑 𝑡 3
Also find the unit tangent vector at any point t on the curve.
6. Find the equation of the osculating plane and binormal to the curve
𝜃 𝜃
(a) 𝑥 = 𝑒 𝜃 cos 𝜃 , 𝑦 = 𝑒 𝜃 sin 𝜃 , 𝑧 = 𝑒 𝜃 at 𝜃 = 0 (b) 𝑥 = 2 cosh 2 , 𝑦 = 2 sinh 2 , 𝑧 = 2𝜃 at 𝜃 = 0
7. Find the curvature of the (a) ellipse 𝑟 = 𝑎 cos 𝑡 𝑖 + 𝑏 sin 𝑡 𝑗 (b) parabola 𝑟 = 2𝑡 𝑖 + 𝑡 2 𝑗 at point
𝑡 = 1.

17.4 VELOCITY AND ACCELERATION

1. Velocity: Let the position of particle P at a time t on the curve C is 𝑟 𝑡 and it comes to point Q
𝛿𝑟
at time 𝑡 + 𝛿𝑡 having position 𝑟 𝑡 + 𝛿𝑡 , then 𝛿𝑟 = 𝑟 𝑡 + 𝛿𝑡 − 𝑟 𝑡 i.e. is directed along PQ.
𝛿𝑡
As 𝑄 → 𝑃 along C, the line PQ becomes tangent at P to the curve C.

𝑑𝑟 𝛿𝑟
So 𝑣 𝑡 = = Lt 𝛿𝑡 →0 is the tangent vector to C at point P which is the velocity vector 𝑣(𝑡) of
𝑑𝑡 𝛿𝑡
𝑑𝑠
the motion and its magnitude gives the speed 𝑣 = , where s is the arc length of P from a fixed
𝑑𝑡
point P0 (s=0) on C.

2. Acceleration: Acceleration vector 𝑎 𝑡 of a particle is the derivative of the velocity vector 𝑣(𝑡),
𝑑𝑣 𝑑 2𝑟
and it is given by 𝑎 𝑡 = = . It is an interesting fact that the magnitude of acceleration is not
𝑑𝑡 𝑑𝑡 2
always the rate of change of 𝑣 = 𝑣 , as 𝑎 𝑡 is not always tangential to the curve C. There are two
components of acceleration, which are given as: (i) Tangential Acceleration (ii) Normal
Acceleration
𝑑𝑠
Observation: The acceleration is the time rate of change of 𝑣 𝑡 = , if and only if the normal acceleration
𝑑𝑡
𝑑2𝑠 𝑑𝑟 𝑑2𝑠
is zero, for then 𝑎 = 𝑑𝑡 2 𝑑𝑠
= 𝑑𝑡 2
.

3. Relative Velocity and Acceleration: Let two particles P and


Q moving along the curves C1 and C2 have position vectors 𝑟1 (𝑡)
and 𝑟2 (𝑡) at time t, so that 𝑟 𝑡 = 𝑃𝑄 = 𝑟2 𝑡 − 𝑟1 (𝑡)
𝑑𝑟 𝑑𝑟2 𝑑𝑟1
Differentiating w. r. to t, = − … (1)
𝑑𝑡 𝑑𝑡 𝑑𝑡
This defines the relative velocity of Q w. r. t. P and states that the
velocity of Q relative to P = Velocity vector of Q – Velocity
vector of P.
Again differentiating (1) w. r. to t., we have
𝑑 2𝑟 𝑑 2 𝑟2 𝑑 2 𝑟1
= −
𝑑𝑡2 𝑑𝑡2 𝑑𝑡 2
This defines the relative acceleration of Q w. r. t. and states that Acceleration of Q relative to P =
Acceleration of Q – Acceleration of P.

11
Example 11: Find the tangential and normal acceleration of a particle moving in a plane
curve in Cartesian coordinates.

Solution: Let 𝑟 be the position vector the point P, a function of a scalar t. In particular, if the scalar
variable t is taken as an arc length s along the curve C measured from some fixed point, that is,
𝑥 = 𝑥 𝑠 , 𝑦 = 𝑦 𝑠 , 𝑧 = 𝑧(𝑠) then 𝑟 = 𝑥 𝑠 𝑖 + 𝑦 𝑠 𝑗 + 𝑧(𝑠)𝑘
𝑑𝑟 𝑑𝑥 𝑑𝑦 𝑑𝑧
So that = 𝑖+ 𝑗+ 𝑘 … (1)
𝑑𝑠 𝑑𝑠 𝑑𝑠 𝑑𝑠
𝑑𝑟 2 𝑑𝑥 2 𝑑𝑦 2 𝑑𝑧 2
And 𝑑𝑠
= 𝑑𝑠
+ 𝑑𝑠
+ 𝑑𝑠
… (2)

For two dimension curves we have in calculus


2 2 2
𝑑𝑠 = 𝑑𝑥 + 𝑑𝑦
which when extended to the space, becomes
2 2 2 2
𝑑𝑠 = 𝑑𝑥 + 𝑑𝑦 + 𝑑𝑧
𝑑𝑥 2 𝑑𝑦 2 𝑑𝑧 2
Or + + =1
𝑑𝑠 𝑑𝑠 𝑑𝑠
2
𝑑𝑟
Therefore (2) gives =1
𝑑𝑠
𝑑𝑟
That means, is a unit vector along the tangent and (1) represents a unit tangent vector along the
𝑑𝑠
curve C in space.
Therefore, Velocity 𝑣 of the particle at any point of the curve is given by
𝑑𝑟 𝑑𝑟 𝑑𝑠
𝑣 = 𝑑𝑡 = 𝑑𝑠 𝑑𝑡 = 𝑣 𝑇 … (3)
𝑑𝑠 𝑑𝑟
where 𝑣 = and 𝑇 = is the unit vector along the tangent.
𝑑𝑡 𝑑𝑠
𝑑𝑠
Thus 𝑣 = 𝑑𝑡 is the tangential component of the velocity and the normal component of the velocity
is zero.
𝑑𝑣 𝑑(𝑣𝑇) 𝑑𝑣 𝑑𝑇
Next, acceleration 𝑎 = 𝑑𝑡 = 𝑑𝑡 = 𝑑𝑡 𝑇 + 𝑣
𝑑𝑡
2
𝑑 𝑠 𝑑𝑠 𝑑𝑇 𝑑𝑠 𝑑𝑣 𝑑𝑇 𝑑𝜓
or 𝑎 = 2 𝑇+ = 𝑇 + 𝑣2
𝑑𝑡 𝑑𝑡 𝑑𝑠 𝑑𝑡 𝑑𝑡 𝑑𝜓 𝑑𝑠
𝑑𝑣 𝑣 2 𝑑𝑇 𝑑𝜓
= 𝑇+ (since radius of curvature, 𝜌 = ) … (4)
𝑑𝑡 𝜌 𝑑𝜓 𝑑𝑠
From the adjoining figure, 𝑇 = 𝑃𝑄 is along the tangent at P to the curve C and 𝑁 is the unit vector
along the normal to P.
𝜋 𝜋
∴ 𝑇 = cos 𝜓 𝑖 + sin 𝜓 𝑗 and 𝑁 = cos + 𝜓 𝑖 + sin + 𝜓 𝑗 = −sin 𝜓 𝑖 + cos 𝜓 𝑗
2 2
𝑑𝑇
Now = −sin 𝜓 𝑖 + cos 𝜓 𝑗 = 𝑁
𝑑𝜓
𝑑𝑣 𝑣2
Therefore, equation (4) becomes 𝑎= 𝑇+ 𝑁
𝑑𝑡 𝜌
𝑑𝑣 𝑣2
Which shows that tangential and normal components of acceleration at the point P are
𝑑𝑡
and .
𝜌
𝑑𝑣 𝑑𝑣 𝑑𝑠 𝑑𝑣 𝑑𝑣
Since = =v , so the tangential component of acceleration is also written as v .
𝑑𝑡 𝑑𝑠 𝑑𝑡 𝑑𝑠 𝑑𝑠
12
Example 12: Find the radial and transverse acceleration of a particle moving in a plane curve
in Polar coordinates.
Solution: Let the position vector of a moving particle 𝑃(𝑟, 𝜃) be 𝑟 so that
𝑟 = 𝑟 𝑟 = 𝑟 (cos 𝜃 𝑖 + sin 𝜃 𝑗) at any time t.
𝑑𝑟 𝑑𝑟 𝑑𝑟
Then the velocity of the particle is 𝑣 = = 𝑟+𝑟
𝑑𝑡 𝑑𝑡 𝑑𝑡
𝑑𝑟 𝑑𝜃
As 𝑟 = (cos 𝜃 𝑖 + sin 𝜃 𝑗) so = (−sin 𝜃 𝑖 + cos 𝜃 𝑗)
𝑑𝑡 𝑑𝑡
𝑑𝑟 𝑑𝑟 𝑑𝜃
Therefore, is perpendicular to 𝑟 and = i.e. if 𝑢
𝑑𝑡 𝑑𝑡 𝑑𝑡
𝑑𝑟 𝑑𝜃
is a unit vector perpendicular to 𝑟, then = 𝑢
𝑑𝑡 𝑑𝑡
𝑑𝑟 𝑑𝑟 𝑑𝜃
And thus, 𝑣= = 𝑟+𝑟 𝑢
𝑑𝑡 𝑑𝑡 𝑑𝑡
𝑑𝑟 𝑑𝜃
So the radial and transverse components of the velocity are and .
𝑑𝑡 𝑑𝑡
𝑑𝑣 𝑑 2𝑟 𝑑𝑟 𝑑𝑟 𝑑𝑟 𝑑𝜃 𝑑 2𝜃 𝑑𝜃 𝑑𝑢
Also 𝑎= = 𝑟+ + 𝑢 +𝑟 𝑢+𝑟
𝑑𝑡 𝑑𝑡 2 𝑑𝑡 𝑑𝑡 𝑑𝑡 𝑑𝑡 𝑑𝑡 2 𝑑𝑡 𝑑𝑡
𝑑2𝑟 𝑑𝜃 2 𝑑𝑟 𝑑𝜃 𝑑 𝜃 2
= −𝑟 𝑟 + 2 +𝑟 𝑢
𝑑𝑡 2 𝑑𝑡 𝑑𝑡 𝑑𝑡 𝑑𝑡2
𝑑𝑢 𝑑𝜃
𝑠𝑖𝑛𝑐𝑒 𝑢 = − sin 𝜃 𝑖 + cos 𝜃 𝑗 𝑔𝑖𝑣𝑒𝑠 =− 𝑟
𝑑𝑡 𝑑𝑡
𝑑2𝑟 𝑑𝜃 2
Thus radial and transverse components of the acceleration are −𝑟 and
𝑑𝑡 2 𝑑𝑡
𝑑𝑟 𝑑𝜃 𝑑2𝜃
2 +𝑟 .
𝑑𝑡 𝑑𝑡 𝑑𝑡 2

Example 13: A particle moves along the curve 𝒓 = 𝐭 𝟑 − 𝟒𝐭 𝒊 + 𝐭 𝟐 + 𝟒𝐭 𝒋 + 𝟖𝒕𝟐 − 𝟑𝒕𝟑 𝒌


where t denotes the time. Find the magnitudes of acceleration along the tangent and normal at
time t = 2.
𝑑𝑟
Solution: The velocity of the particle is 𝑣 = = 3𝑡 2 − 4 𝑖 + 2𝑡 + 4 𝑗 + 16𝑡 − 9𝑡 2 𝑘
𝑑𝑡
𝑑 2𝑟
And the acceleration is 𝑎 = = 6𝑡 𝑖 + 2 𝑗 + 16 − 18𝑡 𝑘
𝑑𝑡 2
At t = 2, 𝑣 = 8 𝑖 + 8 𝑗 − 4 𝑘 and 𝑎 = 12 𝑖 + 2 𝑗 − 20 𝑘
Since the velocity vector is also the tangent vector to the curve, so the magnitude of acceleration
𝑣 8 𝑖 +8 𝑗 −4 𝑘
along the tangent at t = 2 is = 𝑎 ∙ = 12 𝑖 + 2 𝑗 − 20 𝑘 ∙
𝑣 64+64+16
12 8 + 2 8 + −20 (−4)
= = 16
12
And, the magnitude of acceleration along the normal at t = 2 is
8 𝑖+8 𝑗 −4 𝑘
𝑎 − 𝐶𝑜𝑚𝑝𝑜𝑛𝑒𝑛𝑡 𝑜𝑓 𝑎 along the tangent at 𝑡 = 2 = 12 𝑖 + 2 𝑗 − 20 𝑘 − 16 12
4 𝑖 −26 𝑗 −44 𝑘
= = 2 73
3

Example 14: A person going east wards with a velocity of 4 km per hour, finds that the wind
appears to blow directly from the north. He doubles his speed and the wind seems to come
from north-east. Find the actual velocity of the wind.

13
Solution: Let the actual velocity of the wind is 𝑣 = 𝑥𝑖 + 𝑦𝑗, where 𝑖 and 𝑗 represent velocities of 1
km per hour towards the east and north respectively. As the person is going eastwards with a
velocity of 4 km per hour, his actual velocity is 4𝑖.
Then the velocity of the wind relative to the man is 𝑥𝑖 + 𝑦𝑗 − 4𝑖, which is parallel to −𝑗, as it
appears to blow from the north. Hence x = 4.
When the velocity of the person becomes 8𝑖, the velocity of the wind relative to a man is 𝑥𝑖 + 𝑦𝑗 −
8𝑖. But this is parallel to − 𝑖 + 𝑗 .
𝑥−8
∴ = 1 which gives 𝑦 = −4 (using (1))
𝑦
Hence the actual velocity of the wind is 4𝑖 − 4𝑗 i.e. 4 2 km per hour towards south east.

Example 15: A particle moves along the curve 𝒙 = 𝒕𝟑 + 𝟏, 𝒚 = 𝒕𝟐 , 𝒛 = 𝟐𝒕 + 𝟑 where t is the


time. Find the components of velocity and acceleration at t = 1in the direction of the vector
𝒊 + 𝒋 + 𝟑𝒌.

Solution: Let 𝑟 be the position vector of the particle at any time t,


then 𝑟 = 𝑡 3 + 1 𝑖 + 𝑡 2 𝑗 + 2𝑡 + 3 𝑘
𝑑𝑟
So the velocity is 𝑣 = = 3𝑡 2 𝑖 + 2𝑡 𝑗 + 2 𝑘
𝑑𝑡
𝑑 2𝑟
And the acceleration is 𝑎 = = 6𝑡 𝑖 + 2 𝑗 + 0 𝑘
𝑑𝑡 2
At t =1, 𝑣 = 3 𝑖 + 2 𝑗 + 2 𝑘 and 𝑎 = 6 𝑖 + 2 𝑗 + 0 𝑘
𝑖 +𝑗 +3𝑘 𝑖 +𝑗 +3𝑘
Also the unit vector in the direction of the given vector 𝑖 + 𝑗 + 3𝑘 is = =
𝑖 +𝑗 +3𝑘 11
Now the component of velocity at t = 1, in the direction of the vector 𝑖 + 𝑗 + 3𝑘 =
3 𝑖+2 𝑗+2 𝑘 ∙ 𝑖 +𝑗 +3𝑘 3+2+6
= = 11
11 11
And the component of acceleration at t = 1, in the direction of the vector 𝑖 + 𝑗 + 3𝑘 =
6 𝑖+2 𝑗+0 𝑘 ∙ 𝑖 +𝑗 +3𝑘 6+2+0 8
= =
11 11 11

ASSIGNMENT 2

1. The particle moves along a curve 𝑥 = 𝑒 −𝑡 , 𝑦 = 2 cos 3𝑡 , 𝑧 = 2 sin 3𝑡, where t is the time variable.
Determine its velocity and acceleration vectors and also the magnitudes of velocity and
acceleration at 𝑡 = 0.
2. A particle (with position vector 𝑟 ) is moving in a circle with constant angular velocity 𝜔. Show
by vector methods, that the acceleration is equal to −𝜔2 𝑟.
3. A particle moves on the curve 𝑥 = 2𝑡 2 , 𝑦 = 𝑡 2 − 4𝑡, 𝑧 = 3𝑡 − 5, where t is the time. Find the
components of velocity and acceleration at time t = 1 in the direction 𝑖 − 3𝑗 + 2𝑘.
4. The position vector of a particle at time t is 𝑟 = cos(𝑡 − 1) 𝑖 + sinh 𝑡 − 1 𝑗 + 𝑎𝑡 3 𝑘 . Find the
condition imposed on a by requiring that at time t = 1, the acceleration is normal to the position
vector.
14
5. A particle moves so that its position vector is given by 𝑟 = cos 𝜔𝑡 𝑖 + sin 𝜔𝑡 𝑗. Show that the
velocity 𝑣 of the particle is perpendicular to 𝑟 and 𝑟 × 𝑣 is a constant vector.
6. A particle moves along a catenary 𝑠 = 𝑐 tan 𝜓. The direction of acceleration at any point makes
equal angles with the tangent and normal to the path at that point. If the speed at vertex (𝜓 = 0)
be 𝑣0 , show that the magnitude of velocity and acceleration at any point are given by 𝑣0 𝑒 𝜓 and
2
𝑐 0
𝑣 2 𝑒 2𝜓 cos 2 𝜓 respectively.

7. The position vector of a moving particle at a time t is 𝑟 = 𝑡 2 𝑖 − 𝑡 3 𝑗 + 𝑡 4 𝑘. Find the tangential


and normal components of acceleration at t = 1.
8. A vessel A is a sailing with a velocity of 11 knots per hour in the direction south-east and a
second vessel B is sailing with a velocity of 13 knots per hour in a direction 300 of north. Find
the velocity of A relative to B.
9. The velocity of a boat relative to water is represented by 3𝑖 + 4𝑗 and that of water relative to
earth is 𝑖 − 3𝑗. What is the velocity of the boat relative to earth if 𝑖 and 𝑗 represent one KM an
hour east and north respectively?
10. A person travelling towards the north-east with a velocity of 6 KM per hour finds that the wind
appears to blow from the north, but when he doubles his speed it seems to come from a
direction inclined at an angle tan−1 2 to the north of east. Show that the actual velocity of the
wind is 3 2 KM per hour towards the east.

17.5 DEL APPLIED TO SCALAR POINT FUNCTIONS: GRADIENT [KUK 2009]

Del Operator: Del operator is a vector differential operator and is written as


𝜕 𝜕 𝜕
𝛁= 𝑖 +𝑗 +𝑘
𝜕𝑥 𝜕𝑦 𝜕𝑧

Gradient of a Scalar Function: Let ∅(𝑥, 𝑦, 𝑧) be a scalar function of three variables defined over a
region R of space. Then gradient of is a vector function defined as
𝜕∅ 𝜕∅ 𝜕∅
∅=𝑖 + 𝑗 𝜕𝑦 + 𝑘 𝜕𝑧 ,
𝜕𝑥

wherever the partial derivatives exists. It may also be denoted as 𝑔𝑟𝑎𝑑(∅). The del operator is also
called the gradient operator.

Level Surface: Let ∅(𝑥, 𝑦, 𝑧) be a scalar valued function and C is a constant. The surface given by
∅ 𝑥, 𝑦, 𝑧 = 𝐶 through a point 𝑃(𝑟) is such that at each point on it the function has same value, is
called the level surface of ∅ 𝑥, 𝑦, 𝑧 through P, e.g. equi-potential or isothermal surfaces.

In other words, locus of the point 𝑃(𝑟) satisfying ∅ 𝑟 = 𝐶 form a surface through P. This surface is
called the level surface through P.

Gradient as Normal or Geometrical Interpretation of Gradient

15
Let ∅ 𝑟 = ∅ 𝑥, 𝑦, 𝑧 is a scalar function where 𝑟 = 𝑥𝑖 + 𝑦𝑗 + 𝑧𝑘 . And ∅ 𝑥, 𝑦, 𝑧 = 𝐶 is the level
surface of ∅ through 𝑃(𝑟). Let 𝑄(𝑟 + 𝛿𝑟) be a point on neighboring level surface ∅ + 𝛿∅, then

𝜕∅ 𝜕∅ 𝜕∅
𝛁∅ ∙ δr = 𝑖 +𝑗 +𝑘 ∙ 𝛿𝑥𝑖 + 𝛿𝑦𝑗 + 𝛿𝑧𝑘
𝜕𝑥 𝜕𝑦 𝜕𝑧

𝜕∅ 𝜕∅ 𝜕∅
= 𝛿𝑥 + 𝛿𝑦 + 𝛿𝑧 = 𝛿∅ … (1)
𝜕𝑥 𝜕𝑦 𝜕𝑧

Now if P and Q lie on same level surface i.e. ∅ & ∅ + 𝛿∅ are


same, then 𝛿∅ = 0.

Implies 𝛁∅ ∙ δr = 0 (using (1))

Therefore, 𝛁∅ is perpendicular (normal) to every δr lying on


this surface.

Hence, 𝛁∅ is normal to the surface ∅ 𝑥, 𝑦, 𝑧 = 𝐶 and we can write 𝛁∅ = 𝛁∅ n, where 𝑛 is unit


normal vector to the surface.

See the Fig. 17.7, if the perpendicular distance PM between the surfaces through P and Q be 𝛿𝑛, then
rate of change of ∅ along the normal to the surface through P is

𝜕∅ 𝛿∅ 𝛁∅∙δr
= 𝐿𝑡𝛿𝑛 →0 = 𝐿𝑡𝛿𝑛 →0 (using (1))
𝜕𝑛 𝛿𝑛 𝛿𝑛

𝛁∅ n ∙δr n ∙δr
= Lt 𝛿𝑛 →0 = 𝛁∅ Lt 𝛿𝑛 →0 … (2)
𝛿𝑛 𝛿𝑛

δn
= 𝛁∅ Lt 𝛿𝑛 →0 = 𝛁∅ (𝐴𝑠 𝑛 ∙ 𝛿𝑟 = 𝛿𝑟 cos 𝜃 = 𝛿𝑛)
𝛿𝑛

𝜕∅
Hence the magnitude of 𝛁∅ i.e. 𝛁∅ = … (3)
𝜕𝑛

Thus 𝑔𝑟𝑎𝑑 ∅ is normal vector to the level surface ∅ 𝑥, 𝑦, 𝑧 = 𝐶 and its magnitude represents the rate
of change of ∅ along this normal.

Directional Derivative: If 𝛿𝑟 denotes the length PQ and 𝒖 be the unit vector in the direction of PQ,
𝛿𝑓 𝜕𝑓
the limiting value of as 𝛿𝑟 → 0 (i.e. ) is known as the directional derivative of f along the
𝛿𝑟 𝜕𝑟
direction PQ.
𝛿𝑛 𝛿𝑛 𝜕𝑓 𝛿𝑓 𝛿𝑓
Since 𝛿𝑟 = = , therefore = Lt 𝛿𝑟 →0 𝑛 ∙ 𝑢 =𝑢∙ 𝑛 = 𝑢 ∙ 𝛁𝑓
cos 𝛼 𝑛 ∙𝑢 𝜕𝑟 𝛿𝑛 𝛿𝑛

Thus the directional derivative of f in the direction of 𝑢 is the resolved part of 𝛁𝑓 in the direction of 𝑢.
Since 𝛁𝑓 ∙ 𝑢 = 𝛁𝑓 cos 𝛼 ≤ 𝛁𝑓
It follows that 𝛁𝑓 gives the maximum rate of change of f.

Properties of Gradient Operator: Let ∅(𝑥, 𝑦, 𝑧) & ψ(𝑥, 𝑦, 𝑧) are two differentiable scalar functions
defined over some region R. then the gradient operator has following properties:

16
(i) Gradient of a constant multiple of scalar function ∅
𝑔𝑟𝑎𝑑 𝐶∅ = 𝐶 𝑔𝑟𝑎𝑑(∅) or ∇ 𝐶∅ = 𝐶 ∇∅

Proof: Consider 𝑔𝑟𝑎𝑑 𝐶∅ = ∇ 𝐶∅


𝜕 𝜕 𝜕
= 𝑖 +𝑗 +𝑘 𝐶∅
𝜕𝑥 𝜕𝑦 𝜕𝑧

𝜕 𝜕 𝜕
= 𝑖 𝜕𝑥 (𝐶∅) + 𝑗 𝜕𝑦 (𝐶∅) + 𝑘 𝜕𝑧 (𝐶∅)

𝜕∅ 𝜕∅ 𝜕∅
= 𝐶 𝑖 𝜕𝑥 + 𝐶 𝑗 𝜕𝑦 + 𝐶 𝑘 𝜕𝑧

𝜕∅ 𝜕∅ 𝜕∅
= 𝐶 𝑖 𝜕𝑥 + 𝑗 𝜕𝑦 + 𝑘 𝜕𝑧

𝜕 𝜕 𝜕
= 𝐶 𝑖 𝜕𝑥 + 𝑗 𝜕𝑦 + 𝑘 𝜕𝑧 ∅

= 𝐶 ∇∅ = 𝐶 𝑔𝑟𝑎𝑑 ∅

Hence 𝑔𝑟𝑎𝑑 𝐶∅ = 𝐶 𝑔𝑟𝑎𝑑(∅).

(ii) Gradient of sum or difference of two scalar functions


𝑔𝑟𝑎𝑑 ∅ ± 𝜓 = 𝑔𝑟𝑎𝑑 ∅ ± 𝑔𝑟𝑎𝑑(𝜓) or ∇ ∅ ± 𝜓 = ∇∅ ± ∇𝜓

Proof: Consider 𝑔𝑟𝑎𝑑 ∅ ± 𝜓 = ∇ ∅ ± 𝜓


𝜕 𝜕 𝜕
= 𝑖 𝜕𝑥 + 𝑗 𝜕𝑦 + 𝑘 𝜕𝑧 ∅±𝜓

𝜕 𝜕 𝜕
= 𝑖 𝜕𝑥 ∅ ± 𝜓 + 𝑗 𝜕𝑦 ∅ ± 𝜓 + 𝑘 𝜕𝑧 ∅ ± 𝜓

𝜕∅ 𝜕𝜓 𝜕∅ 𝜕𝜓 𝜕∅ 𝜕𝜓
=𝑖 ± 𝜕𝑥 + 𝑗 ± 𝜕𝑦 + 𝑘 ±
𝜕𝑥 𝜕𝑦 𝜕𝑧 𝜕𝑧

𝜕∅ 𝜕∅ 𝜕∅ 𝜕𝜓 𝜕𝜓 𝜕𝜓
= 𝑖 𝜕𝑥 + 𝑗 𝜕𝑦 + 𝑘 𝜕𝑧 ± 𝑖 𝜕𝑥 + 𝑗 𝜕𝑦 + 𝑘 𝜕𝑧

𝜕 𝜕 𝜕 𝜕 𝜕 𝜕
= 𝑖 𝜕𝑥 + 𝑗 𝜕𝑦 + 𝑘 𝜕𝑧 ∅ ± 𝑖 𝜕𝑥 + 𝑗 𝜕𝑦 + 𝑘 𝜕𝑧 𝜓

= ∇∅ ± ∇𝜓 = 𝑔𝑟𝑎𝑑 ∅ ± 𝑔𝑟𝑎𝑑(𝜓).

Hence 𝑔𝑟𝑎𝑑 ∅ ± 𝜓 = 𝑔𝑟𝑎𝑑 ∅ ± 𝑔𝑟𝑎𝑑(𝜓).

(iii) Gradient of product of two scalar functions


𝑔𝑟𝑎𝑑 ∅𝜓 = ∅ 𝑔𝑟𝑎𝑑 𝜓 + 𝜓 𝑔𝑟𝑎𝑑(∅) or ∇ ∅ 𝜓 = ∅ ∇𝜓 + 𝜓 ∇∅

Proof: Consider 𝑔𝑟𝑎𝑑 ∅𝜓 = ∇ ∅ 𝜓

𝜕 𝜕 𝜕
= 𝑖 𝜕𝑥 + 𝑗 𝜕𝑦 + 𝑘 𝜕𝑧 ∅𝜓

𝜕 𝜕 𝜕
= 𝑖 𝜕𝑥 ∅ 𝜓 + 𝑗 𝜕𝑦 ∅ 𝜓 + 𝑘 𝜕𝑧 ∅ 𝜓

17
𝜕𝜓 𝜕∅ 𝜕𝜓 𝜕∅ 𝜕𝜓 𝜕∅
=𝑖 ∅ +𝜓 +𝑗 ∅ +𝜓 + 𝑘 ∅ +𝜓
𝜕𝑥 𝜕𝑥 𝜕𝑦 𝜕𝑦 𝜕𝑧 𝜕𝑧

𝜕𝜓 𝜕𝜓 𝜕𝜓 𝜕∅ 𝜕∅ 𝜕∅
= ∅ 𝑖 𝜕𝑥 + 𝑗 𝜕𝑦 + 𝑘 𝜕𝑧 + 𝜓 𝑖 𝜕𝑥 + 𝑗 𝜕𝑦 + 𝑘 𝜕𝑧

= ∅ ∇𝜓 + 𝜓 ∇∅ = ∅ 𝑔𝑟𝑎𝑑 𝜓 ± 𝜓 𝑔𝑟𝑎𝑑(∅)

Hence 𝑔𝑟𝑎𝑑 ∅𝜓 = ∅ 𝑔𝑟𝑎𝑑 𝜓 ± 𝜓 𝑔𝑟𝑎𝑑(∅).

(iv) Gradient of quotient of two scalar functions


∅ 𝜓 𝑔𝑟𝑎𝑑 ∅ −∅ 𝑔𝑟𝑎𝑑 (𝜓 ) ∅ 𝜓 ∇ ∅ −∅ ∇(𝜓 )
𝑔𝑟𝑎𝑑 = 2
or ∇ = , provided 𝜓 ≠ 0.
𝜓 𝜓 𝜓 𝜓 2

∅ ∅
Proof: Consider 𝑔𝑟𝑎𝑑 =∇
𝜓 𝜓

𝜕 𝜕 𝜕 ∅
= 𝑖 𝜕𝑥 + 𝑗 𝜕𝑦 + 𝑘 𝜕𝑧 𝜓

𝜕 ∅ 𝜕 ∅ 𝜕 ∅
= 𝑖 𝜕𝑥 + 𝑗 𝜕𝑦 + 𝑘 𝜕𝑧
𝜓 𝜓 𝜓

𝜕∅ 𝜕𝜓 𝜕∅ 𝜕𝜓 𝜕∅ 𝜕𝜓
𝜓 −∅ 𝜓 −∅ 𝜓 −∅
𝜕𝑥 𝜕𝑥 𝜕𝑦 𝜕𝑦 𝜕𝑧 𝜕𝑧
=𝑖 +𝑗 + 𝑘
𝜓2 𝜓 2 𝜓2

1 𝜕∅ 𝜕∅ 𝜕∅ 𝜕𝜓 𝜕𝜓 𝜕𝜓
= 𝜓 𝑖 +𝑗 +𝑘 −∅ 𝑖 +𝑗 +𝑘
𝜓2 𝜕𝑥 𝜕𝑦 𝜕𝑧 𝜕𝑥 𝜕𝑦 𝜕𝑧

1 𝜓 𝑔𝑟𝑎𝑑 ∅ −∅ 𝑔𝑟𝑎𝑑 (𝜓 )
= 𝜓 2 𝜓 𝑔𝑟𝑎𝑑(∅) − ∅ 𝑔𝑟𝑎𝑑(𝜓) =
𝜓 2

𝜓 ∇ ∅ −∅ ∇(𝜓 )
=
𝜓 2

∅ 𝜓 𝑔𝑟𝑎𝑑 ∅ −∅ 𝑔𝑟𝑎𝑑 (𝜓 )
Hence 𝑔𝑟𝑎𝑑 = .
𝜓 𝜓 2

Example 16: If ∅ = 𝟑𝒙𝟐 𝒚 − 𝒚𝟑 𝒛𝟐 , then find 𝒈𝒓𝒂𝒅 ∅ at (1, -2, -1).

Solution: 𝑔𝑟𝑎𝑑 ∅ = ∇ 3𝑥 2 𝑦 − 𝑦 3 𝑧 2
𝜕 𝜕 𝜕
= 𝑖 𝜕𝑥 3𝑥 2 𝑦 − 𝑦 3 𝑧 2 + 𝑗 𝜕𝑦 3𝑥 2 𝑦 − 𝑦 3 𝑧 2 + 𝑘 𝜕𝑧 3𝑥 2 𝑦 − 𝑦 3 𝑧 2

= 𝑖 6𝑥𝑦 + 𝑗 3𝑥 2 − 3𝑦 2 𝑧 2 + 𝑘 −2𝑦 3 𝑧
2 2 2 3
At (1, -2, -1), 𝑔𝑟𝑎𝑑 ∅ = 𝑖 6 1 −2 + 𝑗 3 1 − 3 −2 −1 + 𝑘 −2 −2 −1
= −12𝑖 − 9𝑗 − 16𝑘

Example 17: Prove that 𝛁 𝒓𝒏 = 𝒏 𝒓𝒏−𝟐 𝒓, where 𝒓 = 𝒙𝒊 + 𝒚𝒋 + 𝒛𝒌.

Solution: Here 𝑟 = 𝑥𝑖 + 𝑦𝑗 + 𝑧𝑘 and 𝑟2 = 𝑥2 + 𝑦2 + 𝑧2

𝜕𝑟 𝑥 𝜕𝑟 𝑦 𝜕𝑟 𝑧
So differentiating partially w. r. t. x, = , Similarly, = and = … (1)
𝜕𝑥 𝑟 𝜕𝑦 𝑟 𝜕𝑧 𝑟

18
𝜕 𝜕 𝜕
Consider ∇ 𝑟 𝑛 = 𝑖 𝑟𝑛 + 𝑗 𝑟𝑛 + 𝑘 𝑟𝑛
𝜕𝑥 𝜕𝑦 𝜕𝑧

𝜕𝑟 𝜕𝑟 𝜕𝑟
= 𝑖 𝑛 𝑟 𝑛 −1 + 𝑗 𝑛 𝑟 𝑛 −1 + 𝑘 𝑛 𝑟 𝑛−1
𝜕𝑥 𝜕𝑦 𝜕𝑧

𝑥 𝑦 𝑧
= 𝑖 𝑛 𝑟 𝑛 −1 + 𝑗 𝑛 𝑟 𝑛 −1 + 𝑘 𝑛 𝑟 𝑛 −1
𝑟 𝑟 𝑟

= 𝑖 𝑛 𝑟 𝑛 −2 𝑥 + 𝑗 𝑛 𝑟 𝑛 −2 𝑦 + 𝑘 𝑛 𝑟 𝑛 −2 𝑧

= 𝑛 𝑟 𝑛−2 𝑥𝑖 + 𝑦𝑗 + 𝑧𝑘 = 𝑛 𝑟𝑛−2 𝑟
Hence ∇ 𝑟 𝑛 = 𝑛 𝑟 𝑛−2 𝑟

Example 18: Find the unit vector normal to the surface 𝒙𝟑 + 𝒚𝟑 + 𝟑𝒙𝒚𝒛 = 𝟑 at

(i) the point (1, 2, -1) * (ii) the point (1, 3, -1)** [KUK *2006, **2011]

Solution: We know that a vector normal to a surface is given by its gradient, so if 𝑛 is the vector
normal to the given surface then
𝜕 𝜕 𝜕
𝑛 = ∇ 𝑥 3 + 𝑦 3 + 3𝑥𝑦𝑧 − 3 = 𝑖 +𝑗 +𝑘 𝑥 3 + 𝑦 3 + 3𝑥𝑦𝑧 − 3
𝜕𝑥 𝜕𝑦 𝜕𝑧

= 3𝑥 2 + 3𝑦𝑧 𝑖 + 3𝑦 2 + 3𝑥𝑧 𝑗 + 3𝑥𝑦 𝑘


2 2
(i) At point (1, 2, -1), 𝑛= 3 1 + 3 2 −1 𝑖 + 3 2 + 3 1 −1 𝑗 + 3 1 2 𝑘
= −𝟑𝑖 + 9𝑗 + 6𝑘

Also 𝑛 = −3 2 + 9 2 + 6 2 = 126
Therefore the unit normal vector to the given surface at a point (1, 2, -1) is
1
𝑛= −𝟑𝑖 + 9𝑗 + 6𝑘 .
126

2 2
(ii) At point (1, 3, -1), 𝑛= 3 1 + 3 3 −1 𝑖 + 3 3 + 3 1 −1 𝑗 + 3 1 3 𝑘
= −6𝑖 + 24𝑗 + 9𝑘
Also 𝑛 = −6 2 + 24 2 + 9 2 = 693
Therefore the unit normal vector to the given surface at a point (1, 3, -1) is
1
𝑛= – 6 𝑖 + 24 𝑗 + 9 𝑘 .
693

𝟐 +𝒚𝟐 +𝒛𝟐 ) 𝟐
Example 19: Show that 𝒈𝒓𝒂𝒅 𝒆(𝒙 = 𝟐𝒆𝒓 , 𝐰𝐡𝐞𝐫𝐞 𝒓𝟐 = 𝒓 𝟐
= 𝒙𝟐 + 𝒚𝟐 + 𝒛𝟐.

Solution: Here 𝑟 2 = 𝑥 2 + 𝑦 2 + 𝑧 2
𝜕𝑟 𝑥 𝜕𝑟 𝑦 𝜕𝑟 𝑧
So differentiating w. r. t. x, = , Similarly, = and = .
𝜕𝑥 𝑟 𝜕𝑦 𝑟 𝜕𝑧 𝑟

𝑥 2 +𝑦 2 +𝑧 2 2 𝜕 2 𝜕 2 𝜕 2
Now 𝑔𝑟𝑎𝑑 𝑒 = ∇ 𝑒 𝑟 = 𝑖 𝜕𝑥 𝑒 𝑟 + 𝑗 𝜕𝑦 𝑒 𝑟 + 𝑘 𝜕𝑧 𝑒 𝑟
2 𝜕𝑟 2 𝜕𝑟 2 𝜕𝑟
= 𝑖 𝑒 𝑟 2𝑟 𝜕𝑥 + 𝑗 𝑒 𝑟 2𝑟 𝜕𝑦 + 𝑘 𝑒 𝑟 2𝑟 𝜕𝑧

19
2 𝜕𝑟 𝜕𝑟 𝜕𝑟 2 𝑥 𝑦 𝑧
= 𝑒 𝑟 2𝑟 𝑖 +𝑗 + 𝑘 𝜕𝑧 = 𝑒 𝑟 2𝑟 𝑖 +𝑗 +𝑘𝑟
𝜕𝑥 𝜕𝑦 𝑟 𝑟

𝑟2
= 2𝑒 𝑟
Example 20: If 𝒖 = 𝒙 + 𝒚 + 𝒛, 𝒗 = 𝒙𝟐 + 𝒚𝟐 + 𝒛𝟐 𝐚𝐧𝐝 𝒘 = 𝒚𝒛 + 𝒛𝒙 + 𝒙𝒚 , then show that
𝒈𝒓𝒂𝒅 𝒖, 𝒈𝒓𝒂𝒅 𝒗 𝐚𝐧𝐝 𝒈𝒓𝒂𝒅 𝒘 are coplanar.

𝜕𝑢 𝜕𝑢 𝜕𝑢
Solution: Consider 𝑔𝑟𝑎𝑑 𝑢 = 𝑖 +𝑗 +𝑘 =𝑖 1 +𝑗 1 +𝑘 1 =𝑖+𝑗+𝑘
𝜕𝑥 𝜕𝑦 𝜕𝑧

𝜕𝑣 𝜕𝑣 𝜕𝑣
𝑔𝑟𝑎𝑑 𝑣 = 𝑖 +𝑗 +𝑘 = 𝑖 2𝑥 + 𝑗 2𝑦 + 𝑘 2𝑧 = 2𝑥 𝑖 + 2𝑦 𝑗 + 2𝑧 𝑘
𝜕𝑥 𝜕𝑦 𝜕𝑧

𝜕𝑤 𝜕𝑤 𝜕𝑤
𝑔𝑟𝑎𝑑 𝑤 = 𝑖 +𝑗 +𝑘 = 𝑦+𝑧 𝑖+ 𝑧+𝑥 𝑗+ 𝑥+𝑦 𝑘
𝜕𝑥 𝜕𝑦 𝜕𝑧

We know that three vectors 𝑎, 𝑏 𝑎𝑛𝑑 𝑐 are coplanar if their scalar triple product is zero i.e.
𝑎 𝑏 𝑐 =0
1 1 1
Consider 𝑔𝑟𝑎𝑑 𝑢 𝑔𝑟𝑎𝑑 𝑣 𝑔𝑟𝑎𝑑 𝑤 = 2𝑥 2𝑦 2𝑧
𝑦+𝑧 𝑧+𝑥 𝑥+𝑦

1 1 1
=2 𝑥 𝑦 𝑧 (taking common 2 from R2)
𝑦+𝑧 𝑧+𝑥 𝑥+𝑦

1 1 1
=2 𝑥+𝑦+𝑧 𝑥+𝑦+𝑧 𝑥 + 𝑦 + 𝑧 (adding R2 and R3)
𝑦+𝑧 𝑧+𝑥 𝑥+𝑦

1 1 1
= 2(𝑥 + 𝑦 + 𝑧) 1 1 1
𝑦+𝑧 𝑧+𝑥 𝑥+𝑦

(taking common (x + y + z) from R2)

=2 𝑥+𝑦+𝑧 0 =0

Hence 𝑔𝑟𝑎𝑑 𝑢, 𝑔𝑟𝑎𝑑 𝑣 and 𝑔𝑟𝑎𝑑 𝑤 are coplanar.

Example 21: Show that 𝒈𝒓𝒂𝒅 𝒇 𝒓 × 𝒓 = 𝟎.


𝜕 𝜕 𝜕
Solution: Here 𝑔𝑟𝑎𝑑 𝑓 𝑟 = ∇ 𝑓(𝑟) = 𝑖 𝜕𝑥 𝑓(𝑟) + 𝑗 𝜕𝑦 𝑓(𝑟) + 𝑘 𝜕𝑧 𝑓(𝑟)
𝜕𝑟 𝜕𝑟 𝜕𝑟
= 𝑖 𝑓 ′ (𝑟) + 𝑗 𝑓 ′ (𝑟) 𝜕𝑦 + 𝑘 𝑓 ′ (𝑟) 𝜕𝑧
𝜕𝑥
𝜕𝑟 𝜕𝑟 𝜕𝑟 𝑥 𝑦 𝑧
= 𝑓 ′ (𝑟) 𝑖 +𝑗 + 𝑘 𝜕𝑧 = 𝑓 ′ (𝑟) 𝑖 +𝑗 +𝑘𝑟
𝜕𝑥 𝜕𝑦 𝑟 𝑟

𝑟
= 𝑓 ′ (𝑟) 𝑟

20
𝑟 1
Now 𝑔𝑟𝑎𝑑 𝑓 𝑟 × 𝑟 = 𝑓 ′ (𝑟) × 𝑟 = 𝑓 ′ (𝑟) 𝑟×𝑟 =0
𝑟 𝑟

𝑠𝑖𝑛𝑐𝑒 𝑟 × 𝑟 = 0
Example 22: Find the directional derivative of 𝒇 𝒙, 𝒚, 𝒛 = 𝒙𝟐 𝒚𝟐 𝒛𝟐 at the point (1, 1, -1) in the
direction of the tangent to the curve 𝒙 = 𝒆𝒕 , 𝒚 = 𝟐 𝒔𝒊𝒏 𝒕 + 𝟏, 𝒛 = 𝒕 − 𝒄𝒐𝒔 𝒕 at t = 0.

Solution: Consider ∇ 𝑓 𝑥, 𝑦, 𝑧 = ∇ 𝑥 2 𝑦 2 𝑧 2 = 2𝑥𝑦 2 𝑧 2 𝑖 + 2𝑦𝑥 2 𝑧 2 𝑗 + 2𝑧𝑥 2 𝑦 2 𝑘


At (1, 1, -1), ∇ 𝑓 𝑥, 𝑦, 𝑧 = 2 𝑖 + 2 𝑗 − 2 𝑘
Now 𝑟 = 𝑥𝑖 + 𝑦𝑗 + 𝑧𝑘 = 𝑒 𝑡 𝑖 + 2 sin 𝑡 + 1 𝑗 + (𝑡 − cos 𝑡)𝑘
𝑑𝑟
So tangent to the curve is = 𝑒 𝑡 𝑖 + 2 cos 𝑡 𝑗 + (1 + sin 𝑡)𝑘
𝑑𝑡
𝑑𝑟
At t = 0, = 𝑖 + 2𝑗 + 𝑘
𝑑𝑡
𝑑𝑟 𝑑𝑟 𝑖+2𝑗+𝑘
And the unit tangent vector is / =
𝑑𝑡 𝑑𝑡 6
𝑑𝑟 𝑑𝑟
So the required directional derivative in the direction of the tangent is ∇𝑓 𝑥, 𝑦, 𝑧 ∙ /
𝑑𝑡 𝑑𝑡

4 2 3
= 2 𝑖 + 2 𝑗 − 2 𝑘 ∙ 𝑖 + 2𝑗 + 𝑘 / 6 = =
6 3

Example 23: If the directional derivative ∅ = 𝒂 𝒙𝟐 𝒚 + 𝒃 𝒚𝟐 𝒛 + 𝒄 𝒛𝟐 𝒙 at the point (1, 1, 1) has


𝒙−𝟏 𝒚−𝟑 𝒛
maximum magnitude 15 in the direction parallel to the line = = , find the values of
𝟐 −𝟐 𝟏
a, b and c. [Madrass 2004]

Solution: Consider ∇ ∅ = ∇ (𝑎 𝑥 2 𝑦 + 𝑏 𝑦 2 𝑧 + 𝑐 𝑧 2 𝑥)
= 2𝑎 𝑥𝑦 + 𝑐 𝑧 2 𝑖 + 𝑎 𝑥 2 + 2𝑏 𝑦𝑧 𝑗 + (𝑏 𝑦 2 + 2𝑐 𝑧𝑥)𝑘
At (1, 1, 1), ∇ ∅ = 2𝑎 + 𝑐 𝑖 + 𝑎 + 2𝑏 𝑗 + (𝑏 + 2𝑐)𝑘
We know that directional derivative of ∅ is maximum in the direction of its normal vector ∇ ∅, but it
𝑥−1 𝑦−3 𝑧
is given to be maximum in the direction of the line = = .
2 −2 1
Therefore, the line and normal vector are parallel to each other, which results as:
2𝑎+𝑐 𝑎+2𝑏 𝑏+2𝑐
= = … (1)
2 −2 1
Taking first two members of (1), 3𝑎 + 2𝑏 + 𝑐 = 0
and by last two members of (1), 𝑎 + 4𝑏 + 4𝑐 = 0
𝑎 𝑏 𝑐
Solving the two obtained equations, = = = 𝜆 𝐿𝑒𝑡
4 −11 10
=> 𝑎 = 4𝜆, 𝑏 = −11𝜆 𝑎𝑛𝑑 𝑐 = 10𝜆 … (2)
Also given that maximum magnitude of directional derivative is 15 units i.e. ∇∅ = 15
So , 2𝑎 + 𝑐 2
+ 𝑎 + 2𝑏 2
+ 𝑏 + 2𝑐 2
= 15 2
…(3)
Putting the values of a, b and c from (2),

21
5
8𝜆 + 10𝜆 2
+ 4𝜆 − 22𝜆 2
+ −11𝜆 + 20𝜆 2
= 15 2
=> 𝜆 = ±
9
20 55 50
Hence 𝑎=± , 𝑏=∓ , 𝑐=± .
9 9 9

Example 24: In what direction from (3, 1, -2) is the directional derivative of ∅ = 𝒙𝟐 𝒚𝟐 𝒛𝟒
maximum? Find also the magnitude of this maximum. [KUK 2010, 2007, 2006]
Solution: The vector normal to the given surface is

𝜕 𝜕 𝜕
𝑛 = ∇ ∅ = ∇ 𝑥2 𝑦2𝑧4 = 𝑖 +𝑗 +𝑘 𝑥2 𝑦2𝑧4
𝜕𝑥 𝜕𝑦 𝜕𝑧

= 2𝑥𝑦 2 𝑧 4 𝑖 + 2𝑥 2 𝑦𝑧 4 𝑗 + 4𝑥 2 𝑦 2 𝑧 3 𝑘
2 4 2 4 2 2 3
At point (3, 1, -2) 𝑛=2 3 1 −2 𝑖+2 3 1 −2 𝑗+4 3 1 −2 𝑘

= 96 𝑖 + 288 𝑗 − 288 𝑘

Also 𝑛 = 96 2 + 288 2 + −288 2 = 96 19

So the directional derivative of given surface will be maximum in the direction of


96 𝑖 + 288 𝑗 − 288 𝑘 and the magnitude of this maximum is 96 19.

Example 25: Find the angle between the surfaces 𝒙𝟐 + 𝒚𝟐 + 𝒛𝟐 = 𝟗 and 𝒛 = 𝒙𝟐 + 𝒚𝟐 − 𝟑 at


(2, -1, 2). [KUK 2008]

Solution: Given surfaces are


∅1 = 𝑥 2 + 𝑦 2 + 𝑧 2 − 9 = 0 … (1)
and ∅2 = 𝑥 2 + 𝑦 2 − 𝑧 − 3 = 0 … (2)
We know that gradient of a surface gives the vector normal to the surface. Let 𝑛1 and 𝑛2 are the
vectors normal to the surfaces (1) and (2) respectively.
𝜕 𝜕 𝜕
Now ∇∅1 = 𝑖 +𝑗 +𝑘 ∙ 𝑥 2 + 𝑦 2 + 𝑧 2 = 9 = 2𝑥 𝑖 + 2𝑦 𝑗 + 2𝑧 𝑘 … (3)
𝜕𝑥 𝜕𝑦 𝜕𝑧
𝜕 𝜕 𝜕
∇∅2 = 𝑖 +𝑗 +𝑘 ∙ 𝑥 2 + 𝑦 2 − 𝑧 − 3 = 2𝑥 𝑖 + 2𝑦 𝑗 − 𝑘 … (4)
𝜕𝑥 𝜕𝑦 𝜕𝑧
So, 𝑛1 = ∇∅1 at (2,−1,2) = 4 𝑖 − 2 𝑗 + 4 𝑘 and 𝑛2 = ∇∅2 at (2,−1,2) =4𝑖−2𝑗− 𝑘
Let 𝜃 be the angle between the given surfaces at point (2, -1, 2), then 𝜃 will also be an angle between
their normals 𝑛1 and 𝑛2 .
𝑛 1 ∙𝑛 2 4 𝑖 −2 𝑗 +4 𝑘 ∙ 4 𝑖 −2 𝑗 − 𝑘 16+4−4 8
Therefore, cos 𝜃 = = = = .
𝑛1 𝑛2 4 𝑖 −2 𝑗 +4 𝑘 4 𝑖 −2 𝑗 − 𝑘 36 21 3 21

Example 26: Find the constants a and b so that the surface 𝒂𝒙𝟐 − 𝒃𝒚𝒛 = 𝒂 + 𝟐 𝒙 is orthogonal
to the surface 𝟒𝒙𝟐 𝒚 + 𝒛𝟑 = 𝟒 at the point (1, -1, 2).
Solution: Given surfaces are
𝑓 = 𝑎𝑥 2 − 𝑏𝑦𝑧 − 𝑎 + 2 𝑥 = 0 … (1)
and 𝑔 = 4𝑥 2 𝑦 + 𝑧 3 − 4 = 0 …(2)

22
Let 𝑛1 and 𝑛2 are the vectors normal to the surfaces (1) and (2) at (1, -1, 2), respectively.
𝜕𝑓 𝜕𝑓 𝜕𝑓
Consider ∇𝑓 = 𝑖 +𝑗 +𝑘 = 𝑖 2𝑎𝑥 − 𝑎 + 2 + 𝑗 (−𝑏𝑧) + 𝑘 (−𝑏𝑦) … (3)
𝜕𝑥 𝜕𝑦 𝜕𝑧
𝜕𝑔 𝜕𝑔 𝜕𝑔
And ∇𝑔 = 𝑖 +𝑗 +𝑘 = 𝑖 8𝑥𝑦 + 𝑗 (4𝑥 2 ) + 𝑘 (3𝑧 2 ) … (4)
𝜕𝑥 𝜕𝑦 𝜕𝑧

So, 𝑛1 = ∇𝑓 (1, −1, 2) = 𝑖 𝑎 − 2 + 𝑗 −2𝑏 + 𝑘 𝑏 = 𝑎 − 2 𝑖 − 2𝑏𝑗 + 𝑏𝑘

𝑛2 = ∇𝑔 (1, −1, 2) = 𝑖 −8 + 𝑗 4 + 𝑘 12 = −8𝑖 + 4𝑗 + 12𝑘

Given that surfaces (1) and (2) cut orthogonally at (1, -1, 2), so their normal vectors i.e. 𝑛1 and 𝑛2
should also be orthogonal to each other.
Therefore, 𝑛1 . 𝑛2 = 0
=> 𝑎 − 2 𝑖 − 2𝑏𝑗 + 𝑏𝑘 . −8𝑖 + 4𝑗 + 12𝑘 = 0
=> −8 𝑎 − 2 − 8𝑏 + 12𝑏 = 0
=> 2𝑎 − 𝑏 = 4 … (5)
Also the point (1, -2, 1) lies on the surface (1), so we have
𝑎 + 2𝑏 − 𝑎 + 2 = 0 𝑜𝑟 2𝑏 − 2 = 0 𝑜𝑟 𝒃 = 𝟏
𝟓
Putting value of b in (3), we get 2𝑎 − 1 = 4 𝑜𝑟 𝒂 = 𝟐

Example 27: Show that the components of a vector 𝒓 along and normal (perpendicular) to a
𝒓∙𝒂 𝐚× 𝒓×𝒂
vector 𝒂, in the plane of 𝒓 and 𝒂, are 𝒂 and .
𝒂𝟐 𝒂 𝟐

Solution: Let 𝑂𝐴 = 𝑎 and 𝑂𝐵 = 𝑟 and 𝑂𝑀 be the projection


of 𝑟 on 𝑎 (Fig. 17.8)
∴ Component of 𝑟 along 𝑎 = OM (unit vector along 𝑎)
𝑟 ∙𝑎 𝑎 𝑟 ∙𝑎
= 𝑟∙𝑎 𝑎 = = 𝑎
𝑎 𝑎 𝑎2

Also the component of 𝑟 normal to 𝑎 = 𝑀𝐵 = 𝑂𝐵 − 𝑂𝑀


𝑟 ∙𝑎 𝑎 ∙𝑎 𝑟 − 𝑟 ∙𝑎 𝑎 a × 𝑟 ×𝑎
=𝑟− 𝑎= =
𝑎2 𝑎2 𝑎 2

Example 28: If f and 𝑭 are point functions, prove that the components of the latter normal and
𝑭∙𝛁𝒇 𝛁𝒇 𝛁𝒇× 𝑭×𝛁𝒇
tangential to the surface f = 0 are 𝟐
and .
𝛁𝒇 𝛁𝒇 𝟐

Solution: We know that for the given surface f = 0, the vector normal to the surface is given by the
gradient i.e. ∇𝑓.
∇𝑓 ∇𝑓 𝐹 ∙∇𝑓 ∇𝑓 𝐹 ∙∇𝑓 ∇𝑓
Now the component of 𝐹 normal to the given surface is = 𝐹 ∙ = =
∇𝑓 ∇𝑓 ∇𝑓 2 ∇𝑓 2

And the component of 𝐹 tangential to the given surface is = 𝐹 − 𝑡𝑕𝑒 𝑛𝑜𝑟𝑚𝑎𝑙 𝑐𝑜𝑚𝑝𝑜𝑛𝑒𝑛𝑡 𝑜𝑓 𝐹

23
𝐹 ∙∇𝑓 ∇𝑓 𝐹 ∇𝑓 2 − 𝐹 ∙∇𝑓 ∇𝑓 𝐹 ∇𝑓∙∇𝑓 − 𝐹 ∙∇𝑓 ∇𝑓 ∇𝑓× 𝐹 ×∇𝑓
=𝐹− = = =
∇𝑓 2 ∇𝑓 2 ∇𝑓 2 ∇𝑓 2

ASSIGNMENT 3

1. Find ∇∅, if ∅ = log 𝑥 2 + 𝑦 2 + 𝑧 2 .


1 𝑟
2. Show that = − 3.
𝑟 𝑟
3. What is the directional derivative of ∅ = 𝑥𝑦 2 + 𝑦𝑧 3 at the point (2, -1, 1) in the direction of the
normal to the surface 𝑥 log 𝑧 − 𝑦 2 = −4 at (-1, 2, 1)? [JNTU 2005; VTU 2004]
4. What is the directional derivative of ∅ = 𝑥 2 𝑦𝑧 + 4𝑥𝑧 2 at the point (1, -2, 1) in the direction of the
vector 2𝑖 − 𝑗 − 2𝑘 . [VTU 2007; UP Tech, JNTU 2006]
5. The temperature of points in a space is given by 𝑇 𝑥, 𝑦, 𝑧 = 𝑥 2 + 𝑦 2 − 𝑧. A mosquito located at
(1, 1, 2) desires to fly in such a direction that it will get warm as soon as possible. In what
direction should it move?
6. What is the greatest rate of increase of 𝑢 = 𝑥 2 + 𝑦𝑧 2 at the point (1, -1, 3)?
7. Find the angle between the tangent planes to the surfaces 𝑥 log 𝑧 = 𝑦 2 − 1 and 𝑥 2 𝑦 = 2 − 𝑧 at the
point (1, 1, 1). [JNTU 2003]
8. Calculate the angle between the normals to the surface 𝑥𝑦 = 𝑧 2 at the points (4, 1, 2) and
(3, 3, -3).
9. Find the angle between the surfaces 𝑥 2 + 𝑦 2 + 𝑧 2 = 9 and 𝑧 = 𝑥 2 + 𝑦 2 − 3 at (2, -1, 2).
10. Find the values of 𝜆 and 𝜇 so that the surface 𝜆𝑥 2 𝑦 + 𝜇𝑧 3 = 4 may cut the surface
5𝑥 2 = 2𝑦𝑧 + 9𝑥 orthogonally at (1, -1, 2)

17.6 DEL APPLIED TO VECTOR POINT FUNCTIONS (Divergence & Curl)

1. Divergence: Let 𝑓 (𝑥, 𝑦, 𝑧) = 𝑓1 (𝑥, 𝑦, 𝑧)𝑖 + 𝑓2 (𝑥, 𝑦, 𝑧)𝑗 + 𝑓3 (𝑥, 𝑦, 𝑧)𝑘 be a continuously differentiable
vector point function. Divergence 𝑓 (𝑥, 𝑦, 𝑧) of is a scalar which is denoted by ∇ ∙ 𝑓 and is defined as
𝜕 𝜕 𝜕 𝜕𝑓1 𝜕𝑓2 𝜕𝑓3
∇∙𝑓 = 𝑖 +𝑗 +𝑘 ∙ 𝑓1 𝑖 + 𝑓2 𝑗 + 𝑓3 𝑘 = + + .
𝜕𝑥 𝜕𝑦 𝜕𝑧 𝜕𝑥 𝜕𝑦 𝜕𝑧

t is also denoted by 𝑑𝑖𝑣 𝑓.

Physical interpretation of Divergence


Consider the case of fluid flow.
Let 𝑣 = 𝑣𝑥 𝑖 + 𝑣𝑦 𝑗 + 𝑣𝑧 𝑘 be the velocity of the fluid at a
point 𝑃(𝑥, 𝑦, 𝑧) . Consider a small parallelopiped with
edges 𝛿𝑥, 𝛿𝑦 and 𝛿𝑧 parallel to the x, y and z axis

24
respectively in the mass of fluid, with one of its corner at point 𝑃.
So, the mass of fluid flowing in through the face 𝑃𝑄𝑅𝑆 per unit time = 𝑣𝑦 𝛿𝑧 𝛿𝑥
and the mass of fluid flowing out of the face 𝑃′ 𝑄′𝑅′ 𝑆 ′ per unit time
𝜕𝑣𝑦
= 𝑣𝑦 +𝛿𝑦 𝛿𝑧 𝛿𝑥 = 𝑣𝑦 + 𝛿𝑦 𝛿𝑧 𝛿𝑥
𝜕𝑦

∴ The net decrease in fluid mass in the parallelopiped corresponding to flow along y-axis
𝜕𝑣𝑦 𝜕𝑣𝑦
= 𝑣𝑦 + 𝛿𝑦 𝛿𝑧 𝛿𝑥 − 𝑣𝑦 𝛿𝑧 𝛿𝑥 = 𝛿𝑥 𝛿𝑦 𝛿𝑧
𝜕𝑦 𝜕𝑦

Similarly, the net decrease in fluid mass in the parallelopiped corresponding to the flow along x-axis
𝜕𝑣𝑥 𝜕𝑣𝑧
and z-axis is 𝛿𝑥 𝛿𝑦 𝛿𝑧 and 𝛿𝑥 𝛿𝑦 𝛿𝑧 respectively.
𝜕𝑥 𝜕𝑧
So, total decrease in mass of fluid mass in the parallelopiped per unit time
𝜕𝑣𝑥 𝜕𝑣𝑦 𝜕𝑣𝑧
= + + 𝛿𝑥 𝛿𝑦 𝛿𝑧
𝜕𝑥 𝜕𝑦 𝜕𝑧

𝜕𝑣𝑥 𝜕𝑣𝑦 𝜕𝑣𝑧


Thus, the rate of loss of fluid per unit volume = + +
𝜕𝑥 𝜕𝑦 𝜕𝑧
𝜕 𝜕 𝜕
= 𝑖 +𝑗 +𝑘 ∙ 𝑣𝑥 𝑖 + 𝑣𝑦 𝑗 + 𝑣𝑧 𝑘
𝜕𝑥 𝜕𝑦 𝜕𝑧

= ∇ ∙ 𝑣 = 𝑑𝑖𝑣 𝑣
Hence, 𝑑𝑖𝑣 𝑣 gives the rate at which fluid is originating or diminishing at a point per unit volume.
If the fluid is incompressible, there can be no loss or gain in the volume element i.e. 𝑑𝑖𝑣 𝑣 = 0.
Observations:
(i) if 𝑣 represent the electric flux, then 𝑑𝑖𝑣 𝑣 is the amount of flux which diverges per unit volume in unit
time.
(ii) if 𝑣 represent the heat flux, then 𝑑𝑖𝑣 𝑣 is the rate at which the heat is issuing from a point per unit
volume.
(iii) If the flux entering any element of space is the same as that leaving it i.e.𝑑𝑖𝑣 𝑣 = 0 everywhere, then
such a vector point function is called Solenoidal.

Example 29: If 𝒓 = 𝒙𝒊 + 𝒚𝒋 + 𝒛𝒌 then show that 𝒅𝒊𝒗 𝒓 = 𝟑.

𝜕 𝜕 𝜕
Solution: 𝑑𝑖𝑣 𝑟 = ∇ ∙ 𝑟 = 𝑖 +𝑗 +𝑘 ∙ 𝑥𝑖 + 𝑦𝑗 + 𝑧𝑘
𝜕𝑥 𝜕𝑦 𝜕𝑧

𝜕 𝜕 𝜕
= 𝑥 + 𝑦 + 𝑧 =1+1+1=3
𝜕𝑥 𝜕𝑦 𝜕𝑧

Example 30: Evaluate 𝒅𝒊𝒗 𝒇 where 𝒇 = 𝟐𝒙𝟐 𝒛 𝒊 − 𝒙𝒚𝟐 𝒛 𝒋 + 𝟑𝒚𝟐 𝒙 𝒌 at (1, 1, 1).

𝜕∅ 𝜕∅ 𝜕∅
Solution: 𝑑𝑖𝑣 𝑓 = 𝑖 +𝑗 +𝑘 ∙ 2𝑥 2 𝑧 𝑖 − 𝑥𝑦 2 𝑧 𝑗 + 3𝑦 2 𝑥 𝑘
𝜕𝑥 𝜕𝑦 𝜕𝑧

25
𝜕 𝜕 𝜕
= 2𝑥 2 𝑧 + −𝑥𝑦 2 𝑧 + 3𝑦 2 𝑥
𝜕𝑥 𝜕𝑦 𝜕𝑧

= 4𝑥𝑧 − 2𝑥𝑦𝑧 + 0

At (1, 1, 1), 𝑑𝑖𝑣 𝑓 = 4 1 1 − 2 1 1 1 = 2

Example 31: Determine the constant a so that the vector 𝒇 = 𝒙 + 𝟑𝒚 𝒊 + 𝒚 − 𝟐𝒛 𝒋 + (𝒙 + 𝒂𝒛) 𝒌


is solenoidal.

Solution: Given that the vector 𝑓 is solenoidal, so 𝑑𝑖𝑣 𝑓 = 0

𝜕 𝜕 𝜕
𝑖 +𝑗 +𝑘 ∙ 𝑥 + 3𝑦 𝑖 + 𝑦 − 2𝑧 𝑗 + (𝑥 + 𝑎𝑧) 𝑘 = 0
𝜕𝑥 𝜕𝑦 𝜕𝑧
𝜕 𝜕 𝜕
𝑥 + 3𝑦 + 𝑦 − 2𝑧 + 𝑥 + 𝑎𝑧 = 0
𝜕𝑥 𝜕𝑦 𝜕𝑧

1 + 1 + 𝑎 = 0 => 𝑎 = −2

2. Curl: Let 𝑓 (𝑥, 𝑦, 𝑧) = 𝑓1 (𝑥, 𝑦, 𝑧)𝑖 + 𝑓2 (𝑥, 𝑦, 𝑧)𝑗 + 𝑓3 (𝑥, 𝑦, 𝑧)𝑘 be a continuously differentiable vector
point function. Curl of 𝑓 (𝑥, 𝑦, 𝑧) of is a vector which is denoted by ∇ × 𝑓 and is defined as

𝜕 𝜕 𝜕 𝜕𝑓 𝜕𝑓 𝜕𝑓
∇×𝑓 = 𝑖 +𝑗 +𝑘 ×𝑓 =𝑖× +𝑗 × +𝑘 ×
𝜕𝑥 𝜕𝑦 𝜕𝑧 𝜕𝑥 𝜕𝑦 𝜕𝑧

Also in component form, curl of 𝑓(𝑥, 𝑦, 𝑧) is

𝜕 𝜕 𝜕
∇×𝑓 = 𝑖 +𝑗 +𝑘 × 𝑓1 𝑖 + 𝑓2 𝑗 + 𝑓3 𝑘
𝜕𝑥 𝜕𝑦 𝜕𝑧

𝑖 𝑗 𝑘
𝜕 𝜕 𝜕 𝜕𝑓3 𝜕𝑓2 𝜕𝑓1 𝜕𝑓3 𝜕𝑓2 𝜕𝑓1
= =𝑖 − +𝑗 − +𝑘 −
𝜕𝑥 𝜕𝑦 𝜕𝑧 𝜕𝑦 𝜕𝑧 𝜕𝑧 𝜕𝑥 𝜕𝑥 𝜕𝑦
𝑓1 𝑓2 𝑓3

Physical Interpretation of Curl

Consider the motion of a rigid body rotating with


angular velocity 𝜔 about an axis OA, where O is a
fixed point in the body. Let 𝑟 be the position vector of
any point P of the body. The point P describing a circle
whose center is M and radius is PM = 𝑟 sin 𝜃 where 𝜃 is
the angle between 𝜔 and 𝑟 , then the velocity of P is
𝜔𝑟 sin 𝜃. This velocity is normal to the plane POM i.e.
normal to the plane of 𝜔 and 𝑟.
So, if 𝑣 is the linear velocity of P, then 𝑣 =
𝜔 𝑟 sin 𝜃 𝑛 = 𝜔 × 𝑟
26
(𝑛 𝑏𝑒𝑖𝑛𝑔 𝑛𝑜𝑟𝑚𝑎𝑙 𝑡𝑜 𝜔 𝑎𝑛𝑑 𝑟)

Now, if 𝜔 = 𝜔1 𝑖 + 𝜔2 𝑗 + 𝜔3 𝑘 and 𝑟 = 𝑥 𝑖 + 𝑦 𝑗 + 𝑧 𝑘 , then

𝑖 𝑗 𝑘
𝐶𝑢𝑟𝑙 𝑣 = 𝜔 × 𝑟 = 𝜔1 𝜔2 𝜔3 = 𝑖 𝜔2 𝑧 − 𝜔3 𝑦 + 𝑗 𝜔3 𝑥 − 𝜔1 𝑧 + 𝑘 𝜔1 𝑦 − 𝜔2 𝑥
𝑥 𝑦 𝑧

And

𝑖 𝑗 𝑘
𝜕 𝜕 𝜕
𝐶𝑢𝑟𝑙 𝑣 =
𝜕𝑥 𝜕𝑦 𝜕𝑧
𝜔2 𝑧 − 𝜔3 𝑦 𝜔3 𝑥 − 𝜔1 𝑧 𝜔1 𝑦 − 𝜔2 𝑥

= 𝑖 𝜔1 + 𝜔1 + 𝑗 𝜔2 + 𝜔2 + 𝑘 𝜔3 + 𝜔3
= 2𝜔1 𝑖 + 2𝜔2 𝑗 + 2𝜔3 𝑘 = 2 𝜔

1
Hence 𝜔 = 2 𝐶𝑢𝑟𝑙 𝑣

Thus the angular velocity of rotation at any point is equal to half the curl of the velocity vector.
Observations:
(i) The curl of a vector point function gives the measure of the angular velocity at a point.
(ii) If the curl of a vector point function becomes zero i.e. ∇ × 𝑓 = 0, then 𝑓 is called an irrotational
vector.

Example 32: If 𝒓 = 𝒙𝒊 + 𝒚𝒋 + 𝒛𝒌 then show that 𝒄𝒖𝒓𝒍 𝒓 = 𝟎.

𝑖 𝑗 𝑘
𝜕 𝜕 𝜕 𝜕𝑧 𝜕𝑦 𝜕𝑥 𝜕𝑧 𝜕𝑦 𝜕𝑥
Solution: 𝑐𝑢𝑟𝑙 𝑟 = ∇ × 𝑟 = =𝑖 − +𝑗 − +𝑘 −
𝜕𝑥 𝜕𝑦 𝜕𝑧 𝜕𝑦 𝜕𝑧 𝜕𝑧 𝜕𝑥 𝜕𝑥 𝜕𝑦
𝑥 𝑦 𝑧

= 𝑖 0 − 0 + 𝑗 0 − 0 + 𝑘 0 − 0 = 0.

Example 33: Find a so that the vector 𝒇 = 𝒂𝒙𝒚 − 𝒛𝟑 𝒊 + 𝒂 − 𝟐 𝒙𝟐 𝒋 + 𝟏 − 𝒂 𝒙𝒛𝟐 𝒌 is


irrotational.

Solution: Given that 𝑓 is irrotational, therefore 𝑐𝑢𝑟𝑙 𝑓 = 0 … (1)

𝑖 𝑗 𝑘
𝜕 𝜕 𝜕
But 𝑐𝑢𝑟𝑙 𝑓 =
𝜕𝑥 𝜕𝑦 𝜕𝑧
𝑎𝑥𝑦 − 𝑧 3 𝑎 − 2 𝑥2 1 − 𝑎 𝑥𝑧 2

= 𝑖 0 − 0 + 𝑗 −3𝑧 2 − 1 − 𝑎 𝑧 2 + 𝑘 𝑎 − 2 2𝑥 − 𝑎𝑥

= 0 𝑖 + −4 + 𝑎 𝑧 2 𝑗 + −4 + 𝑎 𝑥 𝑘

27
Using (1), 0 𝑖 + −4 + 𝑎 𝑧 2 𝑗 + −4 + 𝑎 𝑥 𝑘 = 0 𝑖 + 0 𝑗 + 0 𝑘

Comparing the corresponding components both sides,

−4 + 𝑎 = 0 => 𝑎 = 4.

Example 34: If 𝒇 = 𝒙𝒚𝟐 𝒊 + 𝟐𝒙𝟐 𝒚𝒛 𝒋 − 𝟑𝒚𝒛𝟐 𝒌, find the 𝒄𝒖𝒓𝒍 𝒇 at the point (1, -1, 1).

𝑖 𝑗 𝑘
𝜕 𝜕 𝜕
Solution: 𝑐𝑢𝑟𝑙 𝑓 =
𝜕𝑥 𝜕𝑦 𝜕𝑧
2 2
𝑥𝑦 2𝑥 𝑦𝑧 −3𝑦𝑧 2
= 𝑖 −3𝑧 2 − 2𝑥 2 𝑦 + 𝑗 0 − 0 + 𝑘 4𝑥𝑦𝑧 − 2𝑥𝑦
At (1, -1, 1),
𝑐𝑢𝑟𝑙 𝑓 = 𝑖 −3(1)2 − 2 1 2 (−1) + 𝑗 0 − 0 + 𝑘 4 1 −1 (1) − 2(1)(−1)
= −𝑖 − 2 𝑘.

17.7 DEL APPLIED TO THE PRODUCT OF POINT FUNCTIONS

Let 𝜙, 𝜓 are two scalar point functions and 𝑓 , 𝑔 are two vector point functions, then
1. ∇ ϕ ψ = ϕ ∇ψ + ψ ∇ϕ
2. ∇ ∙ ϕ 𝑓 = ∇ϕ ∙ 𝑓 + ϕ ∇ ∙ 𝑓

3. ∇ × ϕ 𝑓 = ∇ϕ × 𝑓 + ϕ ∇ × 𝑓

4. ∇ 𝑓 ∙ 𝑔 = 𝑓 ∙ ∇ 𝑔 + 𝑔 ∙ ∇ 𝑓 + 𝑓 × ∇ × 𝑔 + 𝑔 × ∇ × 𝑓 [KUK 2007]

5. ∇ ∙ 𝑓 × 𝑔 = 𝑔 ∙ ∇ × 𝑓 − 𝑓 ∙ ∇ × 𝑔

6. ∇ × 𝑓 × 𝑔 = 𝑓 ∇ ∙ 𝑔 − 𝑔 ∇ ∙ 𝑓 + 𝑔 ∙ ∇ 𝑓 − 𝑓 ∙ ∇ 𝑔 [KUK 2011]
𝜕 𝜕ψ 𝜕ϕ 𝜕ψ 𝜕ϕ
Proof 1: Consider ∇ ϕ ψ = 𝑖 ϕψ = 𝑖 ϕ 𝜕𝑥 + ψ 𝜕𝑥 = 𝑖 ϕ 𝜕𝑥 + 𝑖 ψ 𝜕𝑥
𝜕𝑥
𝜕ψ 𝜕ϕ
=ϕ 𝑖 +ψ 𝑖 = ϕ ∇ψ + ψ ∇ϕ
𝜕𝑥 𝜕𝑥

𝜕 𝜕ϕ 𝜕𝑓 𝜕ϕ 𝜕𝑓
Proof 2: Consider ∇ ∙ ϕ 𝑓 = 𝑖∙ 𝜕𝑥
ϕ𝑓 = 𝑖 ∙ 𝜕𝑥
𝑓 + ϕ 𝜕𝑥 = 𝑖 ∙ 𝜕𝑥
𝑓 + 𝑖 ∙ ϕ 𝜕𝑥

𝜕ϕ 𝜕𝑓
= 𝑖 𝜕𝑥 ∙ 𝑓 + ϕ 𝑖 ∙ 𝜕𝑥 = ∇ϕ ∙ 𝑓 + ϕ ∇ ∙ 𝑓

𝜕 𝜕ϕ 𝜕𝑓 𝜕ϕ 𝜕𝑓
Proof 3: Consider ∇ × ϕ 𝑓 = 𝑖× 𝜕𝑥
ϕ𝑓 = 𝑖 × 𝜕𝑥
𝑓 + ϕ 𝜕𝑥 = 𝑖 × 𝜕𝑥
𝑓 + 𝑖 × ϕ 𝜕𝑥

𝜕ϕ 𝜕𝑓
= 𝑖 𝜕𝑥 × 𝑓 + ϕ 𝑖 × 𝜕𝑥 = ∇ϕ × 𝑓 + ϕ ∇ × 𝑓
𝜕 𝜕𝑓 𝜕𝑔 𝜕𝑓 𝜕𝑔
Proof 4: Consider ∇ 𝑓 ∙ 𝑔 = 𝑖 𝜕𝑥
𝑓∙𝑔 = 𝑖 𝜕𝑥
∙ 𝑔 + 𝑓 ∙ 𝜕𝑥 = 𝑖 𝜕𝑥
∙𝑔+ 𝑖 𝑓 ∙ 𝜕𝑥

… (1)
28
𝜕𝑓 𝜕𝑓 𝜕𝑓
But, 𝑔 × 𝑖 × 𝜕𝑥 = 𝑔 ∙ 𝜕𝑥 𝑖 − 𝑔 ∙ 𝑖 𝜕𝑥

𝜕𝑓 𝜕𝑓 𝜕𝑓
or 𝑔 ∙ 𝜕𝑥 𝑖 = 𝑔 × 𝑖 × 𝜕𝑥 + 𝑔 ∙ 𝑖 𝜕𝑥

𝜕𝑓 𝜕𝑓 𝜕𝑓
So, 𝑔 ∙ 𝜕𝑥 𝑖 = 𝑔 × 𝑖 × 𝜕𝑥 + 𝑔∙𝑖 = 𝑔× ∇×𝑓 + 𝑔∙∇ 𝑓 … (2)
𝜕𝑥

Interchanging 𝑓 and 𝑔 in (2)


𝜕𝑔 𝜕𝑔 𝜕𝑔
𝑓 ∙ 𝜕𝑥 𝑖 = 𝑓 × 𝑖 × 𝜕𝑥 + 𝑓∙𝑖 = 𝑓× ∇×𝑔 + 𝑓∙∇ 𝑔 … (3)
𝜕𝑥

Using (2) and (3) in (1), we get


∇ 𝑓∙𝑔 = 𝑓∙∇ 𝑔+ 𝑔∙∇ 𝑓+𝑓× ∇×𝑔 +𝑔× ∇×𝑓
𝜕 𝜕𝑓 𝜕𝑔
Proof 5: Consider ∇ ∙ 𝑓 × 𝑔 = 𝑖∙ 𝑓×𝑔 = 𝑖 ∙ ×𝑔+𝑓×
𝜕𝑥 𝜕𝑥 𝜕𝑥

𝜕𝑓 𝜕𝑔 𝜕𝑓 𝜕𝑔
= 𝑖 ∙ ×𝑔 + 𝑖 ∙ 𝑓 × 𝜕𝑥 = 𝑔 ∙ 𝑖 × 𝜕𝑥 − 𝑓 ∙ 𝑖 × 𝜕𝑥
𝜕𝑥

=𝑔∙ ∇×𝑓 −𝑓∙ ∇×𝑔

[using 𝑎 ∙ 𝑏 × 𝑐 = 𝑐 ∙ 𝑎 × 𝑏 = −𝑏 ∙ 𝑎 × 𝑐 ]
𝜕 𝜕𝑓 𝜕𝑔
Proof 6: Consider ∇ × 𝑓 × 𝑔 = 𝑖× 𝑓×𝑔 = 𝑖 × × 𝑔 + 𝑓 × 𝜕𝑥
𝜕𝑥 𝜕𝑥
𝜕𝑓 𝜕𝑔
= 𝑖× ×𝑔 + 𝑖 × 𝑓 × 𝜕𝑥
𝜕𝑥

𝜕𝑓 𝜕𝑓 𝜕𝑔 𝜕𝑔
= 𝑖∙𝑔 − 𝑖∙ 𝑔 + 𝑖∙ 𝑓− 𝑖∙𝑓
𝜕𝑥 𝜕𝑥 𝜕𝑥 𝜕𝑥

[using 𝑎 × 𝑏 × 𝑐 = 𝑎 ∙ 𝑐 𝑏 − 𝑎 ∙ 𝑏 𝑐 ]
𝜕𝑓 𝜕𝑓 𝜕𝑔 𝜕𝑔
= 𝑔∙𝑖 −𝑔 𝑖 ∙ 𝜕𝑥 + 𝑓 𝑖 ∙ 𝜕𝑥 − 𝑓∙𝑖
𝜕𝑥 𝜕𝑥

= 𝑔∙∇ 𝑓−𝑔 ∇∙𝑓 +𝑓 ∇∙𝑔 − 𝑓∙∇ 𝑔

=𝑓 ∇∙𝑔 −𝑔 ∇∙𝑓 + 𝑔∙∇ 𝑓− 𝑓∙∇ 𝑔

17.8 DEL APPLIED TWICE TO POINT FUNCTIONS

Let ∅ be a scalar point function and 𝑓 be a vector point function, then ∇∅ and ∇ × 𝑓 being the vector
point functions, we can find their divergence and curl; whereas ∇ ∙ 𝑓 being the scalar point function,
we can find its gradient only. Thus we have following formulae:
𝜕2∅ 𝜕2∅ 𝜕2∅
1. 𝑑𝑖𝑣 𝑔𝑟𝑎𝑑 ∅ = ∇ ∙ ∇∅ = ∇2 ∅ = 𝜕𝑥2
+ 𝜕𝑦 2
+ 𝜕𝑧 2

2. 𝑐𝑢𝑟𝑙 𝑔𝑟𝑎𝑑 ∅ = ∇ × ∇∅ = 0
3. 𝑑𝑖𝑣 𝑐𝑢𝑟𝑙 𝑓 = ∇ ∙ ∇ × 𝑓 = 0

4. 𝑐𝑢𝑟𝑙 𝑐𝑢𝑟𝑙 𝑓 = ∇ × ∇ × 𝑓 = ∇ ∇ ∙ 𝑓 − ∇2 𝑓 = 𝑔𝑟𝑎𝑑 𝑑𝑖𝑣 𝑓 − ∇2 𝑓 [KUK 2006]

5. 𝑔𝑟𝑎𝑑 𝑑𝑖𝑣 𝑓 = ∇ ∇ ∙ 𝑓 = 𝑐𝑢𝑟𝑙 𝑐𝑢𝑟𝑙 𝑓 + ∇2 𝑓 = ∇ × ∇ × 𝑓 + ∇2 𝑓

29
𝜕∅ 𝜕∅ 𝜕∅
Proof 1: ∇2 ∅ = ∇ ∙ ∇∅ =∇∙ 𝑖 +𝑗 +𝑘
𝜕𝑥 𝜕𝑦 𝜕𝑧

𝜕 𝜕∅ 𝜕 𝜕∅ 𝜕 𝜕∅ 𝜕2∅ 𝜕2∅ 𝜕2∅


= + + = + +
𝜕𝑥 𝜕𝑥 𝜕𝑦 𝜕𝑦 𝜕𝑧 𝜕𝑧 𝜕𝑥 2 𝜕𝑦 2 𝜕𝑧 2

Here ∇2 is called the Laplacian Operator and ∇2 ∅ = 0 is called the Laplace’s Equation.

𝜕∅ 𝜕∅ 𝜕∅
Proof 2: 𝑐𝑢𝑟𝑙 𝑔𝑟𝑎𝑑 ∅ = ∇ × ∇∅ = ∇ × 𝑖 +𝑗 +𝑘
𝜕𝑥 𝜕𝑦 𝜕𝑧

𝑖 𝑗 𝑘
𝜕 𝜕 𝜕
𝜕2∅ 𝜕2∅
= 𝜕𝑥 𝜕𝑦 𝜕𝑧 = 𝑖 − =0
𝜕𝑦𝜕𝑧 𝜕𝑧𝜕𝑦
𝜕∅ 𝜕∅ 𝜕∅
𝜕𝑥 𝜕𝑦 𝜕𝑧

𝜕 𝜕𝑓 𝜕𝑓 𝜕𝑓
Proof 3: 𝑑𝑖𝑣 𝑐𝑢𝑟𝑙 𝑓 = ∇ ∙ ∇ × 𝑓 = 𝑖 𝜕𝑥 ∙ 𝑖 × 𝜕𝑥 + 𝑗 × 𝜕𝑦 + 𝑘 × 𝜕𝑧

𝜕2𝑓 𝜕2𝑓 𝜕2𝑓


= 𝑖 ∙ 𝑖 × 𝜕𝑥 2 + 𝑗 × 𝜕𝑥𝜕𝑦 + 𝑘 × 𝜕𝑥𝜕𝑧

𝜕2𝑓 𝜕2𝑓 𝜕2𝑓


= 𝑖 × 𝑖 ∙ 𝜕𝑥 2 + 𝑖 × 𝑗 ∙ 𝜕𝑥𝜕𝑦 + 𝑖 × 𝑘 ∙ 𝜕𝑥𝜕𝑧

𝜕2𝑓 𝜕2𝑓
= 𝑘 ∙ 𝜕𝑥𝜕𝑦 − 𝑗 ∙ 𝜕𝑥𝜕𝑧 = 0

𝜕 𝜕𝑓 𝜕𝑓 𝜕𝑓
Proof 4: 𝑐𝑢𝑟𝑙 𝑐𝑢𝑟𝑙 𝑓 = ∇ × ∇ × 𝑓 = 𝑖 𝜕𝑥 × 𝑖 × 𝜕𝑥 + 𝑗 × 𝜕𝑦 + 𝑘 × 𝜕𝑧

𝜕2𝑓 𝜕2𝑓 𝜕2𝑓


= 𝑖 × 𝑖 × 𝜕𝑥 2 + 𝑗 × 𝜕𝑥𝜕𝑦 + 𝑘 × 𝜕𝑥𝜕𝑧

𝜕2𝑓 𝜕2𝑓 𝜕2𝑓


= 𝑖 × 𝑖 × 𝜕𝑥 2 + 𝑖 × 𝑗 × 𝜕𝑥𝜕𝑦 + 𝑖 × 𝑘 × 𝜕𝑥𝜕𝑧

𝜕2𝑓 𝜕2𝑓 𝜕2𝑓 𝜕2𝑓 𝜕2𝑓 𝜕2𝑓


= 𝑖 ∙ 𝜕𝑥 2 𝑖 − 𝑖 ∙ 𝑖 + 𝑖 ∙ 𝜕𝑥𝜕𝑦 𝑗 − 𝑖 ∙ 𝑗 + 𝑖 ∙ 𝜕𝑥𝜕𝑧 𝑘 − 𝑖 ∙ 𝑘
𝜕𝑥 2 𝜕𝑥𝜕𝑦 𝜕𝑥𝜕𝑧

𝜕2𝑓 𝜕2𝑓 𝜕2𝑓 𝜕2𝑓


= 𝑖 ∙ 𝜕𝑥 2 𝑖 + 𝑖 ∙ 𝜕𝑥𝜕𝑦 𝑗 + 𝑖 ∙ 𝜕𝑥𝜕𝑧 𝑘 − 𝜕𝑥 2

𝜕 𝜕𝑓 𝜕𝑓 𝜕𝑓 𝜕2𝑓
= 𝑖 𝑖 ∙ 𝜕𝑥 + 𝑗 ∙ 𝜕𝑦 + 𝑘 ∙ 𝜕𝑧 − = ∇ ∇ ∙ 𝑓 − ∇2 𝑓
𝜕𝑥 𝜕𝑥 2

Proof 5: To get this formula we are to re-arrange the terms in last proof.

Example 35: Show that 𝛁 𝟐 𝒓𝒏 = 𝒏(𝒏 + 𝟏)𝒓𝒏−𝟐 .


Solution: We know that 𝑟 2 = 𝑥 2 + 𝑦 2 + 𝑧 2
𝜕𝑟 𝑥
On differentiation w. r. t. x, =
𝜕𝑥 𝑟
𝜕𝑟 𝑦 𝜕𝑟 𝑧
Similarly = and =
𝜕𝑦 𝑟 𝜕𝑧 𝑟
𝜕2 𝜕2 𝜕2 𝜕 𝜕𝑟 𝑛 𝜕 𝜕𝑟 𝑛 𝜕 𝜕𝑟 𝑛
Now ∇2 𝑟 𝑛 = 2
+ 2
+ 𝑟𝑛 = + + … (1)
𝜕𝑥 𝜕𝑦 𝜕𝑧 2 𝜕𝑥 𝜕𝑥 𝜕𝑦 𝜕𝑦 𝜕𝑧 𝜕𝑧

𝜕 𝜕𝑟 𝑛 𝜕 𝜕𝑟 𝜕 𝑥 𝜕
And = 𝑛 𝑟 𝑛 −1 = 𝑛 𝑟 𝑛−1 =𝑛 𝑟 𝑛 −2 𝑥
𝜕𝑥 𝜕𝑥 𝜕𝑥 𝜕𝑥 𝜕𝑥 𝑟 𝜕𝑥
30
𝜕𝑟
= 𝑛 𝑟 𝑛−2 + 𝑥 (𝑛 − 2)𝑟 𝑛−3 𝜕𝑥 = 𝑛 𝑟 𝑛−2 + 𝑥 2 (𝑛 − 2)𝑟 𝑛−4
𝜕 𝜕𝑟 𝑛
Similarly = 𝑛 𝑟 𝑛−2 + 𝑦 2 (𝑛 − 2)𝑟 𝑛−4
𝜕𝑦 𝜕𝑦
𝜕 𝜕𝑟 𝑛
= 𝑛 𝑟𝑛−2 + 𝑧2 (𝑛 − 2)𝑟𝑛−4
𝜕𝑧 𝜕𝑧

Using all these values in (1),


∇2 𝑟 𝑛 = 𝑛 𝑟 𝑛−2 + 𝑥 2 (𝑛 − 2)𝑟 𝑛−4 ) + 𝑛 𝑟 𝑛−2 + (𝑛 − 2)𝑟 𝑛−4 + 𝑛 𝑟 𝑛−2 + 𝑧 2 (𝑛 − 2)𝑟 𝑛−4
= 3𝑛 𝑟 𝑛−2 + 𝑛 𝑛 − 2 𝑟 𝑛−4 𝑥 2 + 𝑦 2 + 𝑧 2
= 3𝑛 𝑟 𝑛−2 + 𝑛 𝑛 − 2 𝑟 𝑛−2 = 𝑛 𝑛 + 1 𝑟 𝑛−2
𝟐
Example 36: Show that *(i) 𝛁 𝟐 𝒇 𝒓 = 𝒇 ′′
𝒓 +
𝒓
𝒇 ′ (𝒓) (ii) 𝛁 ∙ 𝝓 𝛁𝝍 − 𝝍 𝛁𝝓 = 𝝓 𝛁 𝟐 𝝍 − 𝝍 𝛁 𝟐 𝝓

*[KUK 2008]

𝜕2 𝜕2 𝜕2 𝜕 𝜕 𝜕 𝜕 𝜕 𝜕
Solution: (i) ∇2 𝑓 𝑟 = + 𝜕𝑦 2 + 𝜕𝑧 2 𝑓 𝑟 = 𝜕𝑥 𝑓 𝑟 + 𝜕𝑦 𝑓 𝑟 + 𝜕𝑧 𝑓 𝑟 …(1)
𝜕𝑥 2 𝜕𝑥 𝜕𝑦 𝜕𝑧

𝜕 𝜕 𝜕 𝜕𝑟 𝜕 𝑥 𝜕 𝑥 𝑥 𝜕𝑟
And 𝑓 𝑟 = 𝜕𝑥 𝑓 ′(𝑟) = 𝜕𝑥 𝑓 ′(𝑟) = 𝑓 ′ (𝑟) 𝜕𝑥 + 𝑟 𝑓 ′′(𝑟) 𝜕𝑥
𝜕𝑥 𝜕𝑥 𝜕𝑥 𝑟 𝑟

1 𝑥 𝜕𝑟 𝑥 𝜕𝑟
= 𝑓 ′ (𝑟) − + 𝑓 ′′(𝑟)
𝑟 𝑟 2 𝜕𝑥 𝑟 𝜕𝑥
𝑓′ 𝑟 𝑓′ 𝑟 𝑓 ′′ (𝑟)
= − 𝑥2 + 𝑥2
𝑟 𝑟3 𝑟2

𝜕 𝜕 𝑓′ 𝑟 𝑓′ 𝑟 𝑓 ′′ (𝑟)
Similarly 𝑓 𝑟 = − 𝑦2 + 𝑦2
𝜕𝑦 𝜕𝑦 𝑟 𝑟3 𝑟2

𝜕 𝜕 𝑓′ 𝑟 𝑓′ 𝑟 𝑓 ′′ (𝑟)
𝑓 𝑟 = − 𝑧2 + 𝑧2
𝜕𝑧 𝜕𝑧 𝑟 𝑟3 𝑟2

Using all these values in (1)


′′ ′′
2 𝑓′ 𝑟 𝑓′ 𝑟 2𝑓 𝑟 𝑓′ 𝑟 𝑓′ 𝑟 2𝑓 𝑟
∇ 𝑓 𝑟 = − 𝑥2 3 +𝑥 + − 𝑦2 3 +𝑦
𝑟 𝑟 𝑟2 𝑟 𝑟 𝑟2

𝑓′ 𝑟 𝑓′ 𝑟 𝑓 ′′ (𝑟)
+ − 𝑧2 + 𝑧2
𝑟 𝑟3 𝑟2
3 𝑓′ 𝑟 𝑓′ 𝑟 𝑓 ′′ 𝑟
= − 𝑥2 + 𝑦2 + 𝑧2 + 𝑥2 + 𝑦2 + 𝑧2
𝑟 𝑟3 𝑟2
3 𝑓 ′ (𝑟) 𝑓 ′ (𝑟)
= − + 𝑓 ′′ (𝑟)
𝑟 𝑟
2 𝑓 ′ (𝑟)
= + 𝑓 ′′ (𝑟)
𝑟
2
Hence ∇2 𝑓 𝑟 = 𝑓 ′′ 𝑟 + 𝑟 𝑓 ′ (𝑟) .

(ii) Consider ∇ ∙ 𝜙 ∇𝜓 − 𝜓 ∇𝜙 = ∇ ∙ 𝜙 ∇𝜓 − ∇ ∙ 𝜓 ∇𝜙
= ∇𝜙 ∙ ∇𝜓 + 𝜙 ∇ ∙ ∇𝜓 − ∇𝜓 ∙ ∇𝜙 + 𝜓 ∇ ∙ ∇𝜙

= ∇𝜙 ∙ ∇𝜓 + 𝜙 ∇2 𝜓 − ∇𝜙 ∙ ∇𝜓 − 𝜓 ∇2 𝜙

= 𝜙 ∇2 𝜓 − 𝜓 ∇2 𝜙
31
Hence ∇ ∙ 𝜙 ∇𝜓 − 𝜓 ∇𝜙 = 𝜙 ∇2 𝜓 − 𝜓 ∇2 𝜙.

Example 37: Find the value of n for which the vector 𝒓𝒏 𝒓 is solenoidal, where
𝒓 = 𝒙𝒊 + 𝒚𝒋 + 𝒛𝒌.

Solution: Consider 𝑑𝑖𝑣 𝑟 𝑛 𝑟 = ∇ ∙ 𝑟 𝑛 𝑟 = ∇𝑟 𝑛 ∙ 𝑟 + 𝑟 𝑛 ∇ ∙ 𝑟 … (1)


𝜕 𝜕 𝜕 𝜕𝑟 𝜕𝑟 𝜕𝑟
But ∇𝑟 𝑛 = 𝑖 +𝑗 +𝑘 𝑟 𝑛 = 𝑛 𝑟 𝑛−1 𝑖 +𝑗 +𝑘
𝜕𝑥 𝜕𝑦 𝜕𝑧 𝜕𝑥 𝜕𝑦 𝜕𝑧
𝑥 𝑦 𝑧
= 𝑛 𝑟 𝑛−1 𝑖 + 𝑗 + 𝑘 = 𝑛 𝑟 𝑛−2 𝑟 … (2)
𝑟 𝑟 𝑟

And ∇∙𝑟 =3 … (3)


Therefore, 𝑑𝑖𝑣 𝑟 𝑛 𝑟 = 𝑛 𝑟 𝑛−2 𝑟 ∙ 𝑟 + 3 𝑟 𝑛 = 𝑛 𝑟 𝑛−2 𝑟 ∙ 𝑟 + 3𝑟 𝑛
= 𝑛 𝑟 𝑛−2 𝑟 2 + 3𝑟 𝑛 = (𝑛 + 3)𝑟 𝑛 … (4)
As given the vector 𝑟 𝑛 𝑟 is solenoidal, so 𝑑𝑖𝑣 𝑟 𝑛 𝑟 = 0
So using (4), 𝑛 + 3 𝑟𝑛 = 0 implies that 𝑛 = −3 (since 𝑟 ≠ 0)

Example 38: If 𝒂 and 𝒃 are irrotational, prove that 𝒂 × 𝒃 is solenoidal.

Solution: Given 𝑎 and 𝑏 are irrotational, so ∇ × 𝑎 = 0 = ∇ × 𝑏 … (1)

Consider 𝐷𝑖𝑣 𝑎 × 𝑏 = 𝑏 ∙ ∇ × 𝑎 − 𝑎 ∙ ∇ × 𝑏 = 𝑏 ∙ 0 − 𝑎 ∙ 0 = 0 [using (1)]

Thus 𝑎 × 𝑏 is solenoidal.
Example 39: Show that the vector field 𝒇 = 𝒛𝟐 + 𝟐𝒙 + 𝟑𝒚 𝒊 + 𝟑𝒙 + 𝟐𝒚 + 𝒛 𝒋 + (𝒚 + 𝟐𝒛𝒙)𝒌 is
irrotational but not solenoidal. Also obtain a scalar function ∅ such that 𝛁∅ = 𝒇 .

𝑖 𝑗 𝑘
𝜕 𝜕 𝜕
Solution: Consider 𝐶𝑢𝑟𝑙 𝑓 = ∇ × 𝑓 = 𝜕𝑥 𝜕𝑦 𝜕𝑧
𝑧 2 + 2𝑥 + 3𝑦 3𝑥 + 2𝑦 + 𝑧 𝑦 + 2𝑧𝑥
= 𝑖 1 − 1 − 𝑗 2𝑧 − 2𝑧 + 𝑘 3 − 3 = 0

So 𝑓 is irrotational vector field.


𝜕 𝜕 𝜕
Also consider 𝑑𝑖𝑣 𝑓 = ∇ ∙ 𝑓 = 𝑖 𝜕𝑥 + 𝑗 𝜕𝑦 + 𝑘 𝜕𝑧 ∙ 𝑓 = 2 + 2 + 2𝑥 = 2(𝑥 + 2) ≠ 0

So 𝑓 is not solenoidal vector field.


𝜕∅ 𝜕∅ 𝜕∅ 𝜕∅ 𝜕∅ 𝜕∅
Now 𝑑∅ = 𝜕𝑥 𝑑𝑥 + 𝜕𝑦 𝑑𝑦 + 𝜕𝑧 𝑑𝑧 = 𝑖 𝜕𝑥 + 𝑗 𝜕𝑦 + 𝑘 𝜕𝑧 ∙ 𝑑𝑥 𝑖 + 𝑑𝑦 𝑗 + 𝑑𝑧 𝑘

= 𝑔𝑟𝑎𝑑 ∅ ∙ 𝑑𝑟 = 𝑓 ∙ 𝑑𝑟 (as 𝑓 = 𝑔𝑟𝑎𝑑 ∅ )


= 𝑧 2 + 2𝑥 + 3𝑦 𝑖 + 3𝑥 + 2𝑦 + 𝑧 𝑗 + (𝑦 + 2𝑧𝑥)𝑘 ∙ 𝑑𝑥 𝑖 + 𝑑𝑦 𝑗 + 𝑑𝑧 𝑘

= 𝑧 2 + 2𝑥 + 3𝑦 𝑑𝑥 + 3𝑥 + 2𝑦 + 𝑧 𝑑𝑦 + (𝑦 + 2𝑧𝑥)𝑑𝑧
= 𝑧 2 𝑑𝑥 + 2𝑧𝑥 𝑑𝑧 + 3𝑦 𝑑𝑥 + 3𝑥 𝑑𝑦 + 𝑧 𝑑𝑦 + 𝑦 𝑑𝑧 + 2𝑥 𝑑𝑥 + 2𝑦 𝑑𝑦
= 𝑑 𝑥𝑧 2 + 3𝑑 𝑥𝑦 + 𝑑 𝑦𝑧 + 𝑑 𝑥 2 + 𝑑(𝑦 2 )
Integrating both sides, we get
32
∅ = 𝑥𝑧 2 + 3𝑥𝑦 + 𝑦𝑧 + 𝑥 2 + 𝑦 2 + 𝑐

Example 40: If 𝑽𝟏 𝐚𝐧𝐝 𝑽𝟐 be the vectors joining the fixed points 𝒙𝟏 , 𝒚𝟏 , 𝒛𝟏 and 𝒙𝟐 , 𝒚𝟐 , 𝒛𝟐
respectively to a variable point (𝒙, 𝒚, 𝒛), prove that *[KUK 2010]

(i) 𝒅𝒊𝒗 𝑽𝟏 × 𝑽𝟐 = 𝟎 *(ii) 𝒄𝒖𝒓𝒍 𝑽𝟏 × 𝑽𝟐 = 𝟐(𝑽𝟏 − 𝑽𝟐 ) (iii) 𝒈𝒓𝒂𝒅 𝑽𝟏 ∙ 𝑽𝟐 = 𝑽𝟏 + 𝑽𝟐.

Solution: Here, 𝑉1 = 𝑥 − 𝑥1 𝑖 + 𝑦 − 𝑦1 𝑗 + 𝑧 − 𝑧1 𝑘 and 𝑉2 = 𝑥 − 𝑥2 𝑖 + 𝑦 − 𝑦2 𝑗 + 𝑧 − 𝑧2 𝑘

𝑖 𝑗 𝑘
(i) 𝑉1 × 𝑉2 = 𝑥 − 𝑥1 𝑦 − 𝑦1 𝑧 − 𝑧1
𝑥 − 𝑥2 𝑦 − 𝑦2 𝑧 − 𝑧2

= 𝑦 − 𝑦1 𝑧 − 𝑧2 − 𝑦 − 𝑦2 𝑧 − 𝑧1 𝑖 + 𝑥 − 𝑥2 𝑧 − 𝑧1 − 𝑥 − 𝑥1 𝑧 − 𝑧2 𝑗
+ 𝑥 − 𝑥1 𝑦 − 𝑦2 − 𝑥 − 𝑥2 𝑦 − 𝑦1 𝑘

𝜕
So 𝑑𝑖𝑣 𝑉1 × 𝑉2 = 𝜕𝑥 𝑦 − 𝑦1 𝑧 − 𝑧2 − 𝑦 − 𝑦2 𝑧 − 𝑧1
𝜕
+ 𝑥 − 𝑥2 𝑧 − 𝑧1 − 𝑥 − 𝑥1 𝑧 − 𝑧2
𝜕𝑦
𝜕
+ 𝜕𝑧 𝑥 − 𝑥1 𝑦 − 𝑦2 − 𝑥 − 𝑥2 𝑦 − 𝑦1 =0

(ii) 𝑐𝑢𝑟𝑙 𝑉1 × 𝑉2 =

𝑖 𝑗 𝑘
𝜕 𝜕 𝜕
𝜕𝑥 𝜕𝑦 𝜕𝑧
𝑦 − 𝑦1 𝑧 − 𝑧2 − 𝑦 − 𝑦2 𝑧 − 𝑧1 𝑥 − 𝑥2 𝑧 − 𝑧1 − 𝑥 − 𝑥1 𝑧 − 𝑧2 𝑥 − 𝑥1 𝑦 − 𝑦2 − 𝑥 − 𝑥2 𝑦 − 𝑦1

= 𝑥 − 𝑥1 − 𝑥 − 𝑥2 − 𝑥 − 𝑥2 + 𝑥 − 𝑥1 𝑖
+[ 𝑦 − 𝑦1 − 𝑦 − 𝑦2 − 𝑦 − 𝑦2 + 𝑦 − 𝑦1 ]𝑗
+[ 𝑧 − 𝑧1 − 𝑧 − 𝑧2 − 𝑧 − 𝑧2 + 𝑧 − 𝑧1 ]𝑘
= 2 𝑥 − 𝑥1 𝑖 + (𝑦 − 𝑦1 )𝑗 + 𝑧 − 𝑧1 𝑘 − 2[ 𝑥 − 𝑥2 𝑖 + 𝑦 − 𝑦2 𝑗 + 𝑧 − 𝑧2 𝑘 ]

= 2(𝑉1 − 𝑉2 )

(iii) 𝑉1 ∙ 𝑉2 = 𝑥 − 𝑥1 𝑥 − 𝑥2 + 𝑦 − 𝑦1 𝑦 − 𝑦2 + 𝑧 − 𝑧1 𝑧 − 𝑧2

𝜕 𝜕 𝜕
So 𝑔𝑟𝑎𝑑 𝑉1 ∙ 𝑉2 = 𝑖 𝜕𝑥 𝑥 − 𝑥1 𝑥 − 𝑥2 + 𝑗 𝜕𝑦 𝑦 − 𝑦1 𝑦 − 𝑦2 + 𝑘 𝜕𝑧 𝑧 − 𝑧1 𝑧 − 𝑧2

= 𝑖 𝑥 − 𝑥1 + 𝑥 − 𝑥2 + 𝑗 𝑦 − 𝑦1 + 𝑦 − 𝑦2 + 𝑘 𝑧 − 𝑧1 + 𝑧 − 𝑧2

= 𝑥 − 𝑥1 𝑖 + 𝑦 − 𝑦1 𝑗 + 𝑧 − 𝑧1 𝑘 + 𝑥 − 𝑥2 𝑖 + 𝑦 − 𝑦2 𝑗 + 𝑧 − 𝑧2 𝑘

= 𝑉1 + 𝑉2

33
Example 41: If 𝒂 is a constant vector and 𝒓 = 𝒙 𝒊 + 𝒚 𝒋 + 𝒛 𝒌, prove that [KUK 2009]
(i) 𝒈𝒓𝒂𝒅 𝒂 ∙ 𝒓 = 𝒂 (ii) 𝒅𝒊𝒗 𝒂 × 𝒓 = 𝟎 (iii) ) 𝒄𝒖𝒓𝒍 𝒂 × 𝒓 = 𝟐𝒂 (iv) 𝒄𝒖𝒓𝒍 𝒂 ∙ 𝒓 𝒓 = 𝒂 × 𝒓

Solution: Let 𝑎 = 𝑎1 𝑖 + 𝑎2 𝑗 + 𝑎3 𝑘 is the constant vector.


(i) 𝑎 ∙ 𝑟 = 𝑎1 𝑖 + 𝑎2 𝑗 + 𝑎3 𝑘 ∙ 𝑥 𝑖 + 𝑦 𝑗 + 𝑧 𝑘 = 𝑎1 𝑥 + 𝑎2 𝑦 + 𝑎3 𝑧
𝜕 𝜕 𝜕
So 𝑔𝑟𝑎𝑑 𝑎 ∙ 𝑟 = 𝑖 𝜕𝑥 𝑎1 𝑥 + 𝑎2 𝑦 + 𝑎3 𝑧 + 𝑗 𝜕𝑦 𝑎1 𝑥 + 𝑎2 𝑦 + 𝑎3 𝑧 + 𝑘 𝜕𝑧 𝑎1 𝑥 + 𝑎2 𝑦 + 𝑎3 𝑧

= 𝑎1 𝑖 + 𝑎2 𝑗 + 𝑎3 𝑘 = 𝑎
𝑖 𝑗 𝑘
(ii) 𝑎 × 𝑟 = 𝑎1 𝑎2 𝑎3 = 𝑖 𝑎2 𝑧 − 𝑎3 𝑦 + 𝑗 𝑎3 𝑥 − 𝑎1 𝑧 + 𝑘 𝑎1 𝑦 − 𝑎2 𝑥
𝑥 𝑦 𝑧
𝜕 𝜕 𝜕
𝑑𝑖𝑣 𝑎 × 𝑟 = 𝑎2 𝑧 − 𝑎3 𝑦 + 𝑎3 𝑥 − 𝑎1 𝑧 + 𝑎1 𝑦 − 𝑎2 𝑥 = 0
𝜕𝑥 𝜕𝑦 𝜕𝑧

𝑖 𝑗 𝑘
𝜕 𝜕 𝜕
(iii) 𝑐𝑢𝑟𝑙 𝑎 × 𝑟 =
𝜕𝑥 𝜕𝑦 𝜕𝑧
𝑎2 𝑧 − 𝑎3 𝑦 𝑎3 𝑥 − 𝑎1 𝑧 𝑎1 𝑦 − 𝑎2 𝑥
= 𝑖 𝑎1 + 𝑎1 + 𝑗 𝑎2 + 𝑎2 + 𝑘 𝑎3 + 𝑎3 = 2 𝑎1 𝑖 + 𝑎2 𝑗 + 𝑎3 𝑘 = 2𝑎
(iv) 𝑎 ∙ 𝑟 𝑟 = 𝑎1 𝑥 + 𝑎2 𝑦 + 𝑎3 𝑧 𝑥𝑖 + 𝑎1 𝑥 + 𝑎2 𝑦 + 𝑎3 𝑧 𝑦𝑗 + 𝑎1 𝑥 + 𝑎2 𝑦 + 𝑎3 𝑧 𝑧 𝑘
𝑖 𝑗 𝑘
𝜕 𝜕 𝜕
𝐶𝑢𝑟𝑙 𝑎∙𝑟 𝑟 =
𝜕𝑥 𝜕𝑦 𝜕𝑧
𝑎1 𝑥 + 𝑎2 𝑦 + 𝑎3 𝑧 𝑥 𝑎1 𝑥 + 𝑎2 𝑦 + 𝑎3 𝑧 𝑦 𝑎1 𝑥 + 𝑎2 𝑦 + 𝑎3 𝑧 𝑧
= 𝑖 𝑎2 𝑧 − 𝑎3 𝑦 + 𝑗 𝑎3 𝑥 − 𝑎1 𝑧 + 𝑘 𝑎1 𝑦 − 𝑎2 𝑥 = 𝑎 × 𝑟 [using part (ii)]

Example 42: Find 𝒇 × 𝛁 × 𝒈 at the point (1, -1, 2),


if 𝒇 = 𝒙𝒛𝟐 𝒊 + 𝟐𝒚 𝒋 − 𝟑𝒙𝒛 𝒌, 𝒈 = 𝟑𝒙𝒛 𝒊 + 𝟐𝒚𝒛 𝒋 − 𝒛𝟐 𝒌.

𝑖 𝑗 𝑘
𝜕 𝜕 𝜕
Solution: ∇ × 𝑔 = 𝜕𝑥 𝜕𝑦 𝜕𝑧
= 𝑖 0 − 2𝑦 + 𝑗 3𝑥 − 0 + 𝑘 0 − 0 = −2𝑦 𝑖 + 3𝑥 𝑗 + 0 𝑘
3𝑥𝑧 2𝑦𝑧 −𝑧 2
𝑖 𝑗 𝑘
Now 𝑓 × ∇ × 𝑔 = 𝑥𝑧 2 2𝑦 −3𝑥𝑧 = 9𝑥 2 𝑧 𝑖 + 6𝑥𝑦𝑧 𝑗 + 3𝑥 2 𝑧 2 + 4𝑦 2 𝑘
−2𝑦 3𝑥 0
At (1, -1, 2), 𝑓 × ∇ × 𝑔 = 9 1 2 (2) 𝑖 + 6 1 −1 (2) 𝑗 + 3(1)2 (2)2 + 4(−1)2 𝑘
= 18 𝑖 − 12 𝑗 + 16 𝑘

Example 43: If 𝒇 = 𝒚𝒛𝟐 𝒊 − 𝟑𝒙𝒛𝟐 𝒋 + 𝟐𝒙𝒚𝒛 𝒌, 𝒈 = 𝟑𝒙 𝒊 + 𝟒𝒛 𝒋 − 𝒙𝒚 𝒌 and ∅ = 𝒙𝒚𝒛; find


(i) 𝒇 × 𝛁∅ (ii) 𝒇 × 𝛁 ∅ (iii) 𝛁 × 𝒇 × 𝐠 (iv) 𝐠 ∙ 𝛁 × 𝒇

𝜕∅ 𝜕∅ 𝜕∅
Solution: ∇∅ = 𝑖 +𝑗 +𝑘 = 𝑦𝑧 𝑖 + 𝑧𝑥 𝑗 + 𝑥𝑦 𝑘
𝜕𝑥 𝜕𝑦 𝜕𝑧

34
𝑖 𝑗 𝑘
2 2 2 2 3
(i) 𝑓 × ∇∅ = 𝑦𝑧 2 −3𝑥𝑧 2 2𝑥𝑦𝑧 = −5𝑥 𝑦𝑧 𝑖 + 𝑥𝑦 𝑧 𝑗 + 4𝑥𝑦𝑧 𝑘
𝑦𝑧 𝑥𝑧 𝑥𝑦
𝑖 𝑗 𝑘
2
(ii) 𝑓 × ∇= 𝑦𝑧 −3𝑥𝑧 2 2𝑥𝑦𝑧
𝜕 𝜕 𝜕
𝜕𝑥 𝜕𝑦 𝜕𝑧

𝜕 𝜕 𝜕 𝜕 𝜕 𝜕
= 𝑖 −3𝑥𝑧 2 𝜕𝑧 − 2𝑥𝑦𝑧 𝜕𝑦 + 𝑗 2𝑥𝑦𝑧 𝜕𝑥 − 𝑦𝑧 2 𝜕𝑧 + 𝑘 𝑦𝑧 2 𝜕𝑦 + 3𝑥𝑧 2 𝜕𝑥
𝜕∅ 𝜕∅ 𝜕∅ 𝜕∅ 𝜕∅ 𝜕∅
Now 𝑓 × ∇ ∅ = 𝑖 −3𝑥𝑧 2 𝜕𝑧 − 2𝑥𝑦𝑧 𝜕𝑦 + 𝑗 2𝑥𝑦𝑧 𝜕𝑥 − 𝑦𝑧 2 𝜕𝑧 + 𝑘 𝑦𝑧 2 𝜕𝑦 + 3𝑥𝑧 2 𝜕𝑥

= 𝑖 −3𝑥𝑧 2 𝑦𝑥 − 2𝑥𝑦𝑧 𝑥𝑧 + 𝑗 2𝑥𝑦𝑧 𝑦𝑧 − 𝑦𝑧 2 𝑥𝑦 + 𝑘 𝑦𝑧 2 𝑥𝑧 + 3𝑥𝑧 2 𝑦𝑧


= −5𝑥 2 𝑦𝑧 2 𝑖 + 𝑥𝑦 2 𝑧 2 𝑗 + 4𝑥𝑦𝑧 3 𝑘
𝑖 𝑗 𝑘
𝜕 𝜕 𝜕
(iii) ∇ × 𝑓 = 𝜕𝑥 𝜕𝑦 𝜕𝑧
= 2𝑥𝑧 + 6𝑥𝑧 𝑖 + 2𝑦𝑧 − 2𝑦𝑧 𝑗 + (−3𝑧 2 − 𝑧 2 ) 𝑘
𝑦𝑧 2 −3𝑥𝑧 2 2𝑥𝑦𝑧
= 8𝑥𝑧 𝑖 + 0 𝑗 − 4𝑧 2 𝑘
𝑖 𝑗 𝑘
3 2 2 2
Now ∇ × 𝑓 × g = 8𝑥𝑧 0 −4𝑧 2 = 0 + 16𝑧 𝑖 + −12𝑥𝑧 + 8𝑥 𝑦𝑧 𝑗 + (32𝑥𝑧 − 0) 𝑘
3𝑥 4𝑧 −𝑥𝑦
= 16𝑧 3 𝑖 + −12𝑥𝑧 2 + 8𝑥 2 𝑦𝑧 𝑗 + 32𝑥𝑧 2 𝑘

g ∙ ∇ × 𝑓 = 3𝑥 𝑖 + 4𝑧 𝑗 − 𝑥𝑦 𝑘 ∙ 8𝑥𝑧 𝑖 + 0 𝑗 − 4𝑧 2 𝑘 = 24𝑥 2 𝑧 + 4𝑥𝑦𝑧 2

Example 44: Find the directional derivative of 𝛁 ∙ 𝛁∅ at the point (1, -2, 1) in the direction of
the normal to the surface 𝒙𝒚𝟐 𝒛 = 𝟑𝒙 + 𝒛𝟐 where ∅ = 𝟐𝒙𝟑 𝒚𝟐 𝒛𝟒 . [Raipur 2005]

Solution: Given ∅ = 2𝑥 3 𝑦 2 𝑧 4
𝜕∅ 𝜕∅ 𝜕∅
So ∇∅ = 𝑖 +𝑗 +𝑘 = 𝑖 6𝑥 2 𝑦 2 𝑧 4 + 𝑗 (4𝑥 3 𝑦𝑧 4 ) + 𝑘 (8𝑥 3 𝑦 2 𝑧 3 )
𝜕𝑥 𝜕𝑦 𝜕𝑧
𝜕 𝜕 𝜕
And 𝑓 = ∇ ∙ ∇∅ = 6𝑥 2 𝑦 2 𝑧 4 + (4𝑥 3 𝑦𝑧 4 ) + 𝜕𝑧 (8𝑥 3 𝑦 2 𝑧 3 )
𝜕𝑥 𝜕𝑦

= 12𝑥𝑦 2 𝑧 4 + 4𝑥 3 𝑧 4 + 24 𝑥 3 𝑦 2 𝑧 2
𝜕𝑓 𝜕𝑓 𝜕𝑓
Consider 𝑔𝑟𝑎𝑑 𝑓 = ∇𝑓 = 𝑖 +𝑗 +𝑘
𝜕𝑥 𝜕𝑦 𝜕𝑧

= 𝑖 12𝑦 2 𝑧 4 + 12𝑥 2 𝑧 4 + 72𝑥 3 𝑦 2 𝑧 2 + 𝑗 24𝑥𝑦𝑧 4 + 48𝑥 3 𝑦𝑧 2


+ 𝑘 (48𝑥𝑦 2 𝑧 3 + 16𝑥 3 𝑧 3 + 48𝑥 3 𝑦 2 𝑧)
At point (1, -2, 1)
𝑔𝑟𝑎𝑑 𝑓 = 𝑖 (48 + 12 + 288) + 𝑗 −48 − 96 + 𝑘 192 + 16 + 192
= 348 𝑖 − 144 𝑗 + 400 𝑘
Now consider the surface 𝑔 = 𝑥𝑦 2 𝑧 − 3𝑥 − 𝑧 2 = 0

35
𝜕𝑔 𝜕𝑔 𝜕𝑔
So, ∇𝑔 = 𝑖 +𝑗 +𝑘 = 𝑖 𝑦 2 𝑧 − 3 + 𝑗 2𝑥𝑦𝑧 + 𝑘 𝑥𝑦 2 − 2𝑧
𝜕𝑥 𝜕𝑦 𝜕𝑧

The vector normal to the surface (1) at point (1, -2, 1) is given by
𝑛 = 𝑖 − 4𝑗 + 2𝑘
And 𝑛 = 1 + 16 + 4 = 21
So the direction derivative of 𝑓 = ∇ ∙ ∇∅ at the point (1, -2, 1) in the direction of the normal to the
𝑛
surface (1) is 𝑔𝑟𝑎𝑑 𝑓 .
𝑛
1
= 348 𝑖 − 144 𝑗 + 400 𝑘 . 𝑖 − 4𝑗 + 2𝑘
21
1
= 348 + 576 + 800
21

= 𝟏𝟕𝟐𝟒/ 𝟐𝟏

Example 45: If r is the distance of a point (x, y, z) from the origin, prove that
𝟏 𝟏
𝒄𝒖𝒓𝒍 𝒌 × 𝒈𝒓𝒂𝒅 𝒓
+ 𝒈𝒓𝒂𝒅 𝒌 ∙ 𝒈𝒓𝒂𝒅 𝒓
= 𝟎 where 𝒌 is the unit vector in the direction of OZ.

Solution: Here 𝑟 = 𝑥 𝑖 + 𝑦 𝑗 + 𝑧 𝑘 so that 𝑟 = 𝑥2 + 𝑦2 + 𝑧2


1
1 𝜕 𝜕 𝜕 1 𝜕 𝜕 𝜕 −
Now 𝑔𝑟𝑎𝑑 = 𝑖 +𝑗 +𝑘 = 𝑖 +𝑗 +𝑘 𝑥2 + 𝑦2 + 𝑧2 2
𝑟 𝜕𝑥 𝜕𝑦 𝜕𝑧 𝑟 𝜕𝑥 𝜕𝑦 𝜕𝑧
3
1 − 1
= − 2 𝑥2 + 𝑦2 + 𝑧2 2 (2𝑥 𝑖 + 2𝑦 𝑗 + 2𝑧 𝑘 ) = − 𝑟 3 𝑟
1 1 𝑧
𝑔𝑟𝑎𝑑 𝑘 ∙ 𝑔𝑟𝑎𝑑 = 𝑔𝑟𝑎𝑑 − 𝑟 3 𝑧 = −𝑔𝑟𝑎𝑑
𝑟 𝑥 2 +𝑦 2 +𝑧 2 3/2

3𝑧𝑥 3𝑧𝑦 𝑥 2 +𝑦 2 −2𝑧 2


= 𝑖+ 𝑗+ 𝑘 … (1)
𝑥 2 +𝑦 2 +𝑧 2 5/2 𝑥 2 +𝑦 2 +𝑧 2 5/2 𝑥 2 +𝑦 2 +𝑧 2 5/2
1 1 1
And 𝑐𝑢𝑟𝑙 𝑘 × 𝑔𝑟𝑎𝑑 = 𝑐𝑢𝑟𝑙 𝑘 × − 𝑟 = 𝑐𝑢𝑟𝑙 𝑦𝑖−𝑥𝑗
𝑟 𝑟3 𝑟3

3𝑧𝑥 3𝑧𝑦 𝑥 2 +𝑦 2 −2𝑧 2


=− 𝑖− 𝑗− 𝑘 … (2)
𝑥 2 +𝑦 2 +𝑧 2 5/2 𝑥 2 +𝑦 2 +𝑧 2 5/2 𝑥 2 +𝑦 2 +𝑧 2 5/2
1 1
Adding (1) and (2), 𝑐𝑢𝑟𝑙 𝑘 × 𝑔𝑟𝑎𝑑 𝑟
+ 𝑔𝑟𝑎𝑑 𝑘 ∙ 𝑔𝑟𝑎𝑑 𝑟
=0.

𝟏 𝟑(𝒂∙𝒓)(𝒃∙𝒓) 𝒂∙𝒃
Example 46: Prove that 𝒂 ∙ 𝛁 𝒃 ∙ 𝛁 𝒓 = − , where 𝒂 and 𝒃 are constant vectors.
𝒓𝟓 𝒓𝟑
3
1 1 −
Solution: From example 45, ∇ 𝑟 = − 𝑟 3 𝑟 = − 𝑥 2 + 𝑦 2 + 𝑧 2 2 (𝑥 𝑖 + 𝑦 𝑗 + 𝑧 𝑘 )
Let the constant vectors are 𝑎 = 𝑎1 𝑖 + 𝑎2 𝑗 + 𝑎3 𝑘 and 𝑏 = 𝑏1 𝑖 + 𝑏2 𝑗 + 𝑏3 𝑘
3
1 −
So ∇ 𝑏 ∙ ∇ 𝑟 = ∇ − 𝑥 2 + 𝑦 2 + 𝑧 2 2 𝑏1 𝑥 + 𝑏2 𝑦 + 𝑏3 𝑧
3 3
− −
= − 𝑥2 + 𝑦2 + 𝑧2 2 ∇ 𝑏1 𝑥 + 𝑏2 𝑦 + 𝑏3 𝑧 − 𝑏1 𝑥 + 𝑏2 𝑦 + 𝑏3 𝑧 ∇ 𝑥 2 + 𝑦 2 + 𝑧 2 2

3 5
− −
= − 𝑥2 + 𝑦2 + 𝑧2 2 𝑏1 𝑖 + 𝑏2 𝑗 + 𝑏3 𝑘 +3 𝑏1 𝑥 + 𝑏2 𝑦 + 𝑏3 𝑧 𝑥2 + 𝑦2 + 𝑧2 2 (𝑥 𝑖 + 𝑦 𝑗 + 𝑧 𝑘 )

𝑏 3 𝑏 ∙𝑟 𝑟
=− +
𝑟3 𝑟5

36
1 𝑏 3 𝑏∙𝑟 𝑟 3(𝑎∙𝑟)(𝑏∙𝑟) 𝑎∙𝑏
Now 𝑎 ∙ ∇ 𝑏 ∙ ∇ 𝑟 = 𝑎 ∙ − 𝑟3 + = − 𝑟3
𝑟5 𝑟5
Hence Proved.

ASSIGNMENT 4

1. If 𝑓 = 𝑥 + 𝑦 + 1 𝑖 + 𝑗 − (𝑥 + 𝑦)𝑘 , show that 𝑓 ∙ 𝑐𝑢𝑟𝑙 𝑓 = 0.


2. Evaluate (a) 𝑑𝑖𝑣 3𝑥 2 𝑖 + 5𝑥𝑦 2 𝑗 + 𝑥𝑦𝑧 3 𝑘 at the point (1, 2, 3).
(b) 𝑐𝑢𝑟𝑙 [𝑒 𝑥𝑦𝑧 𝑖 + 𝑗 + 𝑘 ].
3. Find the value of ‘a’ if the vector 𝑎𝑥 2 𝑦 + 𝑦𝑧 𝑖 + 𝑥𝑦 2 − 𝑥𝑧 2 𝑗 + 2𝑥𝑦𝑧 − 2𝑥 2 𝑦 2 𝑘 has zero
divergence. Find the curl of above vector which has zero divergence .
4. If 𝑣 = 𝑟 / 𝑥 2 + 𝑦 2 + 𝑧 2 , show that ∇ ∙ 𝑣 = 2/ 𝑥 2 + 𝑦 2 + 𝑧 2 and ∇ × 𝑣 = 0.
5. If 𝑢 = 𝑥 2 + 𝑦 2 + 𝑧 2 and 𝑣 = 𝑥𝑖 + 𝑦𝑗 + 𝑧𝑘, show that 𝑑𝑖𝑣 𝑢 𝑣 = 5𝑢.
6. Show that each of following vectors are solenoidal:
(a) 𝑥 + 3𝑦 𝑖 + 𝑦 − 3𝑧 𝑗 + 𝑥 − 2𝑧 𝑘 (b) 3𝑦 4 𝑧 2 𝑖 + 4𝑥 3 𝑧 2 𝑗 + 3𝑥 2 𝑦 2 𝑘 (c) ∇u × ∇𝑣
7. If 𝑟 = 𝑥𝑖 + 𝑦𝑗 + 𝑧𝑘 and 𝑟 = 𝑟 ≠ 0, show that
(a) ∇ 1/𝑟 2 = −2𝑟/𝑟 4 ; ∇ ∙ 𝑟/𝑟 2 = 1/𝑟 2 (b) ∇ ∙ 𝑟 𝑛 𝑟 = (𝑛 + 3)𝑟 𝑛 ; ∇ × 𝑟 𝑛 𝑟 = 0.
𝑟 2
(c) ∇ ∇ ∙ 𝑟 = − 𝑟 3 𝑟

8. Prove that (a) ∇𝑎2 = 2 𝑎 ∙ ∇ 𝑎 + 2𝑎 × ∇ × 𝑎 , where 𝑎 is the constant vector.


(b) ∇ × 𝑟 × 𝑢 = 𝑟 ∇ ∙ u − 2𝑢 − 𝑟 ∙ ∇ 𝑢.
9. (a) If ∅ = 𝑥 2 + 𝑦 2 + 𝑧 2 −𝑛
, find the 𝑑𝑖𝑣 𝑔𝑟𝑎𝑑 ∅ and determine n if 𝑑𝑖𝑣 𝑔𝑟𝑎𝑑 ∅ = 0.
(b) Show that 𝑔𝑟𝑎𝑑 𝑟 𝑛 = 𝑛(𝑛 + 1)𝑟 𝑛−2 , where 𝑟 2 = 𝑥 2 + 𝑦 2 + 𝑧 2 .
1 𝜕𝐻 1 𝜕𝐷
10. In electromagnetic theory, we have ∇ ∙ 𝐷 = 𝜌, ∇ ∙ 𝐻 = 0, ∇ × 𝐷 = − 𝑐 𝜕𝑡
,∇ ×𝐻 =
𝑐
𝜌𝑉 +
𝜕𝑡
.
1 𝜕2𝐻 1 1 𝜕2𝐷 1 𝜕
Prove that ∇2 𝐻 − 𝑐 2 = − 𝑐 ∇ × 𝜌𝑉 and ∇2 𝐷 − 𝑐 2 = ∇𝜌 + 𝑐 2 𝜕𝑡 𝜌𝑉 .
𝜕𝑡 2 𝜕𝑡 2

11. If 𝑢 = 𝑥2 𝑦𝑧, 𝑣 = 𝑥𝑦 − 3𝑧2 , find (i) ∇ ∇𝑢 ∙ ∇𝑣 (ii) ∇ ∙ ∇𝑢 × ∇𝑣 .


12. For a solenoidal vector 𝑓, show that 𝑐𝑢𝑟𝑙 𝑐𝑢𝑟𝑙 𝑐𝑢𝑟𝑙 𝑐𝑢𝑟𝑙 𝑓 = ∇4 𝑓.
13. Calculate (i) 𝑐𝑢𝑟𝑙 𝑔𝑟𝑎𝑑 𝑓 , given 𝑓 𝑥, 𝑦, 𝑧 = 𝑥 2 + 𝑦 2 − 𝑧 [BPTU 2006]
(ii) 𝑐𝑢𝑟𝑙(𝑐𝑢𝑟𝑙 𝑎), given 𝑎 = 𝑥 2 𝑦 𝑖 + 𝑦 2 𝑧 𝑗 + 𝑧 2 𝑦 𝑘
14. Show that each of the following vectors are solenoidal:
(i) −𝑥 2 + 𝑦𝑧 𝑖 + 4𝑦 − 𝑧 2 𝑥 𝑗 + (2𝑥𝑧 − 4𝑧) 𝑘
(ii) 3𝑦 4 𝑧 2 𝑖 + 4𝑥 3 𝑧 2 𝑗 + 3𝑥 2 𝑦 2 𝑘
(iii) ∇𝜙 × ∇𝜓

INTEGRAL VECTOR CALCULUS

37
17.9 INTEGRATION OF VECTORS
𝑑
If two vector functions 𝑓 (𝑡) and 𝑔(𝑡) be such that 𝑔(𝑡) = 𝑓 (𝑡), then 𝑔(𝑡) is called an integral
𝑑𝑡
of 𝑓 (𝑡) with respect to a scalar variable t and we can write 𝑓(𝑡)𝑑𝑡 = 𝑔(𝑡).

Indefinite Integral
𝑑 𝑑
If 𝑐 be an arbitrary constant vector and 𝑓 𝑡 = 𝑑𝑡 𝑔(𝑡) = 𝑑𝑡 𝑔 𝑡 + 𝑐 , then 𝑓(𝑡)𝑑𝑡 = 𝑔 𝑡 + 𝑐 .
This is called the indefinite integral of 𝑓 (𝑡).

Definite Integral
𝑑
If 𝑑𝑡 𝑔(𝑡) = 𝑓 𝑡 for all values of 𝑡 in the interval 𝑎, 𝑏 , then the definite integral of 𝑓 𝑡 between
𝑏 𝑏
𝑎 and 𝑏 is defined and denoted by 𝑎
𝑓 𝑡 𝑑𝑡 = 𝑔(𝑡) 𝑎 =𝑔 𝑏 −𝑔 𝑎 .

𝒅𝟐 𝒇 𝒅𝒇
Example 47: If = 𝟔𝒕 𝒊 − 𝟏𝟐𝒕𝟐 𝒋 + 𝟒 𝐜𝐨𝐬 𝒕 𝒌, find 𝒇, given that = − 𝒊 − 𝟑 𝒌 and
𝒅𝒕𝟐 𝒅𝒕
𝒇 = 𝟐 𝒊 + 𝒋 at 𝒕 = 𝟎.

𝑑2𝑓
Solution: Given that = 6𝑡 𝑖 − 12𝑡 2 𝑗 + 4 cos 𝑡 𝑘 … (1)
𝑑𝑡 2
Integrating (1) with respect to t,
𝑑2𝑓
𝑑𝑡 = 6𝑡 𝑖 − 12𝑡 2 𝑗 + 4 cos 𝑡 𝑘 𝑑𝑡
𝑑𝑡 2

𝑑𝑓
Implying = 3𝑡 2 𝑖 − 4𝑡 3 𝑗 + 4 sin 𝑡 𝑘 + 𝑐1 … (2)
𝑑𝑡

Now, integrating (2) with respect to t,


𝑑𝑓
𝑑𝑡 = 3𝑡 2 𝑖 − 4𝑡 3 𝑗 + 4 sin 𝑡 𝑘 + 𝑐1 𝑑𝑡
𝑑𝑡

Implying 𝑓 = 𝑡 3 𝑖 − 𝑡 4 𝑗 − 4 cos 𝑡 𝑘 + 𝑐1 𝑡 + 𝑐2 … (3)


𝑑𝑓
Also we are given that at 𝑡 = 0, = −𝑖−3𝑘 … (4)
𝑑𝑡

𝑓 =2𝑖+𝑗 … (5)

Using (2) and (4), 𝑐1 = − 𝑖 − 3 𝑘

Using (3) and (5), −4 𝑘 + 𝑐2 = 2 𝑖 + 𝑗 => 𝑐2 = 2 𝑖 + 𝑗 + 4 𝑘

Putting the values of the constant vectors 𝑐1 & 𝑐2 in (3), we get

𝑓 = 𝑡 3 𝑖 − 𝑡 4 𝑗 − 4 cos 𝑡 𝑘 + − 𝑖 − 3 𝑘 𝑡 + 2 𝑖 + 𝑗 + 4 𝑘

= 𝑡 3 − 𝑡 + 2 𝑖 + 1 − 𝑡 4 𝑗 − 4 cos 𝑡 + 3 𝑡 + 4 𝑘

ASSIGNMENT 5

4
1. For given 𝑓 𝑡 = 5𝑡 2 − 3𝑡 𝑖 + 6𝑡 3 𝑗 − 7𝑡 𝑘, evaluate 2
𝑓 𝑡 𝑑𝑡.
1 𝑑2𝑟
2. Given 𝑟 𝑡 = 3𝑡 2 𝑖 + 𝑡 𝑗 − 𝑡 3 𝑘, evaluate 0
𝑟 × 𝑑𝑡 2 𝑑𝑡.

38
2 𝑖 − 𝑗 + 2 𝑘, 𝑤𝑕𝑒𝑛 𝑡 = 1 2 𝑑𝑟
3. If 𝑟 𝑡 = , show that 1
𝑟 ∙ 𝑑𝑡 𝑑𝑡 = 10.
3 𝑖 − 2𝑗 + 4 𝑘 , 𝑤𝑕𝑒𝑛 𝑡 = 2
4. The acceleration of a particle at any time 𝑡 ≥ 0 is given by 12 cos 2𝑡 𝑖 − 8 sin 2𝑡 𝑗 + 16𝑡 𝑘, the
displacement and velocity are initially zero. Find the velocity and displacement at any time.

17.10 LINE INTEGRAL


    
Let r (u)  x(u)i  y(u) j  z (u)k defines a curve C joining points P1 and P2 where r (u ) is the
position vector of ( x, y, z ) and the value of u at P1 and P2 is u1 and u2, respectively.
  
Now if A( x, y, z )  A1i  A2 j  A3 k be vector function of defined position and continuous along C,
P2  
then the integral of the tangential component of A along C from P1 to P2 written as  A . dr is
P1

known as the line integral. Also in terms of Cartesian components, we have


P2   P2      
P1
A. dr   ( A1 i  A2 j  A3 k ) . (dx i  dy j  dz k )
P1

P2
  ( A1 dx  A2 dy  A3 dz )   ( A1 dx  A2 dy  A3 dz )
P1 C

 
If A (vector function of the position) represents the force F on a particle moving along C, then the

line integral represents the work done by the force F . If C is a simple closed curve, then the integral
around C is generally written as

P1 P2
C
X
O

Fig 17.11
Y

 
 . dr   ( A1 dx  A2 dy  A3 dz)
A

In Fluid Mechanics and Aerodynamics, the above integral is called circulation of A about C, where

A represents the velocity of the fluid.

Let A  grad  , then we have

 
Q Q  Q          
P A . dr  P ( ) . dr  P  x y z 
 i  j  k  . ( i dx  j dy  k dz )

39
Q    
   dx  dy  dz 
P x
 y z 

Q
  d  [ ]QP  Q   P … (1)
P

Q  
We see that the integral P
A . dr depends on the value of  at the end P and Q and not on the
particular path. In case  is single valued and the integral is taken round a closed curve, the terminal
points P and Q coincide and  B   P .

[Because function  is uniform]

The integration along a closed curve is denoted by the sign of circle in the mid of the integral sign
i.e. for a uniform function, we have

C
( ) . dr  0 … (2)


The converse of the above result is also true i.e. if there exists a vector A and its integral round
every closed curve in the region under consideration vanishes, then there exist a point function 

such that A  grad .

To prove this consider any closed curve ABCD such that the integral round it is zero, so integral along
ABC must be equal to that along ADC. Similarly, the integral along ABC must be equal to that along
any curve joining A to C, i.e. independent of the path from A to C with A be a fixed point and C a
variable point. Then due to the fact that line integral is independent of the path chosen, the value of
C 
the line integral from A to C must be a scalar point function, say  i.e.  A . dr  
A

 
Now if d is the increment in  due to a small displacement 𝑑𝑟 of 𝑟, then we have d  A. dr

   
But we already know that d   . dr , so A. dr  ( ) . dr ,
   
 ( A   ) . dr  0, which is true for all dr and hence A   .

The vector A is called a potential vector (or gradient vector), and in cartesian component; the
 
condition that A. dr  A1 dx  A2 dy  A3 dz be a perfect differential can be thrown easily into the
form

A2 A3 A3 A1 A1 A2


  0,   0,  0 … (3)
z y x z y x
  
Circulation: If A represents the velocity of a fluid particle, then the line integral  A . dr is called
C

the circulation of A along C.

The vector point function A , is said to be irrotational in a region, if its circulation along every
 
closed curve in the region is zero i.e.  A . dr  0
C

40

Theorem: The necessary and sufficient condition for a vector point function A to be irrotaional in a

simply connected region is the curl A  0 at every point of the region.

Work: If A represents the force acting on a particle moving along an arc PQ then the work done
   
during the small displacement dr is equal to A . dr . Therefore, the total work done by A during the
Q 
displacement from P to Q is given by the line integral  A . dr .
P

Example 48: Evaluate the line integral (𝒙𝟐 + 𝒙𝒚)𝒅𝒙 + (𝒙𝟐 + 𝒚𝟐 )𝒅𝒚 , where 𝑪 is the square
formed by the lines 𝒚 = ±𝟏 and 𝒙 = ±𝟏.

Solution: Curve 𝐶 is square in the 𝑥𝑦 plane where 𝑧 = 0

∴ 𝑟 = 𝑥𝑖 + 𝑦𝑗 in 𝑥𝑦 plane

𝑑𝑟 = 𝑑𝑥𝑖 + 𝑑𝑦𝑗 … (1)

Now 𝐹. 𝑑𝑟 = 𝑥 2 + 𝑥𝑦 𝑑𝑥 + 𝑥 2 + 𝑦 2 𝑑𝑦
𝐶

Path of the integration is shown in figure 17.12, it consists of lines


AB, BC, CD and DA. As curve C is a square, then

On AB, 𝑦 = −1 ⇒ 𝑑𝑦 = 0 and 𝑥 varies from −1 to 1

On BC, 𝑥=1 ⇒ 𝑑𝑥 = 0 and 𝑦 varies from −1 to 1

On CD, 𝑦 = 1 ⇒ 𝑑𝑦 = 0 and 𝑥 varies from 1 to −1

On DA, 𝑥 = −1 ⇒ 𝑑𝑥 = 0 and 𝑦 varies from 1 to −1

𝐶
𝐹 . 𝑑𝑟 = 𝐴𝐵
𝑥 2 − 𝑥 𝑑𝑥 + 𝐵𝐶
1 + 𝑦 2 𝑑𝑦 + 𝐶𝐷
𝑥 2 + 𝑥 𝑑𝑥 + 𝐷𝐴
1 + 𝑦 2 𝑑𝑦

1 1 −1 −1
= −1
𝑥 2 − 𝑥 𝑑𝑥 + −1
1 + 𝑦 2 𝑑𝑦 + 1
𝑥 2 + 𝑥 𝑑𝑥 + 1
1 + 𝑦 2 𝑑𝑦

1 −1
𝑥3 𝑥2 1 𝑥3 𝑥2 1
= − + −1
1 + 𝑦 2 𝑑𝑦 + + − −1
1 + 𝑦 2 𝑑𝑦
3 2 −1 3 2 1

1 1 1 1 1 1 1 1
= − 2 + 3 + 2 + −3 + 2 − 3 − 2 = 0
3

Example 49: If 𝑭 = 𝟑𝒙𝒚 𝒊 − 𝒚𝟐 𝒋, evaluate 𝑭. 𝒅𝒓 where 𝑪 is the arc of the parabola 𝒚 = 𝟐𝒙𝟐
𝑪
from (𝟎, 𝟎) to (𝟏, 𝟐).

Solution: Because the integration is performed in the 𝑥𝑦-plane (𝑧 = 0), we take

𝑟 = 𝑥𝑖 + 𝑦𝑗 so that 𝑑𝑟 = 𝑑𝑥 𝑖 + 𝑑𝑦 𝑗

∴ 𝐹 . 𝑑𝑟 = 3𝑥𝑦 𝑖 − 𝑦 2 𝑗 . 𝑑𝑥 𝑖 + 𝑑𝑦 𝑗 = 3𝑥𝑦 𝑑𝑥 − 𝑦 2 𝑑𝑦

On the curve C: 𝑦 = 2𝑥 2 from (0, 0) to (1, 2)

41
𝐹 . 𝑑𝑟 = 3𝑥 2𝑥 2 𝑑𝑥 − 2𝑥 2 2 4𝑥 𝑑𝑥 = (6𝑥 3 − 16𝑥 5 )𝑑𝑥

Also x varies from 0 to 1.


1
1 6𝑥 4 16𝑥 6 3 8 7
𝐶
𝐹 . 𝑑𝑟 = 0
(6𝑥 3 − 16𝑥 5 )𝑑𝑥 = − = 2 −3 = −6
4 6 0

Note: If the curve is traversed in the opposite direction, that is from (1, 2) to (0, 0), the value of the integral
7
would be 6.

Example 50: A vector field is given by 𝑭 = 𝐬𝐢𝐧 𝒚 𝒊 + 𝒙(𝟏 + 𝐜𝐨𝐬 𝒚)𝒋. Evaluate the line integral
over the circular path given by 𝒙𝟐 + 𝒚𝟐 = 𝒂𝟐 , 𝒛 = 𝟎. [PTU 2003]

Solution: The parametric equations of the circular path are 𝑥 = 𝑎 cos 𝑡 , 𝑦 = 𝑎 sin 𝑡 , 𝑧 = 0 where 𝑡
varies from 0 to 2𝜋. Since the particle moves in the 𝑥𝑦-plane (𝑧 = 0), we can take 𝑟 = 𝑥 𝑖 + 𝑦 𝑗.

𝐶
𝐹 . 𝑑𝑟 = 𝐶
sin 𝑦 𝑖 + 𝑥(1 + cos 𝑦)𝑗 . 𝑑𝑥 𝑖 + 𝑑𝑦 𝑗

= 𝐶
sin 𝑦 𝑑𝑥 + 𝑥 1 + cos 𝑦 𝑑𝑦
= 𝐶
sin 𝑦 𝑑𝑥 + 𝑥 cos 𝑦 𝑑𝑦 + 𝑥 𝑑𝑦 = 𝐶
𝑑(𝑥 sin 𝑦) + 𝐶
𝑥 𝑑𝑦

2𝜋 2𝜋
= 0
𝑑 a cos 𝑡 sin a sin 𝑡 𝑑𝑡 + 0
a cos 𝑡 . a cos 𝑡 𝑑𝑡

2𝜋 2𝜋
= a cos 𝑡 sin(a sin 𝑡) 0 + 𝑎2 0
cos 2 𝑡 𝑑𝑡

𝑎2 2𝜋 𝑎2 sin 2𝑡 2𝜋 𝑎2
= 0
(1 + cos 2𝑡)𝑑𝑡 = 𝑡+ = 2𝜋 = 𝜋𝑎2
2 2 2 0 2

Example 51: Compute the line integral 𝑪


(𝒚𝟐 𝒅𝒙 − 𝒙𝟐 𝒅𝒚) about the triangle whose vertices are
(𝟏, 𝟎), (0, 1) and (−𝟏, 𝟎). [NIT Uttrakhand 2011]

Solution: Here the closed curve C is a triangle ABC.

On AB: Equation of line AB is


1−0
𝑦 − 0 = 0−1 (𝑥 − 1) ⇒ 𝑦 = 1−𝑥

∴ 𝑑𝑦 = −𝑑𝑥 and 𝑥 varies from 1 to 0.

On BC: Equation of line BC is


0−1
𝑦 − 1 = −1−0 (𝑥 − 0) ⇒ 𝑦 =1+𝑥

∴ 𝑑𝑦 = 𝑑𝑥 and 𝑥 varies from 0 to -1.

On CA: 𝑦 = 0. Therefore, 𝑑𝑦 = 0 and 𝑥 varies from -1 to 1.

𝐶
𝑦 2 𝑑𝑥 − 𝑥 2 𝑑𝑦 = 𝐴𝐵
𝑦 2 𝑑𝑥 − 𝑥 2 𝑑𝑦 + 𝐵𝐶
𝑦 2 𝑑𝑥 − 𝑥 2 𝑑𝑦 + 𝐶𝐴
𝑦 2 𝑑𝑥 − 𝑥 2 𝑑𝑦

0 −1 1
= 1
(1 − 𝑥)2 𝑑𝑥 − 𝑥 2 (−𝑑𝑥) + 0
(1 + 𝑥)2 𝑑𝑥 − 𝑥 2 𝑑𝑥 + −1
0 𝑑𝑥

42
0 −1
= 1
2𝑥 2 − 2𝑥 + 1 𝑑𝑥 + 0
2𝑥 + 1 𝑑𝑥 + 0

0 −1
2𝑥 3 2𝑥 2 2𝑥 2 2 2
= − +𝑥 + +𝑥 = −3 + 1 − 1 + 1 − 1 = −3
3 2 1 2 0

Example 52: If 𝑭 = 𝟑𝒙𝟐 + 𝟔𝒚 𝒊 − 𝟏𝟒𝒚𝒛𝒋 + 𝟐𝟎𝒙𝒛𝟐 𝒌, evaluate 𝑪


𝑭. 𝒅𝒓 where
(i) 𝑪 is the line joining the point (𝟎, 𝟎, 𝟎) to (𝟏, 𝟏, 𝟏)
(ii) 𝑪 is given by 𝒙 = 𝒕, 𝒚 = 𝒕𝟐 , 𝒛 = 𝒕𝟑 from the point (𝟎, 𝟎, 𝟎) to (𝟏, 𝟏, 𝟏).

Solution: (i) Equation of line joining (0,0,0) to (1,1,1) is


𝑥−0 𝑦−0 𝑧−0
= = 1−0 = 𝑡 (say)
1−0 1−0

∴ Parametric equations of the line C are 𝑥 = 𝑡, 𝑦 = 𝑡, 𝑧 = 𝑡; 0 ≤ 𝑡 ≤ 1

∴ 𝑟 = 𝑥𝑖 + 𝑦𝑗 + 𝑧𝑘 = 𝑡𝑖 + 𝑡𝑗 + 𝑡𝑘
𝒅𝑟
⇒ =𝑖+𝑗+𝑘
𝒅𝒕

Now 𝐹 = 3𝑥 2 + 6𝑦 𝑖 − 14𝑦𝑧𝑗 + 20𝑥𝑧 2 𝑘 = 3𝑡 2 + 6𝑡 𝑖 − 14𝑡 2 𝑗 + 20𝑡 3 𝑘


𝑑𝑟
𝐶
𝐹 . 𝑑𝑟 = 𝐶
𝐹 . 𝑑𝑡 𝑑𝑡

= 𝐶
3𝑡 2 + 6𝑡 𝑖 − 14𝑡 2 𝑗 + 20𝑡 3 𝑘 . 𝑖 + 𝑗 + 𝑘 𝑑𝑡

1
= 0
3𝑡 2 + 6𝑡 − 14𝑡 2 + 20𝑡 3 𝑑𝑡

1
3𝑡 3 6𝑡 2 14𝑡 3 20𝑡 4 14 13
= + − + = 1+3− +5 =
3 2 3 4 0 3 3

(ii) Here the curve C is given by 𝑥 = 𝑡, 𝑦 = 𝑡 2 , 𝑧 = 𝑡 3 from the point (0,0,0) to (1,1,1)

∴ 𝑟 = 𝑥𝑖 + 𝑦𝑗 + 𝑧𝑘 = 𝑡𝑖 + 𝑡 2 𝑗 + 𝑡 3 𝑘
𝒅𝑟
⇒ = 𝑖 + 2𝑡 𝑗 + 3𝑡 2 𝑘
𝒅𝒕

Now 𝐹 = 3𝑥 2 + 6𝑦 𝑖 − 14𝑦𝑧𝑗 + 20𝑥𝑧 2 𝑘 = 9𝑡 2 𝑖 − 14𝑡 5 𝑗 + 20𝑡 7 𝑘


𝑑𝑟
𝐶
𝐹 . 𝑑𝑟 = 𝐶
𝐹 . 𝑑𝑡 𝑑𝑡

= 𝐶
9𝑡 2 𝑖 − 14𝑡 5 𝑗 + 20𝑡 7 𝑘 . 𝑖 + 2𝑡 𝑗 + 3𝑡 2 𝑘 𝑑𝑡

1
= 0
9𝑡 2 − 28𝑡 6 + 60𝑡 9 𝑑𝑡

1
9𝑡 3 28𝑡 7 60𝑡 10
= − + = 3−4+6 =5
3 7 10 0

Example 53: Find the circulation of 𝑭 around the curve 𝑪 where 𝑭 = 𝒚𝒊 + 𝒛𝒋 + 𝒙𝒌 and 𝑪 is the
circle 𝒙𝟐 + 𝒚𝟐 = 𝟏, 𝒛 = 𝟎.
43
Solution: Circulation of 𝐹 along the curve 𝐶 is 𝐹 . 𝑑𝑟
𝐶

Equation of circle is 𝑥 2 + 𝑦 2 = 1, 𝑧 = 0

Its parameteric equations are 𝑥 = cos θ , 𝑦 = sin θ , 𝑧 = 0

Now 𝑟 = 𝑥𝑖 + 𝑦𝑗 + 𝑧𝑘 = cos θ 𝑖 + sin θ 𝑗 + 0𝑘 so that

𝑑𝑟 = − sin θ 𝑖 + cos θ 𝑗 + 0𝑘 𝑑θ

Also 𝐹 = 𝑦𝑖 + 𝑧𝑗 + 𝑥𝑘 = sin θ 𝑖 + 0𝑗 + cos θ 𝑘

∴ 𝐶
𝐹 . 𝑑𝑟 = 𝐶
sin θ 𝑖 + 0𝑗 + cos θ 𝑘 . − sin θ 𝑖 + cos θ 𝑗 + 0𝑘 𝑑θ

2𝜋
= 0
− sin2 θ 𝑑𝜃 (since along the circle, 𝜃 varies from 0 to 2𝜋)

2𝜋 1−cos 2θ θ sin 2θ 2π
=− 0
𝑑θ =− −
2 2 4 0


=− 2
− 0 − (0 − 0) = −π

Example 54: Find the work done in moving a particle once round a circle 𝑪 in the 𝒙𝒚 plane, if
the circle has its centre at the origin and radius 2 units and the force field is given as

𝑭 = 𝟐𝒙 − 𝒚 + 𝟐𝒛 𝒊 + 𝒙 + 𝒚 − 𝒛 𝒋 + (𝟑𝒙 − 𝟐𝒚 − 𝟓𝒛)𝒌.

Solution: Equation of a circle having centre (0, 0) with radius 2 in 𝑥𝑦 plane is 𝑥 2 + 𝑦 2 = 4 .


Parametric equations of this circle are 𝑥 = 2 cos 𝑡 , 𝑦 = 2 sin 𝑡 , 𝑧 = 0.

Since integration is to be performed around a circle in 𝑥𝑦 plane,


𝑑𝑟
∴ 𝑟 = 𝑥𝑖 + 𝑦𝑗 = 2 cos 𝑡 𝑖 + 2 sin 𝑡 𝑗 ⇒ = −2 sin 𝑡 𝑖 + 2 cos 𝑡 𝑗
𝑑𝑡

𝑑𝑟
Work done, 𝐶
𝐹 . 𝑑𝑡 𝑑𝑡

= 2𝑥 − 𝑦 + 2𝑧 𝑖 + 𝑥 + 𝑦 − 𝑧 𝑗 + (3𝑥 − 2𝑦 − 5𝑧)𝑘 . −2 sin 𝑡 𝑖 + 2 cos 𝑡 𝑗 𝑑𝑡

= 4 cos 𝑡 − 2 sin 𝑡 𝑖 + 2 cos 𝑡 + 2 sin 𝑡 𝑗 + (6 cos 𝑡 − 4 sin 𝑡)𝑘 . −2 sin 𝑡 𝑖 + 2 cos 𝑡 𝑗 𝑑𝑡

In moving round the circle, 𝑡 varies from 0 to 2𝜋


2𝜋
∴ Work done = 0
4 cos 𝑡 − 2 sin 𝑡 (−2 sin 𝑡) + 2 cos 𝑡 + 2 sin 𝑡 (2 cos 𝑡) 𝑑𝑡

2𝜋
= 0
−8 cos 𝑡 sin 𝑡 + 4 sin2 𝑡 + 4 cos 2 𝑡 + 4 sin 𝑡 cos 𝑡 𝑑𝑡

2𝜋
2𝜋 sin 2 𝑡
= 0
4 − 4 sin 𝑡 cos 𝑡 𝑑𝑡 = 4𝑡 − 4 2 0

= 8𝜋 − 2 sin2 2𝜋 − (0 − 0) = 8𝜋

ASSIGNMENT 6
44
1. Using the line integral, compute the work done by the force 𝐹 = 2𝑦 + 3 𝑖 + 𝑥𝑧 𝑗 + (𝑦𝑧 − 𝑥)𝑘,
when it moves a particle from (0,0,0) to (2,1,1) along the curve 𝑥 = 2𝑡 2 , 𝑦 = 𝑡, 𝑧 = 𝑡 3 .
2. Find the work done in moving a particle in the force field 𝐹 = 3𝑥 2 𝑖 + 2𝑥𝑧 − 𝑦 𝑗 + 𝑧𝑘, along
(a) the straight line from (0,0,0) to (2,1,3).
(b) the curve defined by 𝑥 2 = 4𝑦, 3𝑥 3 = 8𝑧 from 𝑥 = 0 to 𝑥 = 2.

3. If C is a simple closed curve in the 𝑥𝑦 plane not enclosing the origin, show that 𝐶
𝐹 . 𝑑𝑟 = 0,
𝑦𝑖 −𝑥𝑗
where 𝐹 = 𝑥 2 +𝑦 2 .

4. If 𝑓 = 5𝑥𝑦 − 6𝑥 2 𝑖 + 2𝑦 − 4𝑥 𝑗, evaluate 𝐶
𝑓 . 𝑑𝑟 along the curve C in xy-plane, 𝑦 = 𝑥 3
from the point 1, 1 to (2, 8).

5. Evaluate 𝐶
(𝑥𝑦 + 𝑧 2 )𝑑𝑠 where C is the arc of the helix 𝑥 = cos 𝑡 , 𝑦 = sin 𝑡 , 𝑧 = 𝑡 which joins
the points 1, 0, 0 𝑎𝑛𝑑 (−1, 0, 𝜋).

6. If 𝐹 = 2𝑦𝑖 − 𝑧𝑗 + 𝑥𝑘, evaluate 𝐶


𝐹 × 𝑑𝑟 along the curve 𝑥 = cos 𝑡, 𝑦 = sin 𝑡, 𝑧 = 2 cos 𝑡 from
𝜋
𝑡 = 0 to 𝑡 = 2 .

17.11 SURFACE INTEGRALS AND FLUX

An integral which is to be evaluated over a surface is called a surface integral. Suppose 𝑆 is a surface
of finite area. Divide the area 𝑆 into n sub-areas 𝛿𝑆1 , 𝛿𝑆2 , … … , 𝛿𝑆𝑛 .
In each area 𝛿𝑆𝑖 , choose an arbitrary point 𝑃𝑖 𝑥𝑖 , 𝑦𝑖 , 𝑧𝑖 . Let 𝜑 define
a scalar point function over the area 𝑆.
𝑛
Now from the sum 𝑖=1 𝜑 𝑃𝑖 𝛿𝑆𝑖 , where 𝜑 𝑃𝑖 = 𝜑 𝑥𝑖 , 𝑦𝑖 , 𝑧𝑖

Now let us take the limit of the sum as 𝑛 → ∞, each sub-area 𝛿𝑆𝑖
reduces to a point and the limit if it exists is called the surface
integral of 𝜑 over 𝑆 and is denoted by 𝜑
𝑑𝑆.

Note: If 𝑆 is piecewise smooth then the function 𝜑 𝑥, 𝑦, 𝑧 is continuous over 𝑆 and then the limit exists and is
independent of sub-divisions and choice of the point 𝑃𝑖 .

Flux: Suppose 𝑆 is a piecewise smooth surface so that the vector function 𝐹 defined over 𝑆 is
continuous over 𝑆. Let 𝑃 be any point of the surface 𝑆 and suppose 𝑛 is a unit vector at 𝑃 in the
direction of outward drawn normal to the surface 𝑆 at 𝑃. Then the component of 𝐹 along 𝑛 is 𝐹 . 𝑛 and
the integral of 𝐹 . 𝑛 over 𝑆 is called the surface integral of 𝐹 over 𝑆 and is denoted by 𝑆
𝐹 . 𝑛 𝑑𝑆. It is
also called flux of 𝐹 over 𝑆.

Different Forms of Surface Integral

(i) Flux of 𝐹 over 𝑆 = 𝑆


𝐹 . 𝑛 𝑑𝑆 … (1)

45
Now let 𝑑𝑆 denote a vector (called vector area) whose magnitude is that of differential of
surface area i.e., 𝑑𝑆 and whose direction is that of 𝑛 . Then clearly
𝑑𝑆 = 𝑛𝑑𝑆
From (1), flux of 𝐹 over 𝑆 = 𝑆
𝐹 . 𝑑𝑆 …. (2)
(ii) Suppose outward drawn normal to the surface 𝑆 at 𝑃 makes angles 𝛼, 𝛽, 𝛾 with the
positive direction of axes and if 𝑙, 𝑚, 𝑛 denote the direction cosines of this outward drawn
normal, then
𝑙 = cos 𝛼 , 𝑚 = cos 𝛽 , 𝑛 = cos 𝛾
Therefore, 𝑛 = cos 𝛼 𝑖 + cos 𝛽 𝑗 + cos 𝛾 𝑘
If 𝐹 = 𝐹1 𝑖 + 𝐹2 𝑗 + 𝐹3 𝑘 then 𝐹 . 𝑛 = 𝐹1 cos 𝛼 + 𝐹2 cos 𝛽 + 𝐹3 cos 𝛾

∴ From (1), flux of 𝐹 over 𝑆 = 𝑆


𝐹1 cos 𝛼 + 𝐹2 cos 𝛽 + 𝐹3 cos 𝛾 𝑑𝑆 … (3)
Now 𝑑𝑆 cos 𝛼 is the projection of area 𝑑𝑆 on the 𝑦𝑧 plane, therefore 𝑑𝑆 cos 𝛼 = 𝑑𝑦𝑑𝑧.
Similarly 𝑑𝑆 cos 𝛽 and 𝑑𝑆 cos 𝛾 are the projections of the area 𝑑𝑆 on the 𝑧𝑥 and 𝑥𝑦 plane
respectively and therefore 𝑑𝑆 cos 𝛽 = 𝑑𝑧𝑑𝑥, 𝑑𝑆 cos 𝛾 = 𝑑𝑥𝑑𝑦.
∴ From (3), 𝐹 . 𝑛 𝑑𝑆 = 𝐹1 𝑑𝑦𝑑𝑧 + 𝐹2 𝑑𝑧𝑑𝑥 + 𝐹3 𝑑𝑥𝑑𝑦 … (4)
𝑆 𝑆

Note: In order to evaluate surface integral it is convenient to express them as double integrals by
taking the projection of surface 𝑆 on one of the coordinate planes. This will happen only if any line
perpendicular to co-ordinate plane chosen meets the surface in one point and not more than one
point. Surface S is divided into sub surfaces, if above requirement is not met, so that sub surfaces
may satisfy the above requirement.

(iii) Suppose surface 𝑆 is such that any line


perpendicular to 𝑥𝑦 plane does not meet 𝑆 in more
than one point. Let the equation of surface 𝑆 be
𝑍 = 𝑕(𝑥, 𝑦).
Let 𝑅1 denotes the orthogonal projection of 𝑆 on the
𝑥𝑦 plane. Then projection of 𝑑𝑆 on the 𝑥𝑦 plane
= 𝑑𝑆 cos 𝛾 , where 𝛾 is the acute angle which the
normal to the surface 𝑆 makes with positive
direction of Z-axis.
∴ 𝑑𝑆 cos 𝛾 = 𝑑𝑥𝑑𝑦 … (5)
𝑛 .𝑘
But cos 𝛾 = = 𝑛. 𝑘
𝑛
𝑑𝑥 𝑑𝑦
Therefore from (5), 𝑑𝑆 = … (6)
𝑛 .𝑘

𝑑𝑥 𝑑𝑦
Thus 𝑆
𝐹 . 𝑛 𝑑𝑆 = 𝑅1
𝐹. 𝑛 … (7)
𝑛 .𝑘

𝑑𝑦 𝑑𝑧
Similarly we have, 𝑆
𝐹 . 𝑛 𝑑𝑆 = 𝑅2
𝐹. 𝑛 … (8)
𝑛 .𝑖

𝑑𝑧 𝑑𝑥
𝑆
𝐹 . 𝑛 𝑑𝑆 = 𝑅3
𝐹. 𝑛 … (9)
𝑛 .𝑗

where 𝑅2 , 𝑅3 are the projections of 𝑆 on 𝑧𝑥 and 𝑥𝑦 planes, respectively.


46
Example 55: Evaluate 𝑺
𝑭. 𝒏 𝒅𝑺 where 𝑭 = 𝒛 𝒊 + 𝒙 𝒋 + 𝟑𝒚𝟐 𝒛 𝒌 and S is the surface of the
cylinder 𝒙𝟐 + 𝒚𝟐 = 𝟏𝟔 included in the first octant between 𝒛 = 𝟎 and 𝒛 = 𝟓.

Solution: A vector normal to the surface 𝑆 is given by

𝑛 = ∇ 𝑥 2 + 𝑦 2 = 2𝑥 𝑖 + 2𝑦 𝑗 … (1)
2𝑥𝑖 +2𝑦𝑗
∴ 𝑛 = unit vector normal to surface 𝑆 at any point 𝑥, 𝑦, 𝑧 = … (2)
4𝑥 2 +4𝑦 2

2𝑥𝑖 +2𝑦𝑗 2𝑥𝑖 +2𝑦𝑗 𝑥 𝑦


∵ 𝑥 2 + 𝑦 2 = 16, therefore 𝑛= = = 4𝑖+4𝑗 … (3)
4(𝑥 2 +𝑦 2 ) 8

𝑑𝑥 𝑑𝑧
Now 𝑆
𝐹 . 𝑛 𝑑𝑆 = 𝑅
𝐹. 𝑛 𝑛 .𝑗

(projection on 𝑥𝑦 plane can’t be taken as the surface 𝑠 is peerpendicular to 𝑥𝑦 plane)


𝑥𝑧 𝑥𝑦 𝑑𝑥 𝑑𝑧 𝑥𝑧 𝑦
= 𝑅
+ 𝑦 = 𝑅
+ 𝑥 𝑑𝑥 𝑑𝑧 , since from 3 , 𝑛. 𝑗 =
4 4 𝑦 4
4

−𝑧
5 4 𝑥𝑧 5 4 (−2𝑥)
2
= 2
+ 𝑥 𝑑𝑥 𝑑𝑧 = + 𝑥 𝑑𝑥 𝑑𝑧
𝑧=0 𝑥=0 16−𝑥 𝑧=0 𝑥=0 16−𝑥 2

4 5
5 𝑧 16−𝑥 2 𝑥2 5 4𝑧 2
= 𝑧=0
−2 + 𝑑𝑧 = 𝑧=0
(4𝑧 + 8)𝑑𝑧 = + 8𝑧 = 90
1 2 2 2 𝑧=0
𝑥=0

Example 56: Evaluate 𝑺


𝑭. 𝒏 𝒅𝑺 where 𝑭 = 𝟔𝒛 𝒊 − 𝟒 𝒋 + 𝒚 𝒌 and S is the portion of the plane
𝟐𝒙 + 𝟑𝒚 + 𝟔𝒛 = 𝟏𝟐 in the first octant.

Solution: Vector normal to surface 𝑆 is given by

∇ 2𝑥 + 3𝑦 + 6𝑧 = 2𝑖 + 3𝑗 + 6𝑘
2𝑖 +3𝑗 +6𝑘 2 3 6
∴ 𝑛 = unit vector normal to surface 𝑆 at any point (𝑥, 𝑦, 𝑧) = = 7𝑖 +7𝑗 +7𝑘
4+9+36

2 3 6 12 12 6
Now 𝐹 . 𝑛 = 6𝑧 𝑖 − 4 𝑗 + 𝑦 𝑘 . 𝑖+7𝑗 +7𝑘 = 𝑧− +7𝑦
7 7 7

Taking projection on 𝑥𝑦 plane


𝑑𝑥 𝑑𝑦 𝑑𝑥 𝑑𝑦
𝑆
𝐹 . 𝑛 𝑑𝑆 = 𝑅
𝐹. 𝑛 = 𝑅
𝐹. 𝑛 … (1)
𝑛 .𝑘 6 7

where 𝑅 is the region of projection of 𝑆 on 𝑥𝑦 plane. 𝑅 is bounded by x-axis, y-axis and the line
2𝑥 + 3𝑦 = 12, 𝑧 = 0. In order to evaluate double integral in (1), y varies from 0 to 4 and x varies
12−3𝑦
from 0 to . Therefore from (1)
2

12 −3𝑦
4
𝑆
𝐹 . 𝑛 𝑑𝑆 = 2
𝑦=0 𝑥=0
2𝑧 − 2 + 𝑦 𝑑𝑥 𝑑𝑦, Find 𝑧 from 2𝑥 + 3𝑦 + 6𝑧 = 12

12 −3𝑦
4 𝑥 𝑦
= 2
𝑦=0 𝑥=0
2 2 − 3 − 2 − 2 + 𝑦 𝑑𝑥 𝑑𝑦

47
12 −3𝑦
12 −3𝑦
4 2𝑥 4 𝑥2 2
= 2
𝑦=0 𝑥=0
2− 𝑑𝑥 𝑑𝑦 = 0
2𝑥 − 𝑑𝑦
3 3 𝑥=0

4 12−3𝑦 1 12−3𝑦 2
= 0
2× −3 𝑑𝑦
2 2

4
12−3𝑦 2 12−3𝑦 3 144 1728
= + = − = 24 − 16 = 8
−6 108 𝑦=0 6 108

𝟑
Example 57: 𝑺
𝝋 𝒏 𝒅𝑺 where 𝝋 = 𝟖 𝒙𝒚𝒛 and 𝑺 is the surface of cylinder 𝒙𝟐 + 𝒚𝟐 = 𝟏𝟔
included in the first octant between 𝒛 = 𝟎 to 𝒛 = 𝟓.

Solution: A vector normal to the surface 𝑆 is given by

𝑛 = ∇ 𝑥 2 + 𝑦 2 = 2𝑥𝑖 + 2𝑦𝑗
2𝑥𝑖 +2𝑦𝑗
∴ 𝑛 = unit vector normal to surface 𝑆 at any point (𝑥, 𝑦, 𝑧) =
4𝑥 2 +4𝑦 2

2𝑥𝑖 +2𝑦𝑗 2𝑥𝑖 +2𝑦𝑗 𝑥 𝑦


𝑛= = = 4𝑖+4𝑗 ∵ 𝑥 2 + 𝑦 2 = 16
4(𝑥 2 +𝑦 2 ) 8

3 𝑥 𝑦 𝑑𝑥 𝑑𝑧
Now 𝑆
𝜑 𝑛 𝑑𝑆 = 𝑅 8
𝑥𝑦𝑧 𝑖+4𝑗 …(1)
4 𝑛 .𝑗

where 𝑅 is the region of projection of 𝑆 on 𝑧𝑥 plane. Therefore, from (1)

5 4 3 2 3 𝑑𝑥 𝑑𝑧
𝑆
𝜑 𝑛 𝑑𝑆 = 𝑧=0 𝑥=0 32
𝑥 𝑦𝑧 𝑖 + 32 𝑥𝑦 2 𝑧 𝑗 , where 𝑦 2 = 16 − 𝑥 2
𝑦 4
5 4 3 2 3
= 𝑧=0 𝑥=0 8
𝑥 𝑧 𝑖 + 8 𝑥𝑧 16 − 𝑥 2 𝑗 𝑑𝑥 𝑑𝑧
3 2 4
3 5 𝑥3𝑧 𝑧 16−𝑥 2
= 𝑖−2 𝑗 𝑑𝑍
8 𝑧=0 3 3 2
𝑥=0
5
3 5 64 64 𝑧2 𝑧2
=8 𝑧=0 3
𝑧 𝑖+ 𝑧 𝑗 𝑑𝑧 = 8 𝑖+ 𝑗 = 100 𝑖 + 100 𝑗
3 2 2 𝑧=0

Example 58: Evaluate 𝑺


𝑭. 𝒅𝑺 where 𝑭 = 𝒙𝒊 − 𝒛𝟐 − 𝒛𝒙 𝒋 − 𝒙𝒚𝒌 and 𝑺 is the triangular
surface with vertices 𝟐, 𝟎, 𝟎 , (𝟎, 𝟐, 𝟎) and (𝟎, 𝟎, 𝟒).

Solution:The triangular surface 𝑆 with vertices 2,0,0 , (0,2,0), and (0,0,4) is given by the equation
𝑥 𝑦 𝑧
+2+4=1 ⇒ 2𝑥 + 2𝑦 + 𝑧 = 4 … (1)
2

Vector normal to surface 𝑆 is given by ∇ 2𝑥 + 2𝑦 + 𝑧 = 2𝑖 + 2𝑗 + 𝑘

2𝑖 +2𝑗 +𝑘 2 2 1
∴ 𝑛 = unit vector normal to surface 𝑆 at any point (𝑥, 𝑦, 𝑧) = = 3𝑖+3𝑗 +3𝑘
4+4+1

2 2 1 2 2 1
Now 𝐹 . 𝑛 = 𝑥𝑖 − 𝑧 2 − 𝑧𝑥 𝑗 − 𝑥𝑦𝑘 . 𝑖 + 3 𝑗 + 3 𝑘 = 3 𝑥 − 3 𝑧 2 − 𝑧𝑥 − 3 𝑥𝑦
3

Taking projection on 𝑥𝑦 plane

48
𝑑𝑥 𝑑𝑦 𝑑𝑥 𝑑𝑦
𝑆
𝐹 . 𝑛 𝑑𝑆 = 𝑅
𝐹. 𝑛 = 𝑅
𝐹. 𝑛 … (2)
𝑛 .𝑘 1 3

where 𝑅 is the region of projection of 𝑆 on 𝑥𝑦 plane. 𝑅 is bounded by x-axis, y-axis and the line
2𝑥 + 2𝑦 = 4 i.e., 𝑥 + 𝑦 = 2, 𝑧 = 0. In order to integrate double integral in (2), y varies from 0 to 2
and x varies from 0 to 2 − 𝑦. Therefore from (2)
2 2−𝑦 2 2 1
𝑆
𝐹 . 𝑛 𝑑𝑆 = 𝑦=0 𝑥=0 3
𝑥 − 3 𝑧 2 − 𝑧𝑥 − 3 𝑥𝑦 𝑑𝑥 𝑑𝑦 Find 𝑧 from 2𝑥 + 2𝑦 + 𝑧 = 4

2 2−𝑦 2 2 2 1
= 𝑦=0 𝑥=0 3
𝑥 − 3 4 − 2𝑥 − 2𝑦 − 4 − 2𝑥 − 2𝑦 𝑥 − 3 𝑥𝑦 𝑑𝑥 𝑑𝑦

1 2 2−𝑦
=3 𝑦=0 𝑥=0
2𝑥 − 2 16 + 4𝑥 2 + 4𝑦 2 − 16𝑥 + 8𝑥𝑦 − 16𝑦 − 4𝑥 + 2𝑥 2 + 2𝑥𝑦 − 𝑥𝑦 𝑑𝑥 𝑑𝑦

1 2 2−𝑦
=3 𝑦=0 𝑥=0
−12𝑥 2 − 8𝑦 2 + 42𝑥 + 32𝑦 − 21𝑥𝑦 − 32 𝑑𝑥 𝑑𝑦

2−𝑦
1 2 𝑥2𝑦
=3 𝑦=0
−4𝑥 3 − 8𝑥𝑦 2 + 21𝑥 2 + 32𝑥𝑦 − 21 − 32𝑥 𝑑𝑦
2 𝑥=0

1 2 3 2−𝑦 2 𝑦
=3 𝑦=0
−4 2 − 𝑦 − 8 2 − 𝑦 𝑦 2 + 21 2 − 𝑦 2
+ 32 2 − 𝑦 𝑦 − 21 − 32 2 − 𝑦 𝑑𝑦
2

1 2 3 3
=3 𝑦=0 2
𝑦 + 9𝑦 2 + 18𝑦 − 12 𝑑𝑦 = 38

Example 59: Evalutate 𝑺


𝒇 ∙ 𝒏 𝒅𝒔 where 𝒇 = 𝟐𝒙𝟐 𝒚 𝒊 − 𝒚𝟐 𝒋 + 𝟒𝒙𝒛𝟐 𝒌 and S is closed surface of
the region in the first octant bounded by the cylinder 𝒚𝟐 + 𝒛𝟐 = 𝟗 and the planes 𝒙 = 𝟎, 𝒙 = 𝟐,
𝒚 = 𝟎 𝐚𝐧𝐝 𝒛 = 𝟎.

Solution: The given closed surface S is piecewise smooth and is


comprised of S1 –the rectangular face OAEB in xy-plane; S2 –the
rectangular face OADC in xz-plane; S3 – the circular quadrant ABC
in yz-plane; S4 – the circular quadrant AED and S5 – the curved
surface BCDE of the cylinder in the first octant (see Fig. 17.16).

∴ 𝑆
𝑓 ∙ 𝑛 𝑑𝑠 = 𝑆1
𝑓 ∙ 𝑛 𝑑𝑠 + 𝑆2
𝑓 ∙ 𝑛 𝑑𝑠 + 𝑆3
𝑓 ∙ 𝑛 𝑑𝑠

+ 𝑆4
𝑓 ∙ 𝑛 𝑑𝑠 + 𝑆5
𝑓 ∙ 𝑛 𝑑𝑠 … (1)

Now 𝑆1
𝑓 ∙ 𝑛 𝑑𝑠 = 𝑆1
2𝑥 2 𝑦 𝑖 − 𝑦 2 𝑗 + 4𝑥𝑧 2 𝑘 ∙ −𝑘 𝑑𝑠 = −4 𝑆1
𝑥𝑧 2 𝑑𝑠 = 0
(𝑎𝑠 𝑧 = 0 𝑖𝑛 𝑥𝑦 − 𝑝𝑙𝑎𝑛𝑒)
Similarly, 𝑆2
𝑓 ∙ 𝑛 𝑑𝑠 = 0 and 𝑆3
𝑓 ∙ 𝑛 𝑑𝑠

𝑆4
𝑓 ∙ 𝑛 𝑑𝑠 = 𝑆4
2𝑥 2 𝑦 𝑖 − 𝑦 2 𝑗 + 4𝑥𝑧 2 𝑘 ∙ 𝑖 𝑑𝑠 = 𝑆4
2𝑥 2 𝑦 𝑑𝑠

3 9−𝑧 2 3
= 0 0
8𝑦 𝑑𝑦𝑑𝑧 = 4 0
(9 − 𝑧 2 )𝑑𝑧 = 72

To find 𝑛 in S5 , we note that ∇ 𝑦 2 + 𝑧 2 − 9 = 2𝑦 𝑗 + 2𝑧𝑘 ,


2𝑦 𝑗 +2𝑧𝑘 𝑦 𝑗 +𝑧𝑘 𝑧
Implying 𝑛 = = and 𝑛 ∙ 𝑘 = 3 so that 𝑑𝑠 = 𝑑𝑥𝑑𝑦/(𝑧/3) (𝑎𝑠 𝑦 2 + 𝑧 2 = 9)
4 𝑦 2 +𝑧 2 3

49
2 3 −𝑦 3 +4𝑥𝑧 3 2 3 −𝑦 3
𝑆5
𝑓 ∙ 𝑛 𝑑𝑠 = 0 0
𝑑𝑥𝑑𝑦/(𝑧/3) = 0 0
+ 4𝑥𝑧 2 𝑑𝑥𝑑𝑦
3 𝑧

Now putting 𝑦 = 3 sin 𝜃 , 𝑧 = 3 cos 𝜃 ∴ 𝑑𝑦 = 3 cos 𝜃 𝑑𝜃


𝜋
2 −27 sin 3 𝜃
0 0
2 + 4𝑥 (9 cos2 𝜃) 3 cos 𝜃 𝑑𝜃𝑑𝑥
3 cos 𝜃

ASSIGNMENT 7

1. If velocity vector is 𝐹 = 𝑦 𝑖 + 2 𝑗 + 𝑥𝑧 𝑘 m/sec., show that the flux of water through the
parabolic cylinder 𝑦 = 𝑥 2 , 0 ≤ 𝑥 ≤ 3, 0 ≤ 𝑧 ≤ 2 is 69 𝑚3 /𝑠𝑒𝑐.

2. Evaluate 𝑆
𝐹 . 𝑛 𝑑𝑆 where 𝐹 = 𝑥 + 𝑦 2 𝑖 − 2𝑥 𝑗 + 2𝑦𝑧 𝑘 and S is the surface of the plane
2𝑥 + 𝑦 + 2𝑧 = 6 in the first octant.

3. If 𝐹 = 4𝑥𝑧 𝑖 − 𝑦 2 𝑗 + 𝑦𝑧 𝑘; evaluate 𝑆
𝐹 . 𝑛 𝑑𝑆, where S is the surface of the cube bounded by
𝑥 = 0, 𝑥 = 1, 𝑦 = 0, 𝑦 = 1, 𝑧 = 0, 𝑧 = 1.
4. If 𝐹 = 2𝑦 𝑖 − 3 𝑗 + 𝑥 2 𝑘 and S is the surface of the parabolic cylinder 𝑦 2 = 8𝑥 in the first

octant bounded by the planes 𝑦 = 4 and 𝑧 = 6, show that 𝑆


𝐹 ∙ 𝑛 𝑑𝑠 = 132.

5. Evaluate 𝑆
𝐹 ∙ 𝑛 𝑑𝑠 where 𝐹 = 6𝑧 𝑖 − 4 𝑗 + 𝑦 𝑘 and S is the portion of the plane
2𝑥 + 3𝑦 + 6𝑧 = 12 in the first octant.

17.12 VOLUME INTEGRALS

Suppose 𝑉 is the volume bounded by a surface 𝑆. Divide the volume 𝑉 into sub-volumes 𝛿𝑉1 , 𝛿𝑉2 ,
…, 𝛿𝑉𝑛 . In each 𝛿𝑉𝑖 , choose an arbitrary point 𝑃𝑖 whose coordinates are (𝑥𝑖 , 𝑦𝑖 , 𝑧𝑖 ). Let 𝜑 be a single
valued function defined over 𝑉. Form the sum 𝜑 𝑃𝑖 𝛿𝑉𝑖 , where 𝜑 𝑃𝑖 = 𝜑 𝑥𝑖 , 𝑦𝑖 , 𝑧𝑖 .

Now let us take the limit of the sum as 𝑛 → ∞, then the limit, if exists, is called the volume integral
of 𝜑 over 𝑉 and is denoted as 𝑉
𝜑 𝑑𝑉.

Likewise if 𝐹 is a vector point function defined in the given region of volume 𝑉 then vector volume
integral of 𝐹 over 𝑉 is 𝑉
𝐹 𝑑𝑉.

Note: Above volume integral becomes 𝑉


𝜑 𝑑𝑥 𝑑𝑦 𝑑𝑧 if we subdivide the volume 𝑉 into small
cuboids by drawing lines parallel to three co-ordinate axes because in that case 𝑑𝑉 = 𝑑𝑥𝑑𝑦𝑑𝑧.

Example 60: If 𝑭 = 𝟐𝒙𝟐 − 𝟑𝒛 𝒊 − 𝟐𝒙𝒚𝒋 − 𝟒𝒙𝒌, evualuate 𝑽


𝛁 × 𝑭 𝒅𝑽 where 𝑽 is the region
bounded by the co-ordinate planes and the plane 𝟐𝒙 + 𝟐𝒚 + 𝒛 = 𝟒.

Solution: Consider

50
𝑖 𝑗 𝑘
𝜕 𝜕 𝜕
∇×𝐹 = 𝜕𝑥 𝜕𝑦 𝜕𝑧
= 0 𝑖 + 𝑗 − 2𝑦 𝑘
2
2𝑥 − 3𝑧 −2𝑥𝑦 −4𝑥

Region bounded by 2𝑥 + 2𝑦 + 𝑧 = 4 and coordinate planes such that

2𝑥 ≤ 4, 2𝑥 + 2𝑦 ≤ 4, 2𝑥 + 2𝑦 + 𝑧 ≤ 4

i.e. 𝑥 ≤ 2, 𝑦 ≤ 2 − 𝑥, 𝑧 ≤ 4 − 2𝑥 − 2𝑦
2 2−𝑥 4−2𝑥−2𝑦
∴ 𝑉
∇ × 𝐹 𝑑𝑉 = 𝑥=0 𝑦=0 𝑧=0
𝑗 − 2𝑦 𝑘 𝑑𝑧 𝑑𝑦 𝑑𝑥

2 2−𝑥 𝑧=4−2𝑥−2𝑦
= 𝑥=0 𝑦=0
𝑧 𝑗 − 2𝑦𝑧 𝑘 𝑧=0
𝑑𝑦 𝑑𝑥

2 2−𝑥
= 𝑥=0 𝑦=0
(4 − 2𝑥 − 2𝑦) 𝑗 − 2𝑦(4 − 2𝑥 − 2𝑦) 𝑘 𝑑𝑦 𝑑𝑥

2 4 𝑦=2−𝑥
= 𝑥=0
4𝑦 − 2𝑥𝑦 − 𝑦 2 𝑗 − 4𝑦 2 − 2𝑥𝑦 2 − 3 𝑦 3 𝑘 𝑑𝑥
𝑦=0

2
2
8 − 4𝑥 − 2𝑥 2 − 𝑥 − 2 − 𝑥 𝑗 −
= 4 𝑑𝑥
𝑥=0 4(2 − 𝑥)2 − 2𝑥(2 − 𝑥)2 − 3 (2 − 𝑥)3 𝑘

2 2 16
= 𝑥=0
4 − 4𝑥 + 𝑥 2 𝑗 − − 3 𝑥 3 + 4𝑥 2 − 8𝑥 + 𝑘 𝑑𝑥
3

𝑥=2 𝑥=2
𝑥3 1 4 16
= 4𝑥 − 2𝑥 2 + 𝑗 − − 6 𝑥 4 + 3 𝑥 3 − 4𝑥 2 + 𝑥 𝑘
3 𝑥=0 3 𝑥=0

8 8 8
= 3 𝑗 − 3 𝑘 = 3 (𝑗 − 𝑘 )

ASSIGNMENT 8

1. Evaluate 𝑉
φ 𝑑𝑉 where 𝜑 = 45𝑥 2 𝑦 and 𝑉 is the region bounded by the planes 4𝑥 + 2𝑦 + 𝑧 =
8, 𝑥 = 0, 𝑦 = 0, 𝑧 = 0.

2. If 𝐹 = 2𝑥𝑧 𝑖 − 𝑥 𝑗 + 𝑦 2 𝑘 ; evaluate 𝑉
F 𝑑𝑉 where 𝑉 is the region bounded by the planes
𝑥 = 0, 𝑦 = 0, 𝑥 = 2, 𝑦 = 6, 𝑧 = 𝑥 2 , 𝑧 = 4.

17.13 STOKE’S THEOREM (Relation between Line and Surface Integral)

Statement: Let S be a piecewise smooth open surface bounded by a piecewise smooth simple curve
𝐶 . If 𝑓 (𝑥, 𝑦, 𝑧) be a continuous vector function which has continuous first partial derivative in a
region of space which contains 𝑆 , then 𝐶
𝑓. 𝑑𝑟 = 𝐶
𝑐𝑢𝑟𝑙 𝑓. 𝑛 𝑑𝑆 , where 𝑛 is the unit normal
vector at any point of 𝑆 and 𝐶 is trasversed in positive direction.

Direction of 𝐶 is positive if an observer walking on the boundary of 𝑆 in this direction with its head
pointing in the direction of outward normal 𝑛 to 𝑆 has the surface on the left.

We may put the statement of Stoke’s theorem in words as under:


51
The line integral of the tangential component of a vector 𝑓 taken around a simple closed curve 𝐶 is
equal to the surface integral of normal component of curl of 𝑓 taken over 𝑆 having 𝐶 as its boundary.

Stoke’s Theorem in Cartesian Form:

a) Cartesian Form of Stoke’s Theorem in Plane (or Green’s Theorem in Plane)

Choose system of coordinate axes such that the plane of the surface is in 𝑥𝑦 plane and normal to the
surface 𝑆 lies along the z-axis. Normal vector is constant in this case.

Suppose 𝑓 = 𝑓1 𝑖 + 𝑓2 𝑗 + 𝑓3 𝑘
𝑑𝑟 𝑑𝑟
∴ 𝐶
𝑓. 𝑑𝑟 = 𝐶
𝑓 . 𝑑𝑠 𝑑𝑠 = 𝐶
𝑓 . 𝑡 𝑑𝑠 where 𝑡 = is unit vector tangent to 𝐶.
𝑑𝑠

𝑑𝑥 𝑑𝑦 𝑑𝑧
∴ 𝐶
𝑓. 𝑑𝑟 = 𝐶
𝑓1 𝑖 + 𝑓2 𝑗 + 𝑓3 𝑘 . 𝑖 +𝑗 +𝑘 𝑑𝑠
𝑑𝑠 𝑑𝑠 𝑑𝑠

𝑑𝑥 𝑑𝑦 𝑑𝑧
= 𝐶
𝑓1 + 𝑓2 + 𝑓3 𝑑𝑠
𝑑𝑠 𝑑𝑠 𝑑𝑠

𝑑𝑧
But tangent at any point lies in the 𝑥𝑦 plane, so 𝑑𝑠 = 0

𝑑𝑥 𝑑𝑦
∴ 𝐶
𝑓. 𝑑𝑟 = 𝐶
𝑓1 + 𝑓2 𝑑𝑠 … (1)
𝑑𝑠 𝑑𝑠

Now 𝑆
𝑐𝑢𝑟𝑙 𝑓 . 𝑛 𝑑𝑆 = 𝑆
𝑐𝑢𝑟𝑙 𝑓 . 𝑘 𝑑𝑆 (Here normal is along Z-axis)

𝜕𝑓2 𝜕𝑓1
= 𝑆
− 𝑑𝑥𝑑𝑦 … (2)
𝜕𝑥 𝜕𝑦

Using (1) and (2), Stoke’s theorem is


𝜕𝑓2 𝜕𝑓1
𝐶
𝑓1 𝑑𝑥 + 𝑓2 𝑑𝑦 = 𝑆
− 𝑑𝑥𝑑𝑦
𝜕𝑥 𝜕𝑦

b) Cartesian Form of Stoke’s Theorem in Space

Suppose 𝑓 = 𝑓1 𝑖 + 𝑓2 𝑗 + 𝑓3 𝑘 and 𝑛 is an outward drawn normal unit vector of 𝑆 making angles 𝛼,


𝛽, 𝛾 with positive direction of axes.

∴ 𝑛 = cos 𝛼 𝑖 + cos 𝛽 𝑗 + cos 𝛾 𝑘

𝑖 𝑗 𝑘
𝜕 𝜕 𝜕
Now, ∇×𝑓 =
𝜕𝑥 𝜕𝑦 𝜕𝑧
𝑓1 𝑓2 𝑓3
𝜕𝑓3 𝜕𝑓2 𝜕𝑓1 𝜕𝑓3 𝜕𝑓2 𝜕𝑓1
𝑖. 𝑒. 𝑐𝑢𝑟𝑙 𝑓 = − 𝑖+ − 𝑗+ − 𝑘
𝜕𝑦 𝜕𝑧 𝜕𝑧 𝜕𝑥 𝜕𝑥 𝜕𝑦

𝜕𝑓3 𝜕𝑓2 𝜕𝑓1 𝜕𝑓3 𝜕𝑓2 𝜕𝑓1


∴ 𝑐𝑢𝑟𝑙 𝑓 . 𝑛 = − cos 𝛼 + − cos 𝛽 + − cos 𝛾 … (1)
𝜕𝑦 𝜕𝑧 𝜕𝑧 𝜕𝑥 𝜕𝑥 𝜕𝑦

Also 𝑓. 𝑑𝑟 = 𝑓1 𝑖 + 𝑓2 𝑗 + 𝑓3 𝑘 . 𝑑𝑥 𝑖 + 𝑑𝑦 𝑗 + 𝑑𝑧 𝑘

52
or 𝑓 . 𝑑𝑟 = 𝑓1 𝑑𝑥 + 𝑓2 𝑑𝑦 + 𝑓3 𝑑𝑧

Then Stoke’s theorem is


𝜕𝑓3 𝜕𝑓2 𝜕𝑓1 𝜕𝑓3 𝜕𝑓2 𝜕𝑓1
𝐶
𝑓1 𝑑𝑥 + 𝑓2 𝑑𝑦 + 𝑓3 𝑑𝑧 = 𝑆
− cos 𝛼 + − cos 𝛽 + − cos 𝛾 𝑑𝑆
𝜕𝑦 𝜕𝑧 𝜕𝑧 𝜕𝑥 𝜕𝑥 𝜕𝑦

Example 61: Verify Stoke’s theorem for 𝒇 = 𝟐𝒙 − 𝒚 𝒊 − 𝒚𝒛𝟐 𝒋 − 𝒚𝟐 𝒛𝒌 when 𝑺 is the upper half
of the surface of the sphere 𝒙𝟐 + 𝒚𝟐 + 𝒛𝟐 = 𝟏 and 𝑪 is its boundary.

Solution: The boundary 𝐶 of the upper half of the sphere 𝑆 is circle in the 𝑥𝑦 plane. Therefore,
parametric equations of 𝐶 are 𝑥 = cos 𝑡 , 𝑦 = sin 𝑡 , 𝑧 = 0 when 0 ≤ t ≤ 2π

Now, 𝐶
𝑓. 𝑑𝑟 = 𝐶
𝑓1 𝑑𝑥 + 𝑓2 𝑑𝑦 + 𝑓3 𝑑𝑧

= 𝐶
2𝑥 − 𝑦 𝑑𝑥 − 𝑦𝑧 2 𝑑𝑦 − 𝑦 2 𝑧 𝑑𝑧 = 𝐶
2 cos 𝑡 − sin 𝑡 (− sin 𝑡) 𝑑𝑡

∵ 𝑥 = cos 𝑡 , ∴ 𝑑𝑥 = − sin 𝑡 𝑑𝑡 and other terms of integrand become zero as 𝑧 = 0


2𝜋
2𝜋 cos 2 𝑡 2𝜋
= 𝑡=0
2 cos 𝑡 − sin 𝑡 + sin2 𝑡 𝑑𝑡 = 2 + 𝑡=0
sin2 𝑡 𝑑𝑡
2 𝑡=0

𝜋 2
= 1−1 +4 𝑡=0
sin2 𝑡 𝑑𝑡 (Property of definite integral)

1 𝜋
= 0 + 4. 2 . 2 = 𝜋 … (1)

𝑖 𝑗 𝑘
𝜕 𝜕 𝜕
Now, 𝑐𝑢𝑟𝑙 𝑓 = 𝜕𝑥 𝜕𝑦 𝜕𝑧
= −2𝑦𝑧 + 2𝑦𝑧 𝑖 + 0 − 0 𝑗 + 0 + 1 𝑘 = 𝑘
2𝑥 − 𝑦 −𝑦𝑧 2 2
−𝑦 𝑧

Now, 𝑐𝑢𝑟𝑙 𝑓 . 𝑛 𝑑𝑆 = 𝑆
𝑘 . 𝑛 𝑑𝑆

𝑑𝑥 𝑑𝑦
= 𝑅
𝑘.𝑛 [where 𝑅 is the projection of S on 𝑥𝑦 − plane]
𝑛 .𝑘

= 𝑅
𝑑𝑥 𝑑𝑦

Now projection of 𝑆 on 𝑥𝑦 plane is circle 𝑥 2 + 𝑦 2 = 1.

1 1−𝑥 2 1 1−𝑥 2
= 𝑥=−1 𝑦=− 1−𝑥 2
𝑑𝑥 𝑑𝑦 = 4 𝑥=0 𝑦=0
𝑑𝑥 𝑑𝑦 [By definite integral]

1
1 𝑥 1−𝑥 2 1
=4 𝑥=0
1 − 𝑥 2 𝑑𝑥 = 4 + 2 sin−1 𝑥
2
𝑥=0

1 𝜋
=4 sin−1 1 = 4 × 4 = 𝜋 … (2)
2

From (1) and (2), Stoke’s theorem is verified.

Example 62: Verify Stoke’s theorem for the function 𝒇 = 𝒙𝟐 + 𝒚𝟐 𝒊 − 𝟐𝒙𝒚 𝒋 taken round the
rectangle bounded by 𝒙 = ±𝒂, 𝒚 = 𝟎, 𝒚 = 𝒃. [KUK 2006]
53
Solution: Given 𝑓 = 𝑥 2 + 𝑦 2 𝑖 − 2𝑥𝑦 𝑗

Therefore, 𝑓. 𝑑𝑟 = 𝑥 2 + 𝑦 2 𝑖 − 2𝑥𝑦 𝑗 . 𝑑𝑥 𝑖 + 𝑑𝑦 𝑗 = 𝑥 2 + 𝑦 2 𝑑𝑥 − 2𝑥𝑦 𝑑𝑦

Y
𝑦=𝑏
D C

𝑥 = −𝑎 𝑥=𝑎
𝑦=0
X
A O B
Fig. 17.17

∴ 𝐶
𝑓 . 𝑑𝑟 = 𝐶
𝑥 2 + 𝑦 2 𝑑𝑥 − 2𝑥𝑦 𝑑𝑦

= 𝐷𝐴
𝑥 2 + 𝑦 2 𝑑𝑥 − 2𝑥𝑦 𝑑𝑦 + 𝐴𝐵
𝑥 2 + 𝑦 2 𝑑𝑥 − 2𝑥𝑦 𝑑𝑦

+ 𝐵𝐶
𝑥 2 + 𝑦 2 𝑑𝑥 − 2𝑥𝑦 𝑑𝑦 + 𝐶𝐷
𝑥 2 + 𝑦 2 𝑑𝑥 − 2𝑥𝑦 𝑑𝑦 … (1)

On DA: 𝑥 = −𝑎, ∴ 𝑑𝑥 = 0
0 0
∴ 𝐷𝐴
𝑥 2 + 𝑦 2 𝑑𝑥 − 2𝑥𝑦 𝑑𝑦 = 𝐷𝐴
−2 −𝑎 𝑦 𝑑𝑦 = 𝑦=𝑏
2𝑎𝑦 𝑑𝑦 = 𝑎𝑦 2 𝑏 = −𝑎𝑏 2

On AB: 𝑦 = 0, ∴ 𝑑𝑦 = 0
𝑎 2
∴ 𝐴𝐵
𝑥 2 + 𝑦 2 𝑑𝑥 − 2𝑥𝑦 𝑑𝑦 = 𝐴𝐵
𝑥 2 𝑑𝑥 = −𝑎
𝑥 2 𝑑𝑥 = 3 𝑎3

On BC: 𝑥 = 𝑎, ∴ 𝑑𝑥 = 0
𝑏
∴ 𝐵𝐶
𝑥 2 + 𝑦 2 𝑑𝑥 − 2𝑥𝑦 𝑑𝑦 = 𝐵𝐶
−2𝑎𝑦 𝑑𝑦 = 𝑦=0
−2𝑎𝑦 𝑑𝑦 = −𝑎𝑏 2

On CD: 𝑦 = 𝑏, ∴ 𝑑𝑦 = 0
−𝑎
∴ 𝐶𝐷
𝑥 2 + 𝑦 2 𝑑𝑥 − 2𝑥𝑦 𝑑𝑦 = 𝐶𝐷
𝑥 2 + 𝑏 2 𝑑𝑥 = 𝑥=𝑎
𝑥 2 + 𝑏 2 𝑑𝑥
−𝑎
𝑥3 𝑎3
= + 𝑏2 𝑥 = −2 + 𝑏2 𝑎
3 𝑥=𝑎
3

Substituting these values in (1), we get


2 𝑎3
𝐶
𝑓. 𝑑𝑟 = −𝑎𝑏 2 + 3 𝑎3 − 𝑎𝑏 2 − 2 + 𝑏 2 𝑎 = −4𝑎𝑏 2 … (2)
3

𝑖 𝑗 𝑘
𝜕 𝜕 𝜕
Now, 𝑐𝑢𝑟𝑙 𝑓 = 𝜕𝑥 𝜕𝑦 𝜕𝑧
= 0 𝑖 + 0 𝑗 + −2𝑦 − 2𝑦 𝑘 = −4𝑦 𝑘
𝑥2 + 𝑦2 −2𝑥𝑦 0

Since Surface lies in xy-plane, therefore 𝑛 = 𝑘.

54
𝑏 𝑎
∴ 𝑆
𝑐𝑢𝑟𝑙 𝑓 . 𝑛 𝑑𝑆 = 𝑆
−4𝑦 𝑘 . 𝑘 𝑑𝑆 = 𝑦=0 𝑥=−𝑎
−4𝑦 𝑑𝑦 𝑑𝑥 = −4𝑎𝑏 2 … (3)

Hence from (2) and (3), theorem is verified.

Example 63: Evaluate by Stoke’s theorem 𝑪


𝒚𝒛 𝒅𝒙 + 𝒙𝒛 𝒅𝒚 + 𝒙𝒚𝒅𝒛 , where 𝑪 is the curve
𝒙𝟐 + 𝒚𝟐 = 𝟏, 𝒛 = 𝒚𝟐 .

Solution: 𝐶
𝑦𝑧 𝑑𝑥 + 𝑥𝑧 𝑑𝑦 + 𝑥𝑦𝑑𝑧 = 𝐶
𝑦𝑧 𝑖 + 𝑥𝑧 𝑗 + 𝑥𝑦 𝑘 . 𝑑𝑥 𝑖 + 𝑑𝑦 𝑗 + 𝑑𝑧 𝑘

= 𝐶
𝑓 . 𝑑𝑟 , where 𝑓 = 𝑦𝑧 𝑖 + 𝑥𝑧 𝑗 + 𝑥𝑦 𝑘

𝑖 𝑗 𝑘
𝜕 𝜕 𝜕
Now, 𝑐𝑢𝑟𝑙 𝑓 = = 𝑥 − 𝑥 𝑖 + (𝑦 − 𝑦) 𝑗 + 𝑧 − 𝑧 𝑘 = 0
𝜕𝑥 𝜕𝑦 𝜕𝑧
𝑦𝑧 𝑧𝑥 𝑥𝑦

∴ 𝐶
𝑓 . 𝑑𝑟 = 𝑆
𝑐𝑢𝑟𝑙 𝑓. 𝑛 𝑑𝑆 = 0 ∵ 𝑐𝑢𝑟𝑙 𝑓 = 0

Example 64: Evaluate 𝑪


𝒇 . 𝒅𝒓 by Stoke’s theorem, where 𝒇 = 𝒚𝟐 𝒊 + 𝒙𝟐 𝒋 − 𝒙 + 𝒛 𝒌 and 𝑪 is
the boundary of triangle with vertices at (𝟎, 𝟎, 𝟎), (𝟏, 𝟎, 𝟎), (𝟏, 𝟏, 𝟎).

Solution: Here,

𝑖 𝑗 𝑘
𝜕 𝜕 𝜕
𝑐𝑢𝑟𝑙 𝑓 = 𝜕𝑥 𝜕𝑦 𝜕𝑧
=0𝑖+ 𝑗+2 𝑥−𝑦 𝑘
2 2
𝑦 𝑥 −(𝑥 + 𝑧)
Y

B (1,1,0)

X
O (0,0,0) A (1,0,0) X

Fig. 17.18

Here triangle is in the xy plane as z co-ordinate of each vertex of the triangle is zero.

∴ 𝑛=𝑘

∴ 𝑐𝑢𝑟𝑙 𝑓 . 𝑛 = 0 𝑖 + 𝑗 + 2 𝑥 − 𝑦 𝑘 . 𝑘 = 2 𝑥 − 𝑦

By Stoke’s theorem, 𝐶
𝑓 . 𝑑𝑟 = 𝑆
𝑐𝑢𝑟𝑙 𝑓 . 𝑛 𝑑𝑆

= 𝑆
2 𝑥 − 𝑦 𝑑𝑦 𝑑𝑥

Note here the equation of OB is 𝑦 = 𝑥, thus for 𝑆, x varies from 0 to 1 and 𝑦 from 0 to 𝑥.

55
𝑥
1 𝑥 1 𝑦2
∴ 𝐶
𝑓 . 𝑑𝑟 = 𝑥=0 𝑦=0
2(𝑥 − 𝑦) 𝑑𝑦 𝑑𝑥 = 𝑥=0
2 𝑥𝑦 − 𝑑𝑥
2 𝑦=0

1 𝑥2 1 1
= 𝑥=0
2 𝑥2 − 𝑑𝑥 = 𝑥=0
𝑥 2 𝑑𝑥 =3
2

Note: Green’s Theorem in plane is special case of Stoke’s Theorem: If R is the region in xy plane
bounded by a closed curve 𝐶 then this is a special case of Stoke’s theorem. In this case 𝑛 = 𝑘 and it
is called vector form of Green’s theorem in plane. Vector form of Green’s theorem can be written as

𝑅
∇ × 𝑓 . 𝑘 𝑑𝑅 = 𝐶
𝑓 . 𝑑𝑟

Example 65: Evaluate 𝑪


𝒚 − 𝐬𝐢𝐧 𝒙 𝒅𝒙 + 𝐜𝐨𝐬 𝒙 𝒅𝒚 , where 𝑪 is the triangle having vertices
𝝅 𝝅
𝟎, 𝟎 , , 𝟎 and ,𝟏 (i) directly (ii) by using Green’s theorem in plane [KUK 2011]
𝟐 𝟐

Solution:

(i) Here 𝐶
𝑦 − sin 𝑥 𝑑𝑥 + cos 𝑥 𝑑𝑦
= 𝐶
𝑦 − sin 𝑥 𝑖 + cos 𝑥 𝑗 . (𝑑𝑥 𝑖 + 𝑑𝑦 𝑗)
= 𝐶
𝑓 . 𝑑𝑟

where 𝑓 = 𝑦 − sin 𝑥 𝑖 + cos 𝑥 𝑗 and 𝑑𝑟 = 𝑑𝑥 𝑖 + 𝑑𝑦 𝑗 and


C is triangle OAB

Now 𝐶
𝑓 . 𝑑𝑟 = 𝐶
𝑦 − sin 𝑥 𝑑𝑥 + cos 𝑥 𝑑𝑦

= 𝑂𝐴
𝑦 − sin 𝑥 𝑑𝑥 + cos 𝑥 𝑑𝑦 + 𝐴𝐵
𝑦 − sin 𝑥 𝑑𝑥 + cos 𝑥 𝑑𝑦

+ 𝐵𝑂
𝑦 − sin 𝑥 𝑑𝑥 + cos 𝑥 𝑑𝑦 … (1)

On OA: 𝑦 = 0, ∴ 𝑑𝑦 = 0
𝜋 2
∴ 𝑂𝐴
𝑦 − sin 𝑥 𝑑𝑥 + cos 𝑥 𝑑𝑦 = 𝑂𝐴
− sin 𝑥 𝑑𝑥 = 𝑥=0
− sin 𝑥 𝑑𝑥 = −1
𝜋
On AB: 𝑥 = 2 , ∴ 𝑑𝑥 = 0

∴ 𝐴𝐵
𝑦 − sin 𝑥 𝑑𝑥 + cos 𝑥 𝑑𝑦 = 0

2 2
On BO: 𝑦 = 𝜋 𝑥, ∴ 𝑑𝑦 = 𝜋 𝑑𝑥

0 2 2
∴ 𝐵𝑂
𝑦 − sin 𝑥 𝑑𝑥 + cos 𝑥 𝑑𝑦 = 𝑥=𝜋 2
𝑥 − sin 𝑥 𝑑𝑥 + cos 𝑥 𝑑𝑥
𝜋 𝜋

0
2 𝑥2 2 𝜋 2
= + cos 𝑥 + 𝜋 sin 𝑥 =1− +𝜋
𝜋 2 𝑥=𝜋 2 4

56
Substituting these values in (1), we get
𝜋 2 𝜋 2
𝐶
𝑓 . 𝑑𝑟 = −1 + 0 + 1 − − = − +
4 𝜋 4 𝜋

(ii) By Green’s Theorem


𝜕𝑓2 𝜕𝑓1
𝐶
𝑓1 𝑑𝑥 + 𝑓2 𝑑𝑦 = 𝑆
− 𝑑𝑥𝑑𝑦
𝜕𝑥 𝜕𝑦
𝜋 2 2𝑥 𝜋 𝜋 2 2𝑥 𝜋
= 𝑥=0 𝑦=0
− sin 𝑥 − 1 𝑑𝑦 𝑑𝑥 = 𝑥=0
− sin 𝑥 − 1 𝑦 𝑦=0 𝑑𝑥

𝜋 2 2 2 𝜋 2
= 𝑥=0
− 𝑥 sin 𝑥 + 1 𝑑𝑥 = − 𝜋 𝑥=0
𝑥 sin 𝑥 + 𝑥 𝑑𝑥
𝜋
𝜋 2
2 𝑥2 2 𝜋2
= − 𝜋 𝑥 − cos 𝑥 + sin 𝑥 + = −𝜋 −0 + 1 + − (0 + 0 + 0)
2 𝑥=0 8

2 𝜋
=− +4
𝜋

Example 66: Verify Green’s theorem in the plane for 𝑪


𝒙𝒚 + 𝒚𝟐 𝒅𝒙 + 𝒙𝟐 𝒅𝒚, where 𝑪 is the
closed curve of the region bounded by 𝒚 = 𝒙 and 𝒚 = 𝒙𝟐 .

Solution:

𝐶
𝑥𝑦 + 𝑦 2 𝑑𝑥 + 𝑥 2 𝑑𝑦 = 𝑂𝐵𝐴
𝑥𝑦 + 𝑦 2 𝑑𝑥 + 𝑥 2 𝑑𝑦 + 𝑂𝐴
𝑥𝑦 + 𝑦 2 𝑑𝑥 + 𝑥 2 𝑑𝑦 … (1)

Along curve OBA: 𝑦 = 𝑥2 ∴ 𝑑𝑦 = 2𝑥 𝑑𝑥


1 19
∴ 𝑂𝐵𝐴
𝑥𝑦 + 𝑦 2 𝑑𝑥 + 𝑥 2 𝑑𝑦 = 𝑥=0
𝑥 3 + 𝑥 4 𝑑𝑥 + 2𝑥 3 𝑑𝑥 = 20

Along curve AO: 𝑦=𝑥 ∴ 𝑑𝑦 = 𝑑𝑥


0
∴ 𝐴𝑂
𝑥𝑦 + 𝑦 2 𝑑𝑥 + 𝑥 2 𝑑𝑦 = 𝐴𝑂
𝑥 2 + 𝑥 2 𝑑𝑥 + 𝑥 2 𝑑𝑥 = 𝑥=1
3𝑥 2 𝑑𝑥 = −1

19 1
∴ from (1), 𝐶
𝑥𝑦 + 𝑦 2 𝑑𝑥 + 𝑥 2 𝑑𝑦 = 20 − 1 = − 20 … (2)

Here 𝑓1 = 𝑥𝑦 + 𝑦 2 , 𝑓2 = 𝑥 2
𝜕𝑓1 𝜕𝑓2
⇒ = 𝑥 + 2𝑦, = 2𝑥
𝜕𝑦 𝜕𝑥

By Green’s theorem,
𝜕𝑓2 𝜕𝑓1
𝐶
𝑓1 𝑑𝑥 + 𝑓2 𝑑𝑦 = 𝑆
− 𝑑𝑦 𝑑𝑥
𝜕𝑥 𝜕𝑦

= 𝑆
2𝑥 − 𝑥 − 2𝑦 𝑑𝑦 𝑑𝑥

1 𝑦=𝑥 1 𝑦 =𝑥
= 𝑥=0 𝑦=𝑥 2
𝑥 − 2𝑦 𝑑𝑦 𝑑𝑥 = 𝑥=0
𝑥𝑦 − 𝑦 2 𝑦 =𝑥 2
𝑑𝑥

x=1
1 x5 x4 1 1 1
= 𝑥=0
𝑥 4 − 𝑥 3 𝑑𝑥 = − = 5 − 4 = − 20 … (3)
5 4 x=0

Equation (2) and (3) verify the result.


57
ASSIGNMENT 9

1. Verify Green’s theorem for 𝐶


3𝑥 − 8𝑦 2 𝑑𝑥 + 4𝑦 − 6𝑥𝑦 𝑑𝑦 where C is the boundary of the
region bounded by x  0, y  0 and x  y  1 . [KUK 2007]
2. Verify Green’s theorem in plane for 𝐶
3𝑥 2 − 8𝑦 2 𝑑𝑥 + 4𝑦 − 6𝑥𝑦 𝑑𝑦 , where C is the
boundary of the region defined by y  x and y  x 2 . [KUK 2008]
3. Apply Green’s theorem to evaluate 𝐶
2𝑥 2 − 𝑦 2 𝑑𝑥 + 𝑥 2 + 𝑦 2 𝑑𝑦 , where C is the boundary
of the area enclosed by x-axis and the upper half of the circle x 2  y 2  1 . [KUK 2010]

4. Evaluate the surface integral  curl F . nˆ dS by transforming it into a line integral, S being that
S

part of the surface of the paraboloid 𝑧 = (1 − 𝑥 2 − 𝑦 2 ) for which z  0 and F  y iˆ  z ˆj  x kˆ .
[KUK 2008]
5. Using Stoke’s theorem, evaluate 𝐶
𝑥 + 𝑦 𝑑𝑥 + 2𝑥 − 𝑧 𝑑𝑦 + 𝑦 + 𝑧 𝑑𝑧 , where C is the
boundary of the triangle with vertices (2, 0, 0) , (0, 3, 0) and (0, 0, 6) . [KUK 2009]
6. Verify Stoke’s theorem for a vector field defined by 𝑓 = −𝑦 3 𝑖 + 𝑥 3 𝑗, in the region
𝑥 2 + 𝑦 2 ≤ 1, 𝑧 = 0.

17.14 GAUSS’S DIVERGENCE THEOREM (Relation between Volume and Surface Integral)

Statement: Suppose 𝑉 is the volume bounded by a closed piecewise smooth surface S. Suppose
𝑓(𝑥, 𝑦, 𝑧) is a vector function which is continuous and has continuous first partial derivatives in
𝑉.Then

𝑉
∇. 𝑓 𝑑𝑉 = 𝑓 . 𝑛 𝑑𝑆

where 𝑛 is the outward unit normal to the surface 𝑆.

In other words: The surface integral of the normal component of a vector 𝑓 taken over a closed
surface is equal to the integral of the divergence of 𝑓 over the volume enclosed by the surface.

Divergence Theorem in Cartesian Coordinates


𝜕𝑓1 𝜕𝑓2 𝜕𝑓3
If 𝑓 = 𝑓1 𝑖 + 𝑓2 𝑗 + 𝑓3 𝑘 then 𝑑𝑖𝑣 𝑓 = ∇. 𝑓 = + + . Suppose 𝛼, 𝛽, 𝛾 are the angles made by
𝜕𝑥 𝜕𝑦 𝜕𝑧
the outward drawn unit normal with the positive direction of axes, then

𝑛 = cos 𝛼 𝑖 + cos 𝛽 𝑗 + cos 𝛾 𝑘

Now, 𝑓 . 𝑛 = 𝑓1 cos 𝛼 + 𝑓2 cos 𝛽 + 𝑓3 cos 𝛾

Then divergence theorem is


𝜕𝑓1 𝜕𝑓2 𝜕𝑓3
𝑉
+ + 𝑑𝑥 𝑑𝑦 𝑑𝑧 = 𝑆
𝑓1 cos 𝛼 + 𝑓2 cos 𝛽 + 𝑓3 cos 𝛾 𝑑𝑆
𝜕𝑥 𝜕𝑦 𝜕𝑧

= 𝑆
𝑓1 𝑑𝑦𝑑𝑧 + 𝑓2 𝑑𝑧𝑑𝑥 + 𝑓3 𝑑𝑥𝑑𝑦

∵ cos 𝛼 𝑑𝑆 = 𝑑𝑦𝑑𝑧 𝑒𝑡𝑐.

58
Proof: Let 𝑓 = 𝑓1 𝑖 + 𝑓2 𝑗 + 𝑓3 𝑘 where 𝑓1 , 𝑓2 and 𝑓3 and their derivatives in any direction are finite
and continuouos.

Suppose 𝑆 is a closed surface such that it is possible to


choose rectangular cartesian co-ordinate system such that
any line drawn parallel to coordinate axes does not cut 𝑆
in more than two points.

Let 𝑅 be the orthogonal projection of 𝑆 on the xy-plane.


Any line parallel to z-axis through a point of 𝑅 meets the
boundary of S in two points. Let 𝑆1 and 𝑆2 be the lower
and upper portions of 𝑆 . Let the equations of these
portions be

𝑧 = 𝛷1 (𝑥, 𝑦) and 𝑧 = 𝛷2 (𝑥, 𝑦)

where 𝛷1 (𝑥, 𝑦) ≥ 𝛷2 (𝑥, 𝑦)

Consider the volume integral


𝜕𝑓3 𝜕𝑓3 𝑧=𝛷 1 (𝑥,𝑦) 𝜕𝑓3
𝑉
𝑑𝑉 = 𝑑𝑥 𝑑𝑦 𝑑𝑧 = 𝑧=𝛷 2 (𝑥,𝑦) 𝜕𝑧
𝑑𝑧 𝑑𝑥 𝑑𝑦
𝜕𝑧 𝜕𝑧

𝜕𝑓3 𝛷 1 (𝑥,𝑦)
𝑉
𝑑𝑉 = 𝑓3 (𝑥, 𝑦, 𝑧) 𝛷 2 (𝑥,𝑦) 𝑑𝑥 𝑑𝑦 = 𝑓3 𝑥, 𝑦, 𝛷1 − 𝑓3 𝑥, 𝑦, 𝛷2 𝑑𝑥 𝑑𝑦 …(1)
𝜕𝑧

Let 𝑛1 be the unit outward drawn vector making an acute angle 𝛾1 with 𝑘 for the upper position 𝑆1 as
shown in the figure.

Now projection 𝑑𝑥 𝑑𝑦 of 𝑑𝑆1 on the 𝑥𝑦 plane is given as 𝑑𝑥𝑑𝑦 = 𝑑𝑆1 cos 𝛾 = 𝑑𝑆1 𝑘. 𝑛1 = 𝑘. 𝑛1 𝑑𝑆1

Now 𝑅
𝑓3 𝑥, 𝑦, 𝛷1 𝑑𝑥𝑑𝑦 = 𝑆1
𝑓3 𝑘. 𝑛1 𝑑𝑆1 … (2)

Similarly if 𝑛2 be the unit outward drawn normal to the lower surface 𝑆2 making an angle 𝛾2 with 𝑘.
Obviously 𝛾2 is an obtuse angle

∴ 𝑑𝑥𝑑𝑦 = 𝑑𝑆2 cos 𝜋 − 𝛾2 = −𝑑𝑆2 cos 𝛾2 = −𝑘. 𝑛2 𝑑𝑆2

∴ 𝑅
𝑓3 𝑥, 𝑦, 𝛷2 𝑑𝑥 𝑑𝑦 = − 𝑆2
𝑓3 𝑘. 𝑛2 𝑑𝑆2 … (3)

From (1), (2) and (3), we have


𝜕𝑓3
𝑉
𝑑𝑉 = 𝑆1
𝑓3 𝑘. 𝑛1 𝑑𝑆1 + 𝑆2
𝑓3 𝑘. 𝑛2 𝑑𝑆2 = 𝑆
𝑓3 𝑘. 𝑛 𝑑𝑆 … (4)
𝜕𝑧

Similarly by projecting 𝑆 on the other coordinate planes


𝜕𝑓2
𝑉 𝜕𝑦
𝑑𝑉 = 𝑆
𝑓2 𝑗. 𝑛 𝑑𝑆 … (5)

𝜕𝑓1
And 𝑉 𝜕𝑥
𝑑𝑉 = 𝑆
𝑓1 𝑖. 𝑛 𝑑𝑆 … (6)

59
Adding (4), (5) and (6), we get
𝜕𝑓1 𝜕𝑓2 𝜕𝑓3
𝑉
+ + 𝑑𝑉 = 𝑆
𝑓1 𝑖 + 𝑓2 𝑗 + 𝑓3 𝑘 . 𝑛 𝑑𝑆 = 𝑆
𝑓 . 𝑛 𝑑𝑆
𝜕𝑥 𝜕𝑦 𝜕𝑧

Note: With the help of this theorem we can express volume integral as surface integral or vice versa.

17.15 GREEN’S THEOREM (For Harmonic Functions)

Statement: If 𝛷 and 𝜓 are two scalar point functions having continuous second order derivatives in a
region 𝑉 bounded by a closed surface 𝑆, then

𝑉
𝛷∇2 𝜓 − 𝜓∇2 𝛷 𝑑𝑉 = 𝑆
𝛷 ∇𝜓 − 𝜓 ∇𝛷 . 𝑛 𝑑𝑆

Proof: By Gauss’s divergence theorem,

𝑉
∇. 𝑓 𝑑𝑉 = 𝑆
𝑓. 𝑛 𝑑𝑆 … (1)

Take 𝑓 = 𝛷 ∇𝜓 so that ∇. 𝑓 = ∇. 𝛷 ∇𝜓

= 𝛷 ∇. ∇𝜓 + ∇𝛷. ∇𝜓

= 𝛷∇2 𝜓 − ∇𝛷. ∇𝜓 … (2)

Now 𝑓 . 𝑛 = 𝛷 ∇𝜓 . 𝑛 . Using this and (2) in (1), we get

𝑉
𝛷 ∇2 𝜓 − ∇𝛷. ∇𝜓 𝑑𝑉 = 𝑆
𝛷 ∇𝜓 . 𝑛 𝑑𝑆 … (3)

Again starting as above by interchanging 𝛷 and 𝜓, we obtain as in (3)

𝑉
𝜓 ∇2 𝛷 − ∇𝜓. ∇𝛷 𝑑𝑉 = 𝑆
𝜓 ∇𝛷 . 𝑛 𝑑𝑆 … (4)

Subtracting (4) from (3), we get

𝑉
𝛷∇2 𝜓 − 𝜓∇2 𝛷 𝑑𝑉 = 𝑆
𝛷 ∇𝜓 − 𝜓 ∇𝛷 . 𝑛 𝑑𝑆 … (5)

Another Form of Green’s Theorem:


𝜕𝛷 𝜕𝜓
and denote the direction of derivative of 𝛷 and 𝜓 along the outward drawn normal at any point
𝜕𝑛 𝜕𝑛
of 𝑆.
𝜕𝛷 𝜕𝜓
∇𝛷 = 𝑛 and ∇𝜓 = 𝑛
𝜕𝑛 𝜕𝑛

𝜕𝜓 𝜕𝛷
∴ 𝛷 ∇𝜓 − 𝜓 ∇𝛷 = 𝛷 𝜕𝑛 𝑛 − 𝜓 𝜕𝑛 𝑛

𝜕𝜓 𝜕𝛷 𝜕𝜓 𝜕𝛷
or 𝛷 ∇𝜓 − 𝜓 ∇𝛷 . 𝑛 = 𝛷 𝜕𝑛 𝑛 − 𝜓 𝜕𝑛 𝑛 . 𝑛 = 𝛷 𝜕𝑛 − 𝜓 𝜕𝑛

∴ Equation (5) becomes


𝜕𝜓 𝜕𝛷
𝑉
𝛷∇2 𝜓 − 𝜓∇2 𝛷 𝑑𝑉 = 𝑆
𝛷 𝜕𝑛 − 𝜓 𝜕𝑛 𝑑𝑆

60
Example 67: Evaluate 𝑺
𝒇. 𝒏 𝒅𝑺 where 𝒇 = 𝟒𝒙𝒚 𝒊 + 𝒚𝒛 𝒋 − 𝒛𝒙 𝒌 and 𝑺 is the surface of the
cube bounded by the planes 𝒙 = 𝟎, 𝒙 = 𝟐, 𝒚 = 𝟎, 𝒚 = 𝟐, 𝒛 = 𝟎, 𝒛 = 𝟐.

Solution: Here 𝑓 = 4𝑥𝑦 𝑖 + 𝑦𝑧 𝑗 − 𝑧𝑥 𝑘 . By Gauss’s divergence theorem

𝑆
𝑓 . 𝑛 𝑑𝑆 = 𝑉
∇. 𝑓 𝑑𝑉

𝜕 𝜕 𝜕
= 𝑉
𝑖 𝜕𝑥 + 𝑗 𝜕𝑦 + 𝑘 𝜕𝑧 . 4𝑥𝑦 𝑖 + 𝑦𝑧 𝑗 − 𝑧𝑥 𝑘 𝑑𝑉

2 2 2
= 𝑉
4𝑦 + 𝑧 − 𝑥 𝑑𝑉 = 𝑥=0 𝑦=0 𝑧=0
4𝑦 + 𝑧 − 𝑥 𝑑𝑧 𝑑𝑦 𝑑𝑥

𝑧=2
2 2 𝑧2
= 𝑥=0 𝑦=0
4𝑦𝑧 + − 𝑥𝑧 𝑑𝑦 𝑑𝑥
2 𝑧=0

2 2
= 𝑥=0 𝑦=0
8𝑦 + 2 − 2𝑥 𝑑𝑦 𝑑𝑥

2 𝑦 =2 2
= 𝑥=0
4𝑦 2 + 2𝑦 − 2𝑥𝑦 𝑦 =0 𝑑𝑥 = 𝑥=0
20 − 4𝑥 𝑑𝑥

𝑥=2
= 20𝑥 − 2𝑥 2 𝑥=0 = 32

𝒂𝟓
Example 68: Use Gauss theorem to show that 𝑺
𝒙𝟑 − 𝒚𝒛 𝒊 − 𝟐𝒙𝟐 𝒚 𝒋 + 𝟐𝒌 . 𝒏 𝒅𝑺 = 𝟑

where S denotes the surface of the cube bounded by the planes, 𝒙 = 𝟎, 𝒙 = 𝒂, 𝒚 = 𝟎, 𝒚 = 𝒂,


𝒛 = 𝟎, 𝒛 = 𝒂 .

Solution: By Gauss’s divergence theorem

𝑆
𝑥 3 − 𝑦𝑧 𝑖 − 2𝑥 2 𝑦 𝑗 + 2𝑘 . 𝑛 𝑑𝑆 = 𝑉
∇. 𝑥 3 − 𝑦𝑧 𝑖 − 2𝑥 2 𝑦 𝑗 + 2𝑘 𝑑𝑉
𝑎 𝑎 𝑎
= 0 0 0
3𝑥 2 − 2𝑥 2 𝑑𝑥 𝑑𝑦 𝑑𝑧
𝑎
𝑎 𝑎 𝑎 𝑎 𝑎 𝑥3
= 0 0 0
𝑥 2 𝑑𝑥 𝑑𝑦 𝑑𝑧 = 0 0
𝑑𝑦 𝑑𝑧
3 0

𝑎
𝑎 𝑎 𝑎3 𝑎 𝑎3 𝑎 𝑎4 𝑎4 𝑎 𝑎5
= 0 0
𝑑𝑦 𝑑𝑧 = 0
𝑦 𝑑𝑧 = 0 3
𝑑𝑧 = 𝑧 0 =
3 3 0 3 3

Example 69: Evaluate 𝑺


𝒇. 𝒏 𝒅𝑺 with the help of Gauss theorem for 𝒇 = 𝟔𝒛 𝒊 + 𝟐𝒙 + 𝒚 𝒋 −
𝒙 𝒌 taken over the region 𝑺 bounded by the surface of the cylinder 𝒙𝟐 + 𝒛𝟐 = 𝟗 included
between 𝒙 = 𝟎, 𝒚 = 𝟎, 𝒛 = 𝟎 and 𝒚 = 𝟖.

Solution: 𝑓 = 6𝑧 𝑖 + 2𝑥 + 𝑦 𝑗 − 𝑥 𝑘
𝜕 𝜕 𝜕
∇. 𝑓 = 𝑖 𝜕𝑥 + 𝑗 𝜕𝑦 + 𝑘 𝜕𝑧 . 6𝑧 𝑖 + 2𝑥 + 𝑦 𝑗 − 𝑥 𝑘 = 1

By Gauss’s divergence theorem,

𝑆
𝑓. 𝑛 𝑑𝑆 = 𝑉
1 𝑑𝑥 𝑑𝑦 𝑑𝑧

61
3 8 9−𝑥 2 3 8 𝑧= 9−𝑥 2
= 𝑥=0 𝑦=0 𝑧=0
𝑑𝑧 𝑑𝑦 𝑑𝑥 = 𝑥=0 𝑦=0
𝑧 𝑧=0 𝑑𝑦 𝑑𝑥

3 8
= 𝑥=0 𝑦=0
9 − 𝑥 2 𝑑𝑦 𝑑𝑥

3 8 3
= 𝑥=0
9 − 𝑥2 𝑦 𝑦=0
𝑑𝑥 = 𝑥=0
8 9 − 𝑥 2 𝑑𝑥

3
𝑥 9−𝑥 2 9 𝑥
=8 + sin−1 3 = 18 𝜋
2 2
𝑥=0

Example 70: Evaluate 𝑺


(𝒙𝒅𝒚𝒅𝒛 + 𝒚𝒅𝒛𝒅𝒙 + 𝒛𝒅𝒙𝒅𝒚) where 𝑺 is the surface of the sphere
𝒙𝟐 + 𝒚𝟐 + 𝒛𝟐 = 𝒂𝟐 . [KUK 2011, 2009]

Solution: By Gauss’s divergence theorem


𝜕𝑥 𝜕𝑦 𝜕𝑧
𝑆
(𝑥𝑑𝑦𝑑𝑧 + 𝑦𝑑𝑧𝑑𝑥 + 𝑧𝑑𝑥𝑑𝑦) = 𝑉
+ 𝜕𝑦 + 𝜕𝑧 𝑑𝑥 𝑑𝑦 𝑑𝑧 = 𝑉
3 𝑑𝑥 𝑑𝑦 𝑑𝑧
𝜕𝑥

=3 𝑉
𝑑𝑥 𝑑𝑦 𝑑𝑧 = 3 × volume of the sphere x 2 + y 2 + z 2 = a2

4
= 3 × 3 πa3 = 4πa3

Example 71: Show that 𝑺


𝒏 𝒅𝑺 = 𝟎 for any closed surface 𝑺.

Solution: Let 𝐶 be any closed vector.

∴ 𝐶 𝑆
𝑛 𝑑𝑆 = 𝑆
𝐶. 𝑛 𝑑𝑆 = 𝑉
𝑑𝑖𝑣 𝐶 𝑑𝑉

∴ 𝐶 𝑆
𝑛 𝑑𝑆 = 0 ∵ 𝐶 is constant, therfore 𝑑𝑖𝑣 𝐶 = 0

⇒ 𝑆
𝑛 𝑑𝑆 = 0

𝟏 𝒓.𝒏
Example 72: Prove that 𝑽 𝒓𝟐
𝒅𝑽 = 𝒅𝑺, where 𝒓 = 𝒙 𝒊 + 𝒚 𝒋 + 𝒛 𝒌 and 𝒓 = 𝒓.
𝑺 𝒓𝟐

𝑟 .𝑛 𝑟 𝑟
Solution: 𝑑𝑆 = . 𝑛 𝑑𝑆 = ∇. 𝑑𝑉 … (1)
𝑆 𝑟2 𝑆 𝑟2 𝑉 𝑟2

𝑟 1 1
Now, ∇. = 𝑟 2 ∇. 𝑟 + 𝑟. ∇ … (2)
𝑟2 𝑟2

Also, 𝑟 =𝑥𝑖+𝑦𝑗+𝑧𝑘 ⇒ 𝑟2 = 𝑥2 + 𝑦2 + 𝑧2
𝜕𝑟 𝜕𝑟 𝜕𝑟
∴ 2𝑟 𝜕𝑥 = 2𝑥; 2𝑟 𝜕𝑦 = 2𝑦; 2𝑟 𝜕𝑧 = 2𝑧

𝜕𝑟 𝑥 𝜕𝑟 𝑦 𝜕𝑟 𝑧
⇒ =𝑟; =𝑟; =𝑟 … (3)
𝜕𝑥 𝜕𝑦 𝜕𝑧

1 𝜕 𝜕 𝜕 1 −2 𝜕𝑟 𝜕𝑟 𝜕𝑟
Now, ∇ = 𝑖 +𝑗 +𝑘 = 𝑖 +𝑗 +𝑘 using (3)
𝑟2 𝜕𝑥 𝜕𝑦 𝜕𝑧 𝑟2 𝑟3 𝜕𝑥 𝜕𝑦 𝜕𝑧

−2 −2
= 𝑥𝑖+𝑦𝑗+𝑧𝑘 = 𝑟
𝑟4 𝑟4

62
Also, ∇. 𝑟 = 1 + 1 + 1 = 3

Substituting these values in (2), we have


𝑟 1 2 3 2 1
∇. = 𝑟 2 . 3 + 𝑟. − 𝑟 4 𝑟 = 𝑟 2 − 𝑟 4 𝑟 2 = 𝑟 2 ∵ 𝑟. 𝑟 = 𝑟 2
𝑟2

𝑟 .𝑛 1
∴ From 1 , 𝑑𝑆 = 𝑑𝑉
𝑆 𝑟2 𝑉 𝑟2

ASSIGNMENT 10
 
1. Find  F . nˆ dS where F  (2 x  z ) iˆ  ( xz  y) ˆj  ( y 2  2 z) kˆ and S is the surface of the sphere
S

having centre (3,  1, 2) and radius 3 units. [KUK 2006]


  
2. Use Divergence theorem to evaluate  F . dS where F  x 3iˆ  y 3 ˆj  z 3 kˆ and S is the outer
S

surface of the sphere x  y  z  1 .


2 2 2
[KUK 2007]

3. Verify Divergence theorem for F  ( x 2  yz ) iˆ  ( y 2  zx) ˆj  ( z 2  xy ) kˆ taken over rectangular
parallelopiped 0  x  a, 0  y  b, 0  z  c. [KUK 2010]

ANSWERS

ASSIGNMENT 1
𝑎𝜋
1. −4(𝑖 + 2𝑗) 3. 𝑥 − 𝑎/ 2 = 𝑦 − 𝑎/ 2 = 𝑧 − 4
tan 𝛼 / 2 𝑡𝑎𝑛 𝛼

1
4. 𝑡 𝑖 + 2𝑗 − 2𝑡 − 3 𝑘 / (5𝑡 2 − 12𝑡 + 13) ; 3 (2 𝑖 + 2𝑗 + 𝑘 )

5. (a) 𝑢 𝑎2 sec 𝛼 (b) 𝑎3 tan 𝛼; (− cos 𝛼 sin 𝑡 𝑖 + cos 𝛼 cos 𝑡 𝑗 + sin 𝛼 𝑘 )


1 1
6. (a) 𝑝 𝑖 + 𝑝 + 2𝑞 𝑗 + 𝑝 + 𝑞 𝑘; 6
−𝑖 − 𝑗 + 2 𝑘 (b) 𝑝 + 𝑞 𝑖 + 𝑞 𝑗 + 2𝑞 𝑘 ;
5
2𝑗 − 𝑘

17. (a) 𝑎𝑏/ 𝑎2 sin2 𝑡 + 𝑏 2 cos2 𝑡 3/2


(b) 1/4 2

ASSIGNMENT 2

8 1 1 70 436
1. 𝑣 = 37 and 𝑎 = 325 at 𝑡 = 0 3. 7 14 ; 14 4. 𝑎 = ± 7. ;
7 6 29 29
0
8. 21.29 knots/hr. in the direction 74 47’ South of East 9. 17 meter per hour in the direction
tan−1 0.25 North of East

ASSIGNMENT 3
15 37 1
1. (a) 2(𝑥 𝑖 + 𝑦 𝑗 + 𝑧 𝑘 )/ (𝑥 2 + 𝑦 2 + 𝑧 2 ) 3. 4. 5. 3
(2 𝑖 + 2 𝑗 − 𝑘) 6. 11
17 3

1 8
7. cos−1 − 30
8. cos−1 1/ 22 9. cos −1 3 21
10. 𝜆 = 4 𝑎𝑛𝑑 𝜇 = 1

63
ASSIGNMENT 4

2. (a) 80 (b) 𝑒 𝑥𝑦𝑧 𝑥 𝑧 − 𝑦 𝑖 + 𝑦 𝑥 − 𝑧 𝑗 + 𝑧 𝑦 − 𝑥 𝑘

3. 𝑎 = −2; 4𝑥 𝑧 − 𝑥𝑦 𝑖 + 𝑦 1 − 2𝑧 + 4𝑥𝑦 𝑗 + (2𝑥 2 + 𝑦 2 − 𝑧 2 − 𝑧)𝑘

2𝑛 2𝑛 −1 1
9. (a) ; 𝑛=
𝑥 2 +𝑦 2 +𝑧 2 𝑛 +1 2

11. (i) 2 𝑦 3 + 3𝑥 2 𝑦 − 6𝑥𝑦 2 𝑧 𝑖 + 2 3𝑥𝑦 2 + 𝑥 3 − 6𝑥 2 𝑦 𝑧 𝑗 + 2 𝑥𝑦 2 + 𝑥 3 − 3𝑥 2 𝑦 𝑦 𝑘 (ii) Zero

13. (i) 0 (ii) 2 𝑥 + 𝑧 𝑗 + 2𝑦 𝑘

ASSIGNMENT 5
226
1. 𝑖 + 360 𝑗 − 42 𝑘 2. −2 𝑖 + 3 𝑗 − 3 𝑘
3

8𝑡 3
4. 𝑣 = 6 sin 2𝑡 𝑖 + 4(cos 2𝑡 − 1) 𝑗 + 8𝑡 2 𝑘 and 𝑟 = 3 1 − cos 2𝑡 𝑖 + 2 sin 2𝑡 𝑗 + 𝑘
3

ASSIGNMENT 6

8 𝜋3 2
1. 8 35 2. 16, 16 4. 35 5. 3

𝜋 1
6. 2 − 4 𝑖 − 𝜋 − 2 𝑗

ASSIGNMENT 7

2. 81 3. 3/2 5. 8

ASSIGNMENT 8

1. 128 2. 128𝑖 − 24𝑗 + 384𝑘

ASSIGNMENT 9

3. 4/3 5. 21

ASSIGNMENT 10

1. 108π 2. 56πa2/9

64

Vous aimerez peut-être aussi